You are on page 1of 235

300 MCQs 

Simulating the Jordanian 


licencing medical exams 
___ 

By Dr. Anas Alfakeer 

Explained MCQs  
This book contains 300 explained Multiple choice questions from the 4 major medical 
subjects (Internal Medicine, Surgery, Pediatrics, Gynecology and obstetrics), the MCQs are 
in a random order, this book will help any General Practitioner to pass the jordanian medical 
licencing exams (emtyaz) and other residency exams. 

Enjoy…….. 
1) 34y male presented with a painless, firm ulcer with indurated borders in his genitals, and
maculopapular rash, he is diagnosed with syphilis, a treatment with penicillin G is indicated
but a penicillin skin test reveals hypersensitivity to the substance. What is the alternative ?

A. Clarithromycin
B. Desensitize and treat with penicillin
C. Doxycycline
D. Rifampicin
E. Clindamycin

Treatment of syphilis:
● First-line therapy: benzathine penicillin G
○ Primary, secondary, or early latent: IM, single dose is sufficient
○ Late latent, tertiary, or date of transmission unknown: weekly IM injections over a
3-week course
○ Neurosyphilis: IV for 10–14 days
● If allergic to penicillin
○ If allergy is confirmed with skin testing:
■ Treat with doxycycline or ceftriaxone
■ In neurosyphilis: desensitize and treat with penicillin
■ During pregnancy: desensitize and treat with penicillin
● Sexual contacts should also be treated
● Notable complication of treatment: Jarisch-Herxheimer reaction

Resistance to macrolides, rifampicin, and clindamycin is often present.

Answer is : C

2) 32y old female athlete complains of dry mouth for the last 3 months she also complains of
fatigue and bilateral knee pain, physical examination reveals conjunctival injection and
itching in which she uses an eye moisturizing drops. Lip cracking is noticed, temperature is
36.8c Bp: 115/80. What is the most likely diagnosis ?

A. Septic arthritis.
B. Sjogren syndrome.
C. Viral conjunctivitis
D. Parotid malignancy.
E. Hodgkin lymphoma.

Sjogren syndrome is a chronic inflammatory autoimmune disease that occurs mainly in


middle-aged women. The cause of primary Sjogren syndrome is unknown, whereas
secondary Sjogren syndrome is associated with underlying autoimmune diseases (e.g.,
rheumatoid arthritis). As the immune system mainly attacks lacrimal and salivary glands,
patients typically present with xerophthalmia (dry eyes) and xerostomia (dry mouth), the
combination of which is also known as sicca syndrome. The disease may also involve the
skin, joints, internal organs, and nervous system. It is diagnosed via the detection of
autoantibodies (anti-SSA/Ro, ANA) and salivary gland biopsy. Management focuses on
supportive measures and, in more severe cases, immunosuppressants.

Answer is : B

3) 73y comes to the emergency department with severe dyspnea, O2 saturation is 81%, BP:
165/100. crackles are heard on auscultation in all lung fields. Medical history is remarkable
for Congestive heart failure. What is the next step in management to relieve his symptoms ?

A. Continuous positive airway pressure.


B. Beta blocker.
C. Intubation.
D. Ipratropium.
E. Thoracocentesis.

This patient is experiencing pulmonary edema caused by CHF. Continuous positive airway
pressure will increase his blood O2 Saturation thus relieving dyspnea the fastest.

Answer is : A

4) A 54y man from Amman known to be alcoholic, presented with pain and redness in his left
calf and Popliteal fossa, his knee is very painful to move, the redness is around the same
site he got injured a week ago by sharp stone in his yard, his temperature is 38c, the doctor
can feel crackles under the affected area of the skin, what is the likely diagnosis ?

A. Botulism.
B. Dermatitis.
C. DVT.
D. Cellulitis
E. Necrotizing fasciitis.

Necrotizing fasciitis
● Definition: rapidly progressive infection resulting in extensive necrosis of deep tissue
(subcutaneous tissue, fascia, and muscle) and systemic infection that can develop
into a life-threatening condition within hours!
● Pathogen: group A Streptococcus ; frequently mixed infection (with aerobic and
anaerobic gram-negative bacteria)
○ Gas gangrene is a type of necrotizing fasciitis caused by Clostridium
perfringens and is discussed in a separate learning card.
● Clinical features
○ Local findings
■ Initially diffuse redness, swelling, initially extreme tenderness
■ Later crepitus, purple skin discoloration, bullae formation, loss of
sensation in the affected area
○ Systemic symptoms: fever, chills, acute illness
○ Fournier's gangrene: necrotizing fasciitis of the external genitals

Answer is : E

5) A 72y male presented with a non-healing wound on his right forehead, what are the lymph
nodes this area drains into ?

A. Superficial parotid lymph nodes


B. Retroauricular lymph nodes
C. Preauricular lymph nodes
D. Submandibular lymph nodes
E. Occipital lymph nodes

The superficial parotid lymph nodes are a group of lymph nodes anterior to the ear.

Their afferent vessels drain the root of the nose, the eyelids, the frontotemporal region, the
external acoustic meatus and the tympanic cavity, possibly also the posterior parts of the
palate and the floor of the nasal cavity.

Answer is : A
6) A 55y male known to be alcoholic presented with progressive weakness and confusion,
Irritability, medical history is unremarkable, what is the most likely diagnosis ?

A. Pancreatitis.
B. Hepatocarcinoma.
C. Thiamine deficiency.
D. Anemia
E. Hepatic encephalopathy.

Most alcohol-related neurologic conditions are caused by deficiencies in thiamine (vitamin


B1)

Thiamine deficiency is a medical condition of low levels of thiamine (vitamin B1). A severe
and chronic form is known as beriberi. There are two main types in adults: wet beriberi, and
dry beriberi.

Early features:
● Fatigue and apathy.
● Irritability.
● Drowsiness, depression, poor concentration.
● Anorexia, nausea, vomiting, abdominal pain.
Later features:
● Paraesthesia, peripheral neuropathy, depressed tendon reflexes, loss of vibration
sense.
● Tender leg muscles and muscle cramps.
● Congestive heart failure with dyspnoea, orthopnoea and oedema.

Risk factors include a diet of mostly white rice, as well as alcoholism, dialysis, chronic
diarrhea, and taking high doses of diuretics.

Answer is : C

7) 28y old male complains of intermittent sudden severe dizziness last for seconds then
resolves on its own, accompanied with nausea, it is associated with movement of the head,
hearing is not impaired, What is the likely diagnosis ?

A. Vestibular neuritis
B. Ménière disease
C. Vasovagal syncope
D. Benign paroxysmal positional vertigo.
E. Panic disorder
Benign paroxysmal positional vertigo (BPPV) is a disease of the inner ear caused by small
particles (otoliths) dislodging and migrating within the endolymph fluid into one of the
semicircular canals. When provoked by certain head movements, these particles change
position and stimulate the vestibular system, which leads to episodes of vertigo generally
lasting less than a minute.

Clinical features
● Vertigo attacks
○ Sudden (“paroxysmal”) and recurrent
○ Triggered by certain head movements (“positional”)
○ Lasts several seconds (generally ≤ 1 minute)
○ A propensity to fall towards the healthy side

Answer is : D

8) 35y old woman presented with fatigue and lethargy, her Lab tests are as follows:
Hb : 10 g/dL
MCV: 71 fL
Serum ferritin: 9 μg/L
B12 serum: 340 ng/ml
What is the diagnosis ?

A. Thalassaemia
B. Iron deficiency anemia.
C. Macrocytic anemia
D. Leukemia.
E. Lymphoma.

Laboratory tests in IDA


● Low Haemoglobin
● Low Mean cell volume (MCV)
● Low Mean cell haemoglobin concentration (MCHC)
● High Red cell distribution width (RDW)
● Low serum ferritin
● High Total iron-binding capacity

Answer is : B

9) 57y old lady complains of severe itching in her vulva, physical examination reveals a white
patch on the vulva, she used antifungal vaginal cream, but no change in severity, she denied
any vaginal discharge, her past PAP smear a few months ago was normal, what is the most
likely diagnosis ?
A. Lichen sclerosus.
B. Toxoplasmosis.
C. Chlamydia.
D. Cervical cancer.
E. Vaginal atrophy.

Lichen sclerosus is a benign dermatologic condition of the anogenital region most commonly
seen in post-menopausal women.

Features of lichen sclerosus


The lesions are white thickened patches (porcelain-white papules and plaques). These may
progress to crinkled white patches (like cigarette paper). Active lesions may have areas of
ecchymosis, hyperkeratosis or bullae.

Women
Symptoms:
● Itch - can be severe and disturb sleep, as it is usually worse at night. This is usually
the first symptom.
● Pain can occur if there are fissures or erosions, leading to dyspareunia.
● Perianal lesions are common and may cause constipation.
● May be asymptomatic and found incidentally.
Signs:
● White lesions as above. These may be patchy, or in a figure-of-eight area around the
vulva and anus.
● Destructive scarring may cause shrinking of the labia, and narrowing of the introitus,
or the clitoris may be obscured by adhesions. Genital involvement does not occur;
the vagina and cervix are always spared.
● Perianal lesions occur in around 30% of cases.
● In girls, the signs may be mistaken for sexual abuse, as ecchymosis often occurs and
can be very striking.

Answer is : A
10) 14y old boy presented with a generalized rash (seen below), mild fever and muscle pain,
white spots are seen in his tongue in which he says it is painful, his mother claims that he
never had these symptoms before, what is the appropriate step in management ?

A. Erythromycin.
B. Calamine lotion.
C. Cetirizine
D. Corticosteroids.
E. Acyclovir.

Treatment of chickenpox

Symptomatic
● Pruritus: topical applications (e.g., calamine lotion or pramoxine gel) and, in more
severe cases, oral antihistamines (e.g., cetirizine)

Antiviral therapy
● Indication
○ Immunosuppressed individuals
○ Primary infection in adults and in unvaccinated adolescents ≥ 13 years
○ Individuals on long-term salicylate therapy (e.g., aspirin)
● Administration: within 24 hours of onset of rash
● Drug of choice: acyclovir (or also: valacyclovir, famciclovir)

Answer is : E
11) A 58y old man presented with ascites and caput medusae, he is a chronic alcohol
abuser, what is the most specific lab test to confirm the diagnosis ?

A. ESR
B. AST
C. ALP
D. ALT
E. BUN

Cirrhosis is characterized by fibrosis and disruption of normal liver architecture. ALT is


believed to be more specific than AST in diagnosing hepatic pathology.

Answer is : D

12) 10y old boy brought by his mother after severe itching on his face, the mother mentioned
they were hiking in the mountains not far from home 2 days ago, physical examination
shows slightly raised red rash, what types of reactions the boy is having ?

A. Type 1 hypersensitivity reaction.


B. Type 2 hypersensitivity reaction.
C. Type 3 hypersensitivity reaction.
D. Type 4 hypersensitivity reaction.
E. Burn.

Allergic contact dermatitis caused by poison ivy, oak or sumac presents as pruritic vesicles,
usually in a linear configuration. This is a type 4 hypersensitivity reaction or delayed
hypersensitivity reaction that is T-cell mediated and usually appears several days after
exposure.

Answer is : D

13) 42y old male complains of pregressive shortness of breath on exertion and chest
tightness for the past 2 months, examination shows a barrel chest and, wheezy chest, O2
saturation is 95%, B/P is 115/76, he is not a smoker, and work in a bank. The patient was
recently diagnosed with liver cirrhosis. Chest x-ray is shown down, What is the likely
diagnosis ?
A. Alpha 1-antitrypsin deficiency
B. Pneumonia.
C. Pneumothorax
D. Dilated cardiomyopathy
E. hypertrophic cardiomyopathy.

The patient is showing symptoms of COPD, which it can be caused by alpha 1-antitrypsin
deficiency, Emphysema in a young patient with no risk factors should raise the question of
alpha 1-antitrypsin deficiency, especially in the presence of cirrhosis, pancreatitis,
gallstones, bronchiectasis, or recurring spontaneous pneumothorax.

Answer is : A

14) 67y woman complains of epigastric pain and vomiting of blood, after an upper
endoscopy reveals the presence of MALToma, what is the initial treatment ?

A. Radiotherapy
B. Chemotherapy
C. Antibiotics
D. Surgery
E. NSAIDs

Mucosa-Associated Lymphoid Tissue (MALT) lymphoma (also called MALToma) is a B-cell


non-Hodgkin lymphoma (NHL) that typically affects elderly patients in the 7th and 8th
decades. MALTomas can be categorized according to their location as either gastric or non
gastric. Gastric MALTomas are frequently associated with Helicobacter pylori (H. pylori)
infection, whereas non gastric MALTomas are rather associated with autoimmune conditions
(e.g., Sjögren syndrome, Hashimoto's thyroiditis).
Treatment

Gastric MALTomas
● First-line: H. pylori eradication therapy
○ Should be performed even if patient tests negative
○ Eradication of H. pylori is curative in up to 90% of low-malignant gastric MALTomas
● If H. pylori eradication therapy fails → radiotherapy or chemotherapy
● Surgery: gastric resection only necessary if complications (e.g., perforation, bleeding,
obstruction) occur.

Non Gastric MALToma


● Depends on the exact type and staging
● Treatment options include : radiation, chemotherapy, surgery for local diseases

The prevalence of ​H. pylori​ infection in low-grade gastric MALTomas is up to 90%. This is supported
by the fact that gastric MALTomas frequently go into remission following H. pylori eradication therapy.

Answer is : C

15) 67y old man complains of dysphagia and dyspnea at night when lying in a supine
position, history is remarkable for rheumatoid arthritis, the patient looks well, auscultation
reveals Diastolic murmur, which of the following diagnostic tools is most important for
establishing diagnosis ?

A. Chest x-ray
B. Coronary CT angiography.
C. Barium swallow.
D. Echocardiography.
E. ECG.

Echocardiography is the most important diagnostic method for detecting and assessing
valvular abnormalities

Mitral stenosis (MS) is a valvular anomaly of the mitral valve that leads to obstruction of
blood flow into the left ventricle. The most common cause of MS is rheumatic fever. The
clinical manifestations depend on the extent of stenosis: reduced mitral opening leads to
progressive congestion behind the stenotic valve. Initial dilation of the left atrium
(complications: atrial fibrillations, emboli) is followed by progressive congestion of the lungs
and subsequent cardiac asthma (coughing, dyspnea). Acute decompensation can cause
pulmonary edema.
Diagnostics
● Auscultation
○ Diastolic murmur typically heard best at the 5th left intercostal space at the
midclavicular line (the apex)
■ Heard loudest when the patient is lying on his/her left side.
○ Loud first heart sound (S1)
○ Opening snap of the mitral valve after S2: A high-frequency, early-to-mid
diastolic sound that occurs when leaflet motion suddenly stops during diastole
after the stenosed valve has reached its maximum opening
■ Shorter interval between S2 and opening snap indicative of more
severe disease, because left atrial pressure is greater than left
ventricular end diastolic pressure (LVEDP)
● Chest x-ray
○ Posterior-anterior image
■ LA enlargement with prominent left auricle (left atrial appendage) →
straightening of the left cardiac border
■ Signs of pulmonary congestion
○ Lateral image
■ Dorsal displacement of the esophagus (visible in barium swallow test)
■ Signs of right ventricular hypertrophy
● ECG
○ P mitrale
○ Atrial fibrillation
○ Signs of right ventricular hypertrophy (Sokolow-Lyon index)
● Echocardiography: most important diagnostic method for detecting and assessing
valvular abnormalities
○ Abnormal valve mobility
○ Subvalvular thickening
○ Leaflet thickening
○ Calcification
● Coronary angiography may be conducted prior to surgical interventions to assess the
associated risk of coronary artery disease

Answer is : D

16) A 39y old male presented with persistent epigastric pain for three days that radiates to
the back with accompanying nausea, vomiting and oily diarrhea, BP is 95/63 Pancreatic
enzymes are slightly elevated, liver enzymes are in normal ranges, abdominal Ultrasound
revealed no abnormalities what is the next useful diagnostic tool to use?

A. CT abdomen.
B. MRCP..
C. ERCP.
D. X-ray.
E. CRP Levels.
Sensitivity of U/S to diagnose pancreatitis is often reduced to 70–80% because of overlying
bowel gas.

Imaging for diagnosis Acute pancreatitis:

Ultrasound (most useful initial test): indicated in all patients with acute pancreatitis
● Main purpose: detection of gallstones and/or dilatation of the biliary tract (indicating
biliary origin)
● Signs of pancreatitis
○ Indistinct pancreatic margins (edematous swelling)
○ Peripancreatic build-up of fluid ; evidence of ascites in some cases
○ Evidence of necrosis, abscesses, pancreatic pseudocysts
CT scan: not routinely indicated
● Indications
○ At admission: only when the diagnosis is in doubt (e.g., not very highly
elevated pancreatic enzymes, non-specific symptoms)
○ > 72 hours of symptom onset: if complications such as necrotizing
pancreatitis or pancreatic abscess (e.g., persistent fever and leukocytosis, no
clinical improvement or evidence of organ failure > 72 hours of therapy) are
suspected
● Findings
○ Enlargement of the pancreatic parenchyma with edema; indistinct pancreatic
margins with surrounding fat stranding
○ Necrotizing pancreatitis: lack of parenchymal enhancement or presence of air
in the pancreatic tissue
○ Pancreatic abscess: circumscribed fluid collection

Answer is : A

17) 49y old man with end-stage renal disease, his doctor indicated to be put on regular
dialysis for treatment, and sent to the surgery department for preparation of fistula creation,
what is the most preferred type of arteriovenous fistula ?

A. Loop LSV thigh fistula


B. Brachiocephalic fistula
C. Radiocephalic fistula
D. Brachial artery–to–transposed basilic vein fistula
E. None
Hemodialysis and hemofiltration

Preparation
● Large-bore venous catheter: insertion and catheter are similar to a central venous
catheter
● Arteriovenous fistula: anastomosis of an artery and a vein as a safe, large-bore
vascular access
○ Preferred location: radiocephalic fistula (Cimino fistula; anastomosis
between radial artery and cephalic vein in the distal forearm)
○ Should be provided early to ensure availability when needed
○ Usually constructed in the nondominant arm (less impairment)
○ Indication: maintenance dialysis in chronic kidney disease

Procedure/application
● Hemodialysis
○ Molecules diffuse across a semipermeable membrane down their
concentration gradient and are removed from the blood.
○ Superior at removing low-weight molecules (e.g., urea, protein-bound drugs,
ammonia)
○ Requires either a catheter (short-term option) or the creation and maturation
of a fistula (long-term option)
○ Common in the United States
● Hemofiltration
○ Molecules are filtered out by a semipermeable membrane, whereas fluid
passes through freely and re-enters the body (as “ultrafiltrate”).
○ Superior at removing middle-weight molecules (e.g., TNF, IL-8, IL-6)
○ Replacement fluid is required because significant amounts of fluid are wasted
in this process (“effluent”).
○ More common in Europe than the United States

Complications
● Arteriovenous fistulas
○ Local: thrombosis, stenosis, aneurysm, infection
○ Systemic: steal syndrome , heart failure
● Hypotension
● Bleeding
● Rarely: amyloidosis, allergic reactions to the equipment
● Dialysis disequilibrium syndrome
○ Pathophysiology: Because of the (rapid) extraction of osmotically active
substances (e.g., urea, NaCl) during dialysis, patients (especially when they
start on dialysis for the first time) can develop acute cerebral edema.
○ Symptoms
■ Nausea, vomiting
■ Disorientation, seizures
○ Prevention: regular and slow hemodialysis

Answer is : C

18) A 70 year-old man was admitted to the Emergency room in a state of shock. At the
presentation, he was pale, cold and sweaty, a filiform pulse with a pulse rate of 150 b/m,
central pulse rate of 150 b/m and the value of blood pressure measurement < 80/50 mmHg,
ECG showed wide QRS complexes with a pulse rate of 140b/m.
Laboratory tests showed:
K+ = 8.3 mmol/L, (N=3.5-5)
Na + = 135mmol/L, (N=135-145)
Creatinine = 1.0 mg/dl, (N=0.8-1.3)
Blood urea nitrogen = 250 mg/dl, (N=8-21)
Blood gas analysis testing showed:
pH = 7.29 7 (N=35-7.45)
PCO2 = 32mmHg (N=35-45)
PO2 = 95mmHg (N=75-100)

Calcium Gluconate was administered, what is the next step in management ?


A. Hemodialysis.
B. Sodium bicarbonate
C. Insulin
D. Beta-2-adrenergic agonists
E. Diuretics.

This patient is showing signs of renal failure and ECG changes.

Treatment of hyperkalemia
● Potassium level ≤ 6.5 mEq/L and no signs of cardiotoxicity: decrease intake/absorption
(slow-acting option)
○ Discontinue drugs that increase serum potassium
○ Avoid high-potassium foods
○ Cation-exchange resins (e.g., sodium polystyrene sulfonate): bind potassium in the
gut via the exchange of Na+ for K+
■ Adverse effect (rare): intestinal necrosis
○ Loop diuretics: promote excretion of potassium and lower total body potassium stores
○ Intravenous, non potassium containing fluids: normal saline, dextrose 5% in water
● Potassium level > 6.5 mEq/L or cardiotoxicity: IV therapy for cardioprotection and to induce
elimination/intracellular shift (rapid-acting option)
○ Calcium gluconate: should be administered first!
○ Insulin, preferably short-acting insulin, in combination with glucose
○ Sodium bicarbonate: in acidemic patients
○ Beta-2-adrenergic agonists
○ Forced diuresis (loop diuretics with normal saline solution)
● Renal failure or ineffective initial treatment

Answer is : A

19) A 63 years old former heavy smoker with a known case of chronic obstructive pulmonary
disease (COPD) presents to the emergency room with a history of shortness of breath and
cough which gradually increased in severity. He also had peripheral chest pain. He was
under medication for COPD for 3-4 years, his vitals are: spo2 - 91%, pulse: 103bpm. Chest
x-ray showed hyperinflated lungs, which of the following parameters is expected to be
increased ?

A. FEV1/FVC ratio
B. Total lung capacity.
C. pO2
D. Forced vital Capacity.
E. Transfer factor for carbon monoxide.

This patient is showing a signs of emphysema (Hyperinflated chest)


Diagnostics for COPD

Pulmonary function tests


● Spirometry and/or body plethysmography
○ FEV1/FVC < 70% (normal value is 70–85%)
○ Normal or decreased FVC
○ Decreased FEV1: used to classify COPD according to GOLD
○ Increased FRC and RV
○ Total lung capacity
■ Chronic bronchitis: normal
■ Emphysema: increased
○ Increased intrathoracic gas volume
○ Transfer factor for carbon monoxide
■ Chronic bronchitis: normal
■ Emphysema: decreased
● Post-bronchodilator test
○ Assesses reversibility of bronchoconstriction
○ Procedure
■ Spirometry to establish a baseline
■ Inhalation (e.g., salbutamol)
■ Perform spirometry again after ∼10–15 min.
○ Results: FEV1/FVC < 0.7 is diagnostic of COPD (in patients with typical
clinical features and exposure to noxious stimuli).
■ FEV1 > 12% (reversible bronchoconstriction): Asthma is more likely
than COPD.
■ If spirometry is normal, COPD can be excluded.
○ FEV1 < 12% (irreversible bronchoconstriction): COPD is more likely than
asthma.

Blood gas analysis (BGA) and pulse oximetry


● Pulse oximetry: assess O2 saturation
● BGA: only indicated when O2 is < 92% or if the patient is severely ill (e.g., altered
mental status, acute exacerbation)
○ Hypoxemia and hypercapnia with acute or chronic respiratory acidosis
■ Decreased pO2: partial respiratory failure
■ Decreased pO2 and increased pCO2: global respiratory failure
○ Possibly increased hemoglobin (polycythemia)

Imaging
● Indications: not required for routine diagnosis but often used as an initial modality,
mainly to rule out alternative conditions (e.g., pulmonary fibrosis, bronchiectasis)
● Chest x-ray
○ Not sensitive, especially during the early stages of COPD
○ Can be used to determine the etiology for an acute COPD exacerbation (e.g.,
pneumonia, congestive heart failure)
○ Signs of hyperinflated lungs (barrel chest)
■ Hyperlucency of lung tissue (decreased lung markings)
■ Increased anteroposterior diameter
■ Diaphragm pushed down and flattened
■ Horizontal ribs and widened intercostal spaces
■ Long narrow heart shadow
■ Parenchymal bullae or subpleural blebs (pathognomonic of
emphysema)
■ The retrosternal space is increased on lateral view due to
emphysematous changes in the lung tissue.
● Chest CT
○ Evaluate possible complications (e.g., pneumothorax, ARDS).
○ Plan surgery (e.g., lung volume reduction, lung transplantation).
○ Rule out differential diagnoses (e.g., bronchiectasis, lung cancer).
○ In most patients with COPD: centriacinar emphysema
○ In patients with AATD: panacinar emphysema , bronchiectasis, bullae

Other tests
● Laboratory studies
○ Increased serum hematocrit
○ In patients suspected of AATD (≤ 50 years of age, hepatic symptoms):
■ α1-Antitrypsin levels
■ Electrophoresis: decreased alpha-1 peak
● Gram stain and sputum culture: in the case of suspected pulmonary bacterial
infection (e.g., fever, productive cough, new infiltrate on chest x-ray)
● Bronchoscopy: to identify the pathogen in severe and acute exacerbation of COPD
with infective etiology, especially if antibiotic treatment fails
● Liver biopsy in patients with AATD: PAS-positive, spherical inclusion bodies in
periportal hepatocytes

Answer is : B

20) A 63-year-old woman with no medical history, had worsening chest pain with exercise for
the past 4 years, tests were done and ruled out any coronary diseases. Stress-ECG is
showing the following diagram:
Which of the following diagnoses is most likely?

A. Heart block mobitz II


B. Prinzmetal angina
C. Hypertrophic heart failure.
D. Left bundle branch block.
E. Hyperkalemia

Signs of left bundle branch block (LBBB) are primarily seen in leads I, V5,6
● Prolonged QRS complex
● Broad, notched R wave
● Loss of Q waves
● Possible rSR' formation in V5 or 6
● Deep S wave in V1,2
● Final negativity (intrinsicoid deflection) in V5,6 after > 0.05

Answer is : D

21) A 67-year-old man with a history of chronic obstructive pulmonary disease (COPD),
hypertension, and dyslipidemia was brought into the ED by his daughter with a 4-day history
of worsening shortness of breath, wheezing, and increased productive cough. He was
experiencing dyspnea on exertion. No fever, night sweats, chest pain, or palpitations. Vital
signs were as follows: blood pressure, 110/60 mm Hg; heart rate, 110/min; temperature, 37
°C; respiratory rate, 34/min; oxygen saturation, 81% on room air Arterial blood gases
showed a pH of 7.20, a Paco2 of 67 mmHg, a Pao2 of 61 mmHg, and an HCO3 of 30
mmol/L on nasal cannula at 2 L/min. White blood cell count was 14,000 cells/µL, hemoglobin
level was 15.1 g/dL, and hematocrit was 48%, which of the following is not used in the
treatment ?

A. Ipratropium bromide
B. Bilevel positive airway pressure
C. Albuterol
D. Bisoprolol.
E. Intubation

Bisoprolol is contraindicated in the management of COPD.

Acute exacerbation of chronic obstructive pulmonary disease


(AECOPD)
● Definition: an acute worsening of respiratory symptoms (e.g., increased dyspnea,
changes in sputum consistency)

Treatment
● Oxygen supplementation: if O2 < 92%, but be mindful that inappropriate O2 therapy
poses a risk of life-threatening hypercapnia (CO2 narcosis)
● Medical treatment
○ Beta agonists: inhaled SABA (e.g., albuterol, levalbuterol) PLUS
○ Anticholinergics: SAMA (e.g., ipratropium bromide)
○ Possibly add:
■ Systemic glucocorticoids: prednisolone (oral) or methylprednisolone
(IV) in severe treatment-refractory exacerbations
■ Antibiotic treatment is indicated for patients with ≥ 2 of the following
cardinal symptoms:
■ Increased dyspnea
■ Increased cough
■ Increased sputum production
■ Antibiotic treatment protocol in exacerbated COPD is the same as that
for bacterial pneumonia and involves aminopenicillin in combination
with clavulanic acid, macrolides, or tetracycline. It should cover
Haemophilus influenzae, Streptococcus pneumonia, and Moraxella
catarrhalis.
■ In the case of suspected of Pseudomonas aeruginosa infection:
piperacillin-tazobactam, cefepime, ceftazidime, or levofloxacin
● Mechanical ventilation: indicated for patients with severe AECOPD
○ Noninvasive ventilation (NIV): most commonly NIV with bilevel positive airway
pressure (BiPAP)
○ Invasive ventilation (i.e., intubation): only if NIV is contraindicated or not
tolerated

Answer is : D
22) A 23 years old male complains about a sore throat and fever for the last 10 days and it is
not getting better, physical examination shows: tonsils that are enlarged and covered with
pus, and enlarged and tender cervical lymph nodes, lab tests show a viral infection, what is
the most likely causative organism?

A. Norovirus.
B. Epstein-Barr virus.
C. Cytomegalovirus.
D. Respiratory syncytial virus.
E. Adenovirus.

Infectious mononucleosis (IM), also called "mono" or the "kissing disease", is an acute
condition caused by the Epstein-Barr virus (EBV). The disease is highly contagious and
spreads via bodily secretions, especially saliva.

Clinical features
● Incubation period: 4–8 weeks
● Clinical course
○ Symptoms typically occur in adolescents and young adults and last for 2–4 weeks.
○ Young children are often asymptomatic.
● Splenomegaly (50% of cases), fever, fatigue, malaise
● Pharyngitis and/or tonsillitis (reddened, ​enlarged tonsils covered in pus​); palatal petechiae
● Bilateral cervical lymphadenopathy (especially posterior) that may become generalized and
can, in severe cases, lead to airway obstruction
● Abdominal pain
● Possibly hepatomegaly and jaundice
● Maculopapular rash (similar to measles): caused by the infection itself in about 5% of cases,
but is generally associated with the administration of aminopenicillin (e.g., ampicillin or
amoxicillin)

Answer is : B

23) 32y old woman presented to your office asking about measures for preventing
pulmonary embolism (PE), she mentioned that her mom died with this condition, her BMI is
31, non smoker, medical history is unremarkable, what would you recommend her to prevent
DVT other than exercise?

A. Aspirin.
B. Compression stockings.
C. Enoxaparin.
D. Inferior vena cava filter.
E. Unfractionated heparin.
Deep Vein thrombosis is the leading cause of PE

This patient has only 1 risk factor for DVT: obesity, thus she is at low risk

● Aspirin or antiplatelet agents should not be considered adequate prophylaxis.

Risk factors for deep vein thrombosis:


● History of DVT or PE (30x increased risk)
● Immobilization: e.g., post-surgery, long-distance flights, trauma (20x increased risk)
● Age > 60 years
● Malignancy
● Hereditary thrombophilia (especially factor V Leiden)
● Pregnancy, estrogen use (oral contraceptives)
● Obesity
● Smoking
● IV drug use
● Nephrotic syndrome
● Insufficient thrombosis prophylaxis, noncompliance with prophylaxis

Prevention
● Preventive measures
○ Regular exercise (most important preventive measure)
○ Early mobilization (e.g., post-surgery)
○ Compression stockings and intermittent compression
○ (Postoperative) anticoagulation with LMWH or unfractionated heparin
○ Avoid certain medications (e.g., OCPs) in patients with thrombophilia (e.g.,
factor V Leiden)
● Indications
○ Low thrombosis risk: exercise, ​compression stockings
○ Medium and high thrombosis risk: anticoagulation treatment

Answer is : B

24) A 4-year-old male child presented with high temperature for the last week after his
brother developed chickenpox, History was not remarkable. In his pedigree chart, his
brother, one of his maternal uncles, and his maternal aunt’s two sons died because of
infections in early infancy. He had an obvious right foot drop and muscle wasting in his right
leg which developed after he was administered oral polio vaccine when he was 1.5 years
old, physical examination shows absent tonsils, flow cytometry was done and showed low B
cells, what is the most important management for this child ?
A. Antibiotics.
B. Stem Cells transplant.
C. IVIG.
D. Paracetamol.
E. MMRV vaccine.

The history and lab tests indicate this boy has Bruton’s agammaglobulinemia.
IVIG and prophylactic antibiotics are given to compensate for low circulating
immunoglobulins.
Children with Bruton’s agammaglobulinemia usually fight-off measles and varicella zoster
viruses without complications.

Bruton agammaglobulinemia (X-linked agammaglobulinemia)


● Definition: X-linked recessive disease that causes a complete deficiency of mature B
lymphocytes
● Epidemiology: occurs mainly in boys
● Etiology: defect of Bruton tyrosine kinase (BTK) expressed in B cells
● Clinical features
○ Onset is > 4 months after birth because of the continual decrease of maternal
IgG in fetal serum.
○ Hypoplasia of tonsils and other lymphoid tissue
○ Recurrent, severe pyogenic infections (e.g., pneumonia, otitis media),
especially with encapsulated organisms (S. pneumoniae, N. meningitidis, and
H. influenzae)
○ Hepatitis virus and enterovirus (e.g., Coxsackie virus) infection
● Diagnosis
○ Flow cytometry
■ Absent or low levels of B cells (marked by CD19, CD20, and CD21)
■ Normal or high T cells
○ Absent lymphoid tissue (e.g., tonsils)
● Treatment
○ IV immunoglobulins
○ Prophylactic antibiotics

Live vaccines (e.g., MMR) are contraindicated in patients with Bruton agammaglobulinemia.

Answer is : C

25) A 40-year-old male presented with abdominal discomfort for 1 year, No history of fever
or weight loss. Abdominal ultrasound shows mass of 3cm in diameter suggesting cystic
swelling in the liver. Routine investigations were completed. (Hemoglobin 12 g/dl) (WBC
9000cells/mcL) and (eosinophils 8%), the elisa test was positive for echinococcosis IgG,
what is the next step in management ?
A. Albendazole.
B. Surgical removal of cyst.
C. Ampicillin IV.
D. Radiotherapy
E. CT-guided percutaneous drainage.

Human echinococcosis, also known as hydatidosis or hydatid disease, is a parasitic disease


caused by small tapeworms of the genus Echinococcus. The two most common forms of
hydatidosis are cystic echinococcosis (CE), caused by E. granulosus, and alveolar
echinococcosis (AE), caused by E. multilocularis.

Treatment

Cystic echinococcosis
● Observation: inactive cyst with heterogeneous hypoechoic/hyperechoic contents, or
solid, calcified wall
● Medical therapy: may be considered as the sole treatment for cysts < 5 cm
○ Drug of choice: albendazole
○ Alternative drugs: mebendazole, praziquantel
● Ultrasonography/CT-guided percutaneous drainage
○ Should only be done in combination with medical therapy (albendazole)
● Surgery
○ Goal: resect the whole cyst to prevent spillage of its content
○ Indications: > 10 cm, complicated cysts
● Follow-up: Because relapse is common, patients should be closely monitored via
imaging for up to five years.

Any invasive procedure (drainage or surgery) of hydatid cysts should be performed with the
utmost care to prevent spillage of cyst contents, which could cause life-threatening
anaphylactic shock and/or secondary seeding of infection!

Answer is : A

26) A 62-year-old male patient approached the surgical unit of the hospital with a painful,
foul smelling, non-healing, necrotizing ulcer above the ankle since one month. He has had a
known case of diabetes since 10. Physical examination shows varicose veins in both legs,
His distal pulse is palpable, what is the next step in investigation ?
A. Magnetic resonance angiography.
B. Duplex Ultrasound.
C. Ascending venography.
D. D-dimer.
E. Transcutaneous oxygen tension.

Since distal pulse is intact the ulcer is most likely venous

Investigations for venous ulcer:


● The diagnosis of venous ulcers is clinical, with the underlying venous insufficiency
confirmed by ​Duplex Ultrasound ​(diagnostic tool of choice).

● Ascending venography also may be considered to obtain detailed anatomic


information.

● An Ankle Brachial Pressure Index (ABPI) is required to assess for any arterial
component to the ulcers and to determine whether compression therapy will be
suitable. If infection is suspected (i.e. erythematous or with purulent exudate) then
consider microbiology swabs and antibiotics.

● Take swab cultures if suspecting an associated infection. Consider a thrombophilia


and vasculitic screening in young patients, especially if there is a suspicion or family
history of prothrombotic and autoimmune diseases.

Answer is : B

27) 28y old male presented to the emergency department complaining of severe rectal pain
that started after defecation, he experienced less severe pain on defecation for months, he
admits he is dependant on fast foods for diet, he complains of hard fecal matter with bright
blood as he described, physical examination of the anus shows dark skin hanging from the
anus about 1cm, the initial treatment of this condition is ?

A. Surgical removal of the skin.


B. Fiber rich diet.
C. Reassurance and discharge.
D. Botulinum toxin A (BTX) injection.
E. Anal advancement flap.

An anal fissure is a longitudinal tear of the perianal skin distal to the dentate line, often due
to increased anal sphincter tone. Anal fissures are classified according to etiology (e.g.,
trauma or underlying disease) or duration of disease (e.g., acute or chronic). They are
typically very painful and may present with bright red blood per rectum (hematochezia). Anal
fissures are a clinical diagnosis based on history and examination findings. Management is
primarily conservative.
Treatment

Conservative
● First‑line treatment for most anal fissures
● Includes:
○ Dietary improvement (e.g., adequate ingestion of dietary fiber and water)
○ Stool softeners (e.g., docusate)
○ Anti‑inflammatory and analgesic creams and/or suppositories (e.g., 2%
lidocaine jelly)
○ Sitz baths
○ Local anesthetic injection
○ Topical vasodilator therapy: calcium channel blocker gel (e.g., nifedipine) or
glyceryl trinitrate ointment (GTN)

Interim
● Persistent symptoms despite > 8 weeks of conservative therapy → endoscopy to
exclude IBD
● If IBD is excluded, then the patient should receive definitive surgical treatment.

Outpatient procedures
● Botulinum toxin A (BTX) injection into the internal anal sphincter

Surgical
● Indicated when conservative treatment is unsuccessful
● The risk of fecal incontinence (e.g., high in multiparous or elderly patients)
determines the type of surgical intervention.
○ Low risk
■ Sphincterotomy (e.g., lateral internal sphincterotomy)
■ Anal dilatation (although there is a high risk of fecal incontinence with
this procedure)
○ High risk
■ Anal advancement flap
■ Fissurectomy (excision of the fissure)

Answer is : B

28) A 24-year-old woman presented with Right-sided flank pain during micturition. She
complained of recurrent urinary tract infections in the past. A physical examination and
laboratory tests were normal, cystourethrogram is shown below, what is the diagnosis ?
A. Cystitis.
B. UTI.
C. Struvite stone
D. Renal agenesis
E. Vesicoureteral reflux.

Vesicoureteral reflux (VUR) is the retrograde flow of urine from the bladder into the ureter.
Primary VUR is the most common type and is due to a congenital defect of the terminal
portion of the ureter. Bladder outlet obstruction, cystitis, and congenital ureteral anomalies
(e.g., ureteral duplication, ectopic ureter) may cause secondary VUR. Children with VUR are
usually asymptomatic until they develop a urinary tract infection (fever, dysuria, urgency.).
Other symptoms include flank pain and hypertension, uremia, and kidney failure in advanced
cases of reflux nephropathy. The initial workup for VUR includes laboratory tests (creatinine
levels, electrolytes) and renal ultrasound for evaluation of kidney function and possible
structural damage. Voiding cystourethrogram is the diagnostic test of choice for
demonstrating urinary reflux and the severity of the disease.

Answer is : E

29) A 64-year-old woman presented with a 2-day history of fever. Physical examination
showed scleral icterus and right upper quadrant tenderness without inspiratory arrest at
palpation (absent Murphy's sign). Laboratory workup revealed leukocytosis (12.4 × 103 μL),
CRP (8.3 mg/dL), (bilirubin 5.4 mg/dL), (alkaline phosphatase 893 U/L), (GGT 1143 U/L)
(AST 231 U/L), (ALT 178 U/L). Abdominal ultrasound demonstrated an impacted large
gallstone in the common bile duct with dilated common and intrahepatic bile ducts. What is
the most appropriate treatment ?

A. Antibiotics.
B. Cholecystectomy.
C. Percutaneous transhepatic cholangiography
D. ERCP.
E. NSAIDs.

Treatment of cholangitis
● Approach: supportive therapy, intravenous antibiotics → urgent biliary
decompression → prevention of recurrence with interval cholecystectomy if there is
concurrent cholelithiasis
● Supportive therapy
○ Analgesia: NSAIDs, opioids
○ Fluid and electrolyte correction
● Intravenous antibiotics.
● Biliary drainage and decompression
○ Timing: perform within 24–48 hours
○ ERCP is the treatment of choice, possibly in combination with:
■ Sphincterotomy (for cholangitis due to cholelithiasis or
choledocholithiasis) or stent placement (acute suppurative cholangitis)
○ Percutaneous transhepatic cholangiography (PTC) if ERCP is unsuccessful
or unavailable
■ In this procedure, contrast material is injected into the bile ducts
through the insertion of a transhepatic needle.
■ PTC allows therapeutic interventions such as infected bile drainage
and biliary tract stone extraction.
○ Surgical decompression if no other route feasible
● Prevention of recurrence
○ Interval cholecystectomy if gallstones are present
■ Timing: recommended within 6 weeks

Answer is : D

30) 22y male presented to emergency after severe pain while having sex, he felt a popping
sound, physical examination shows swelling of the penis, he mentioned that he urinated
witout any problem, what is the most likley injured structure ?

A. Tunica albuginea.
B. Urethra
C. Superficial dorsal vein.
D. Epididymis
E. Testis.

Penile fracture refers to the traumatic rupture of the tunica albuginea and the corpora
cavernosa of the penis. It is a rare condition that results from traumatic bending of the erect
penis, typically during sexual intercourse or masturbation. Penile fractures are usually
present with acute pain and a cracking sound, accompanied by immediate loss of erection,
as well as curving, swelling, and hematoma of the penis. In most cases, the diagnosis is
established clinically, but imaging methods (e.g., ultrasound, MRI) may be needed in
ambiguous cases.

Answer is : A

31) A 34y man rescued from a fire in his house, he has dyspnea and his mouth is filled with
black dust, what is the next step in management ?

A. Intubation.
B. Cricothyroidotomy.
C. Refer to the burn unit.
D. Give oxygen.
E. CPAP.

If there is any evidence of airway obstruction after burns, or there was suspicion of
inhalational burns. Intubation is necessary as soon as possible.

Answer is : A

32) 52 years old male presented with a jaundice for 3 days with oily stools, he admits
complaining of a dull, intermittent left back pain for the past 2-3 years and losing 10 Kg
weight, physical examination shows palpable gallbladder, no tenderness, lab test shows high
serum glucose levels, medical history is unremarkable, what is the most likely diagnosis?
A. Chronic pancreatitis.
B. Pancreatic pseudocyst.
C. Pancreatic cancer.
D. Hepatocellular carcinoma.
E. Colon cancer.

If history of primary sclerosing cholangitis with palpable gallbladder + obstructive jaundice


then it is more likely a Klatskin tumor
Otherwise any palpable GB with obstructive jaundice is periampullary Cancer .

This picture goes with Pancreatic head cancer that obstructs the ampulla of vater

Clinical features of Pancreatic cancer:


● Often no early signs present!
● Belt-shaped epigastric pain which may radiate to the back
● Jaundice
○ Courvoisier sign: enlarged gallbladder and painless jaundice
○ Pale stools, dark urine, and pruritus
● Weight loss, nausea, weakness, poor appetite
● Diarrhea (possibly steatorrhea secondary to exocrine pancreatic insufficiency)
● Superficial thrombophlebitis (in 10% of cases, also called Trousseau syndrome or
thrombophlebitis migrans)
○ Recurring thrombophlebitis in various locations
○ Classically associated with pancreatic cancer
● Thrombosis (e.g., phlebothrombosis, splenic vein thrombosis)
● Impaired glucose tolerance (rarely)

Answer is : C

33) An 18-year-old man was involved in a high-speed collision in which his motorcycle struck
a bull cart. 30 minutes later, he was admitted to the emergency room. At the presentation, he
was confused, violent, and complaining of thoracic and abdominal pain. He developed
hypotension 70/55, pulse rate: 120 bpm,respiratory rate: 35 breaths per minute. Bruise
marks were found on his right flank as well as on his chest. His extremities were cold with
marked peripheral hypoperfusion. his external jugular veins were distended. Cardiovascular
examination revealed auscultated and muffled dual heart sounds with no cardiac murmur.
Breath sounds are clear on both lungs, What is the gold standard diagnostic tool to confirm
the diagnosis ?

A. CT scan
B. MRI.
C. Echocardiography.
D. Chest X-ray
E. ECG.
Echocardiography is a quick and safe diagnostic tool for detecting pericardial effusions and
pericardial tamponade!

Pericardial effusion ​is the acute or chronic accumulation of fluid in the pericardial space
(between the parietal and the visceral pericardium) and is often associated with a variety of
underlying disorders. The fluid can be either bloody (e.g., following aortic dissection) or
serous (usually idiopathic). As the pericardium is rather stiff, the capacity of the pericardial
space is limited. In chronic effusion, the pericardium can stretch to a certain degree,
accommodating slightly more fluid. In the acute setting, however, the added volume quickly
exceeds the maximum capacity of the pericardial space. In both cases, the end result is
often cardiac tamponade: compression of the heart which can lead to a life-threatening
reduction in cardiac output. Pericardial effusion is initially asymptomatic, but cardiac
tamponade has a distinct clinical presentation, including ​hypotension, tachycardia, jugular
venous congestion, and pulsus paradoxus​. Echocardiography is ​the most important
diagnostic procedure​ and usually reveals an anechoic pericardial space. Treatment
depends on hemodynamic stability: unstable patients require quick pericardial fluid drainage,
through either pericardiocentesis or surgery, whereas in stable patients, treatment focuses
on the underlying disease.

Answer is : C

34) 62 years old male presented to emergency department complaining of sudden left chest
pain, and dyspnea for the last 2 days, Pain was constant, worse with inspiration, history is
remarkable for COPD, he says the pain is not resolving despite using his inhaler,
auscultation reveals no breath sounds, on the left side, blood pressure: 125/83, chest x-ray
shows a space between the lung and chest wall of 2cm, what the most important
management for this patient ?

A. Analgesics.
B. Needle aspiration
C. Observation
D. Oxygen
E. Chest tube.

This is a scenario of small symptomatic secondary spontaneous pneumothorax.

Approach to pneumothorax
● Simple pneumothorax
○ If small (≤ 2 to 3 cm between the lung and chest wall on a chest x-ray) and
asymptomatic
■ Usually resolve spontaneously within a few days (∼ 10 days)
■ Supplemental oxygen (4-6 L/min) via nasal cannula or mask with
reservoir
■ Serial follow-up with repeat CXR
○ If small and symptomatic (but hemodynamically stable) or large (> 3 cm
between the lung and chest wall on chest x-ray) primary pneumothorax,
iatrogenic, traumatic, or secondary pneumothorax
■ Immediate supplemental oxygen (4-6 L/min) via nasal cannula or
mask with reservoir
■ Upright positioning
■ Symptomatic treatment
■ Tube thoracostomy
● Open pneumothorax
○ Simple partially occlusive dressings taped at 3 out of 4 sides of the lesion
○ Followed by thoracostomy
○ Observe for development of tension pneumothorax
● Tension pneumothorax
○ Emergency chest decompression via chest tube placement if immediately
available
○ Otherwise perform emergency needle thoracostomy, followed by chest tube
placement

Answer is : E

35) 52 years old male presented with painless mass just below the mandibular bone
posteriorly on the left, ultrasound of the neck reveals normal neck components except for the
enlarged posterior cervical lymph node, no other symptom is apparent, fine needle aspiration
of the enlarged lymph nodes reveals follicular cells, what is the diagnosis ?

A. Thyroid cancer.
B. Ectopic thyroid.
C. Goitre
D. Jugular vein distention.
E. Lymphadenitis.

In rare circumstances, abnormal lymph nodes will be seen in follicular thyroid cancer and
FNA of those lymph nodes will exhibit "follicular cells"

Answer is : A

36) 24 years old male encountered a vehicle collision 3 months ago, ever since he is unable
to walk steadily and legs are apart from normal. He also complains of vertigo and slurred
speech, he can’t write ever since the trauma, where is the lesion ?

A. Cerebellum.
B. Frontal lobe.
C. Temporal lobe.
D. Brain stem.
E. Occipital lobe.
Damage to the cerebellum can lead to:
1) loss of coordination of motor movement (asynergia),
2) the inability to judge distance and when to stop (dysmetria),
3) the inability to perform rapid alternating movements (adiadochokinesia),
4) movement tremors (intention tremor),
5) staggering, wide based walking (ataxic gait),
6) tendency toward falling,
7) weak muscles (hypotonia),
8) slurred speech (ataxic dysarthria), and
9) abnormal eye movements (nystagmus).

Answer is : A

37) 12 years old boy presented with bilateral swelling in the groin after jumping from a high
surface, the left side swelling is painful, physical examination shows increase in size of the
swelling when inhaling, ultrasound was done and confirmed the presence of bilateral inguinal
hernia, what is the best management ?

A. NSAIDs.
B. Laparoscopic hernia repair.
C. Physiotherapy.
D. Open hernia repair
E. Reassurance and send home.

Indications for surgery in inguinal hernia:


● Complicated hernia
● Uncomplicated hernia + moderate symptoms:
○ Inguinal pain associated with exertion
○ Daily activities are limited due to pain
○ Manual reduction is not possible
● Uncomplicated hernia + mild symptoms: elective hernia repair

Open vs. laparoscopic hernia repair


● Indications for open hernia repair
○ Complicated hernias
○ Previous preperitoneal surgeries (e.g., hysterectomy, cesarean section)
○ Presence of ascites
○ Inability to undergo surgery under general anesthesia
○ Recurrent hernia (if the patient initially had a laparoscopic hernia repair)
● Indications for laparoscopic hernia repair
○ Bilateral hernia
○ Recurrent hernia (if the patient initially had an open hernia repair)
Answer is : B

38) 47 years old woman presented to the emergency department complaining of vomiting for
the last 2 days, in the recent hours the vomit became yellow colored, On physical
examination, the Blood pressure is 95/68 respiratory rate is 22 breaths/minute; and
temperature is 37.1˚C, bowel sounds are high pitched on auscultation, Oxygen saturation
was 97% on room air, abdominal X-ray shows air-fluid levels and central loops dilatation,
medical history is remarkable for appendicitis 3 years ago, what confirms the etiology of this
condition ?

A. Barium series
B. MRI.
C. CT scan.
D. Abdominal surgery.
E. Complete Blood Count.

This patient is suspected to have small bowel obstruction secondary to adhesions caused by
previous abdominal surgery.

Definitive confirmation of the adhesive etiology of bowel obstruction is made during operative
treatment

Answer is : D

39) 23 years old female presented with numbness in the middle finger of the left hand
associated with paraesthesia, morning stiffness, loss of delicate movements of middle finger
and index, and dropping of small objects since she had colles fracture 6 weeks ago, what is
the nerve injured ?

A. Ulnar nerve.
B. Median nerve.
C. Radial nerve.
D. Brachial plexus.
E. Musculocutaneous nerve.

Carpal tunnel syndrome (CTS) is a peripheral neuropathy caused by chronic or acute


compression of the median nerve by the transverse carpal ligament.

Clinical features of carpal tunnel syndrome:


● Sensory symptoms on the palmar surface of the thumb, index, and middle finger; and
radial half of the ring finger
○ Burning sensation/paresthesia
○ Loss of sensation/numbness
● These symptoms may be worse at night or after repetitive movements of the hand
and wrist. They may be relieved by shaking the hands, changing position, or
massaging the hands.

The palmar surface of the thenar eminence is spared! The sensory innervation of this area is
supplied by the superficial branch of the median nerve, which arises 5–7 cm proximal to the
carpal tunnel and is therefore not compressed.

Answer is : B

40) 38 years old male asking you for a screening test for colon cancer, because his father
died at 50 years of age of colorectal cancer, what is the prefered age for this patient to start
for colorectal screening?

A. 40.
B. 50.
C. 55.
D. 60.
E. 65.

Screening for colorectal cancer


Screening for colorectal cancer and adenomatous polyps is performed in asymptomatic men and
women ≥ 50 years of age.
● Low-risk individuals: several options
○ Complete colonoscopy (gold standard): Repeat every 10 years if no polyps or
carcinomas are detected.
○ Annual fecal occult blood test (FOBT): screening for occult bleeding, which may
indicate colorectal cancer
○ Sigmoidoscopy every 5 years and FOBT every 3 years
○ Annual fecal immunochemical testing (FIT)
○ CT colonography every 5 years
● High-risk individuals
○ Complete colonoscopy 10 years earlier than the index patient's age at diagnosis or no
later than 40 years of age
○ In case of genetic predisposition: individual screening

Answer is : A

41) A 42 years old female presented to the ER complaining of severe sudden stabbing pain
in the epigastric region then progressed to the entire abdomen, the pain is radiated to the
shoulder, accompanied with vomiting, at presentation the patient blood pressure was
105/72, heart rate: 90bpm, respiratory rate: 30bpm, the patient have a history of h.pylori
infection the recurred after proper treatments, CT scan reveals Pneumoperitoneum, what is
the most appropriate treatment for this patient ?

A. Cholecystectomy.
B. Exploratory laparotomy.
C. IV antibiotics
D. ERCP
E. Ondansetron

Treatment of perforated bowel

General principles
● Bowel rest (NPO)
● IV access with two large-bore peripheral IVs
● Start broad-spectrum IV antibiotics therapy for intra-abdominal infection.
● Aggressive IV fluid resuscitation
● Determine whether indications for surgery are present (i.e., generalized peritonitis,
sepsis) or whether the patient can be managed conservatively with IV antibiotics.
● Provide supportive care (e.g., analgesics, antiemetics).

Supportive care
● NG tube with continuous or intermittent suction
● Consider IV PPI.
● Parenteral analgesics
● Parenteral antiemetics (see antiemetics)
○ Ondansetron
○ Promethazine
Ketorolac is contraindicated in patients with suspected bowel perforation.
Opioids are contraindicated in patients with suspected bowel obstruction.

Surgical management
Most patients with GI tract perforation should be managed with urgent explorative
laparotomy.
● Indications:
○ Signs of generalized peritonitis
○ Signs of sepsis
● Procedure: Exploratory laparotomy with midline incision is usually preferred

Answer is : B
42) A 13 years old boy presented after falling on an outstretched right hand while playing in
the park, physical examination shows a posterior displacement of the distal radial fragment,
X-ray was obtained and shown below, what is the appropriate management ?

A. Open reduction.
B. Closed reduction.
C. Analgesics.
D. Only splinting.
E. Antibiotics.

The image is showing colles fracture.

Management of colles fracture:


● The fracture needs to be reduced (Closed reduction) under whatever form of
anaesthesia is appropriate. The manoeuvre involves disimpaction of the fracture and
a movement forwards and medially (the opposite of the deformity).
● A back slab is applied and a repeat X-ray taken to assess the adequacy of reduction.
If the position is unsatisfactory the procedure needs to be repeated. If the fracture
appears unstable then orthopaedic help is required. Percutaneous pinning is
sometimes necessary.
● The optimum position for immobilising the wrist, whether the cast should be extended
proximal to the elbow and how long the wrist should be immobilised are all still
debated.
● Surgical reduction is recommended for intra-articular fractures if there is any articular
incongruity.
● Healing usually takes about six weeks.

Answer is : B
43) A 43 years old woman Brought to the ED after a road traffic accident, her consciousness
in reduced, she opened her eyes only when cannula was inserted, she doesn't obey
commands, moan and extend her arms and legs, WHat is her Glasgow coma scale ?

A. 4/15
B. 6/15
C. 7/15
D. 8/15
E. 9/15

Open eye to pain = 2 points

Decerebrate posture (extended arms and legs) = 2 points

Moans (Incomprehensible sounds) = 2 points

Glasgow Coma Scale (GCS) is a tool used for evaluation of consciousness

● Assessment of neurological status and trauma severity in patients with traumatic


brain injury (mostly used in acute cases)
● Maximum score 15 points (full consciousness); minimum score 3 points (coma or
death)

Points Eye opening Verbal response Motor response

6 Obeys commands

5 Appropriate words and oriented Localizes pain stimulus

4 Spontaneously Appropriate words but confused Withdraws from pain

3 To verbal command Inappropriate words Decorticate posture


2 To pain Incomprehensible sounds Decerebrate posture

1 No response No response No response

Mild head injury: GCS score 13–15

Moderate head injury: GCS score 9–12

Severe head injury: GCS score ≤ 8 (Indication for endotracheal intubation)

Decorticate posture = Rigid posture with flexed arms and straight legs

Decerebrate posture = Rigid posture with extended (and often rotated) arms and legs

Answer is : B

44) A 31 years old male who underwent appendectomy 2 days ago, presented with vomiting,
diffuse abdominal pain, fever and chills, physical examination shows; BP: 107/70; RR:
24bpm; pulse: 85bpm, absent bowel sounds on auscultation, full blood count shouse
leukocytosis, what is the first step in management ?

A. Observation.
B. Reoperation.
C. IV antibiotics.
D. Diagnostic paracentesis.
E. Heparin.

The diagnosis should be established first, as suspected secondary peritonitis, a peritoneal


fluid analysis should be performed before administering antibiotics.

Answer is : D

45) A 77 years old man presented with dyspnea and pleuritic chest pain, he was diagnosed
with lung cancer 2 months ago,physical examination shows faint breath sounds over the
lower right lobe on auscultation, chest X-ray suggest presence of pleural effusion, what is
the first step in management ?
A. Observation.
B. IV fluids.
C. Chemical pleurodesis.
D. Pleural catheter
E. Thoracentesis.

Answer is : E

46) A 47 years old female after she was diagnosed with hepatocellular carcinoma, asking
about the possible cause of her condition, what will you tell her the major cause of HCC ?

A. Hepatitis B.
B. Hepatitis C.
C. Aflatoxin.
D. Type 2 diabetes.
E. Hemochromatosis.
HCC mostly occurs in people with cirrhosis of the liver, and so risk factors generally include
factors which cause chronic liver disease that may lead to cirrhosis. Still, certain risk factors
are much more highly associated with HCC than others. For example, while heavy alcohol
consumption is estimated to cause 60–70% of cirrhosis, the vast majority of HCC occurs in
cirrhosis attributed to viral hepatitis (although there may be overlap).

Causes of HCC
● Chronic viral hepatitis (estimated cause of 80% cases globally)
○ Chronic hepatitis B (about 50% cases)
○ Chronic hepatitis C (about 25% cases)
● Toxins:
○ Alcohol abuse: the most common cause of cirrhosis
○ Aflatoxin
○ Iron overload state (hemochromatosis)
● Metabolic:
○ Nonalcoholic steatohepatitis: up to 20% progress to cirrhosis
○ Type 2 diabetes (probably aided by obesity)
● Congenital disorders:
○ Alpha 1-antitrypsin deficiency
○ Wilson's disease (controversial; while some theorise the risk increases, case
studies are rare and suggest the opposite where Wilson's disease actually
may confer protection)
○ Hemophilia, although statistically associated with higher risk of HCC, this is
due to coincident chronic viral hepatitis infection related to repeated blood
transfusions over lifetime.

Answer is : A

47) A 41 years old female, 5 days post cholecystectomy developed fever, pain in the
shoulder, physical examination reveals abdominal tenderness, heart rate was 78bpm; blood
pressure: 120/78, CBC reveals leukocytosis, ct scan was done and shows 5cm diameter
abscess, what is the most appropriate treatment?

A. Conservitive .
B. Open drainage.
C. Iv antibiotic.
D. Laparoscopic drainage.
E. Oral antibiotic.

Management of intra-abdominal Abscesses


Drugs
Antibiotics are usually required parenterally. Treatment should be based on the results of
blood or abscess culture material. Both aerobic and anaerobic organisms need to be dealt
with, so a combination of two agents or a broad-spectrum antibiotic is required.

Surgery
Percutaneous drainage​ under CT or ultrasound guidance is the best choice. This is
relatively low-risk and effective in the majority of patients. Failure is usually due to other
complicating factors, such as immune deficiency (the abscess is often tubercular) or
multilocular abscesses.

Answer is : D

48) 32 years old male presented complaining of anal aching pain and bright red bleeding at
the end of defecation for 2 years, physical examination shows manually reducible soft rectal
mass, what is the proper treatment ?

A. Hemorrhoidectomy
B. Sitz bath
C. Fiber rich diet.
D. Lactulose.
E. Rubber band ligation.

Hemorrhoids are a clinical diagnosis


manually reducible = 3rd grade

Hemorrhoids, also called piles, arise from a cushion of dilated arteriovenous blood vessels
and connective tissue in the anal canal that may abnormally enlarge or protrude.
Hemorrhoids are divided into three categories: internal (above the dentate line), external
(below the dentate line), or mixed (above and below the dentate line).
Answer is : E

49) All the following statements are true about Conn's syndrome, except:

A. Conn syndrome is an excess of aldosterone caused by autonomous overproduction.


B. Usually causes hypotension.
C. Polyurea is a recognised symptom.
D. Causes hypokalemia and metabolic alkalosis.
E. typical age of onset is 30-55.

Primary hyperaldosteronism, sometimes referred to as Conn syndrome, is an excess of


aldosterone caused by autonomous overproduction, usually at the adrenal cortex. It is
typically due to adrenal hyperplasia or adrenal adenoma. Primary hyperaldosteronism is one
of the common causes of ​secondary hypertension​. High systemic aldosterone levels result
in increased sodium reabsorption and potassium secretion in the collecting ducts of the
kidney, which leads to the retention of water along with sodium, as well as hypokalemia.
Patients are often asymptomatic and found to have hypertension at routine health checks. It
will often emerge that the patient's hypertension is resistant to pharmaceutical therapy, and
they may have other signs suggestive of secondary hypertension, such as an age of onset
below 30 years or above 55 years. If symptoms are present, they typically include headache,
muscle weakness, and ​polyuria​. Initial labs in primary hyperaldosteronism classically show
a hypertensive patient with ​hypokalemia and metabolic alkalosis​, and high plasma
aldosterone concentration (PAC) and low plasma renin activity (PRA) (PAC/PRA ratio
increased). Following biochemical confirmation of primary hyperaldosteronism with oral or
intravenous sodium loading tests, imaging modalities such as CT and adrenal venous
sampling are used to locate the source of autonomous aldosterone secretion. Treatment of
primary hyperaldosteronism consists of surgical resection of adrenal adenoma or
pharmaceutical therapy with aldosterone antagonists (e.g., spironolactone, eplerenone) in
cases of bilateral adrenal hyperplasia.

Answer is : B

50) A 65 years old male complaining of right upper quadrant pain, on physical examination
pulse: 74bpm; respiratory rate: 30bpm; BP: 110/75, abdominal ultrasound was done and
revealed pericholecystic fluid, what is the most likely diagnosis ?

A. Cholangitis.
B. Pseudomembranous colitis.
C. Cholecystitis.
D. Acute pancreatitis
E. Nephrolithiasis

Diagnostics to reveal cholecystitis:


● Approach: laboratory tests and RUQ ultrasound to identify gallbladder wall inflammation →
perform HIDA scan if US not diagnostic but clinical suspicion remains high
● Laboratory findings
○ Elevated inflammatory markers (especially leukocytes and CRP)
○ Mild elevation in AST, ALT, amylase levels are possible.
○ Elevation in ALP, GGT, bilirubin levels are not common in uncomplicated cholecystitis
and suggest biliary obstruction.
● Imaging
○ Transabdominal ultrasound
■ Gallbladder wall thickening > 4 mm (postprandial > 5 mm)
■ Gallbladder wall edema (double wall sign)
■ Possible free ​fluid surrounding the gallbladder
■ Sonographic Murphy sign
■ Presence of concrement or gallstones
○ HIDA scan (cholescintigraphy) if US is not diagnostic
■ Radioactive tracer 99mTc-hepatic iminodiacetic acid is injected intravenously
→ selective uptake by hepatocytes → subsequent excretion into bile → can
be visualized via a gamma camera
■ Abnormal if gallbladder not visualized within 30–60 minutes: suggests cystic
duct obstruction due to edema or obstructing stone
○ MRCP and abdominal CT are not routinely recommended but may be used to
evaluate for complications (e.g., choledocholithiasis, perforation).

Answer is : C

51) 9y old boy is brought to the emergency because of involuntarily uncoordinated


movements of his extremities and head, his mother said he had a throat infection 2 weeks
ago, on physical examination: his temperature:38,4c / RR: 23bpm / HR: 94bpm, lab test
shows raised ESR and CRP, Elevated Antistreptolysin O titer. What is the most likely
diagnosis ?

A. Meningitis.
B. Huntington’s disease
C. Rheumatic fever.
D. Encephalitis.
E. Wilson disease.

Diagnosis of acute rheumatic fever is based on the Jones criteria, which primarily describe
the clinical findings of the condition. Evidence of a preceding GAS infection is also preferred
(unless carditis or chorea are present). Laboratory tests and imaging may be necessary to
assess any outstanding Jones criteria.

Jones criteria
● Interpretation: two major OR one major plus two minor criteria are required for
diagnosis.
● Major criteria
○ Arthritis (migratory polyarthritis involving primarily the large joints)
○ Carditis (pancarditis, including valvulitis)
○ Sydenham chorea (CNS involvement)
○ Subcutaneous nodules
○ Erythema marginatum
● Minor criteria
○ Arthralgia
○ Fever
○ ↑ Acute phase reactants (ESR, CRP)
○ Prolonged PR interval on electrocardiogram

Additional findings
● Normochromic, normocytic anemia of chronic inflammation
● Leukocytosis
● Confirmation of GAS infection
○ ↑ Antistreptolysin O titer (ASO): antibodies against metabolites of GAS
○ Positive rapid streptococcal antigen test or throat culture for GAS
● Echocardiogram (may show mitral or aortic regurgitation)

Answer is : C

52) A 2 months old baby presented with a cough when feeding for 3 weeks, he had
pneumonia directly after when he was born, his temperature is 37.5, RR: 110Bpm, diagnosis
was made of esophageal atresia what type is most likely to present in his patient ?

A. Gross Type E.
B. Gross Type C
C. Gross Type A
D. Gross Type D
E. Gross Type B

Newborns usually present with symptoms directly after birth! The exception is the Gross type
E fistula: The diagnosis of a small H‑type tracheoesophageal fistula may occur as late as
adulthood.

H-type fistulas are usually present later in infancy as there is no 'blind end' to the
oesophagus and the child is able to feed. Children usually present with recurrent coughs on
feeding or recurrent chest infections.
Answer is : A

53) A neonate was born at 40 weeks gestation by cesarean section, a green amniotic fluid
was noticed, apgar score was 4/10, ABG reveals hypoxia with respiratory acidosis, X-ray
was done and shows increased lung volume and asymmetric, patchy opacities suggestive of
meconium aspiration, what is the treatment?

A. Intubation and endotracheal drainage.


B. Extracorporeal membrane oxygenation
C. Nitric oxide.
D. Artificial surfactant
E. Dexamethasone.

The neonate is in respiratory distress an emergency Intubation and endotracheal drainage


should be performed.

Answer is : A

54) A 7 years old girl brought by her mom complaining of earlier than normal appearance of
breasts and pubic hair, blood tests show High LH levels, what is the diagnosis ?
A. Benign premature thelarche.
B. Peripheral puberty .
C. Central Precocious puberty.
D. Premature adrenarche.
E. Delayed onset of puberty.

Answer is : C

55) A 9 years old male brought by his mom complaining about her child not able to move his
both legs, physical examination shows: no fever, normal heart rate and respiratory rate, his
mom also mentioned that her boy had a sore throat, fever and swollen neck 2 weeks ago,
what's the most likely diagnosis?

A. Meningitis.
B. Guillain-barre syndrome.
C. Poliomyelitis.
D. Encephalitis.
E. Child abuse.

Guillain-Barré syndrome (GBS) is an acute postinfectious polyneuropathy characterized by


symmetric and ascending flaccid paralysis.

● About ⅔ of GBS patients experience symptoms of an upper respiratory or


gastrointestinal tract infection 1–4 weeks prior to onset of GBS.
● Pathogens associated with GBS: Campylobacter jejuni, Cytomegalovirus,
Epstein-Barr virus, HIV, influenza, and Mycoplasma pneumoniae
○ Campylobacter enteritis is the most common disease associated with GBS.
○ Cytomegalovirus is the most common virus associated with GBS

Answer is : B

56) A 7 years old girl brought by her father, presented with high grade fever. Physical
examination shows nick stiffness, positive brudzinski sign and photophobia, lumbar puncture
was performed and the CSF analysis show: high protein levels and low glucose levels and
presence of diplococci, treatment was initiated for her, now the father is considered about
her younger brother, how to protect the younger brother ?

A. Hospital admission and isolation.


B. Meningococcal vaccination
C. Ciprofloxacin
D. Oral rifampin.
E. No need for prophylaxis

Prevention

● Post Exposure chemoprophylaxis:


○ Drugs of choice: rifampin, ciprofloxacin, or ceftriaxone
○ Indicated for all individuals exposed to an index case (close contacts) if
caused by:
■ N. meningitidis (e.g., rifampin, ciprofloxacin, or ceftriaxone)
■ H. influenzae (e.g., rifampin)
Post Exposure prophylaxis Drugs of choice (monotherapy)

Adults ● Rifampin
● Ciprofloxacin
● Ceftriaxone

Infants, children, and adolescents ● Rifampin


● Ceftriaxone

Pregnancy ● Ceftriaxone

Answer is : D

57) A 12 years old boy presented to the ED with his mother complaining of swelling around
his eyes, his mom claims he gained some weight, medical history is unremarkable and do
not take any medication, physical examination shows periorbital edema, dipstick urinalysis
shows protein +3, other urine parameters are normal, what is the proper treatment for him ?

A. SMX/TMP
B. Antihistamine H1.
C. Colchicine.
D. Diuresis.
E. Prednisolone.

Symptoms exhibit nephrotic syndrome, minimal change disease is the most common among
children, patients usually respond well to at least 12 weeks of glucocorticoids (prednisone)

Answer is : E

58) A 10 years old female was rushed to the emergency department with a seizure for the
last few minutes, she has a history of epilepsy, evaluation shows Clonic, involuntary,
repetitive movements of the limbs, the patient is unresponsive, what is the initial
pharmacological agent to administer ?

A. Ketamine.
B. Phenytoin.
C. Valproic acid.
D. Thiopental.
E. Lorazepam.
Answer is : E

59) A 6 years old boy brought by his dad complaining of bulged mass in his left flank,
physical examination shows non tender unilateral left abdominal mass, temperature: 37C;
pulse: 94bpm, Lab test shows hematuria and low blood hemoglobin, what is the most likely
diagnosis?
A. Poststreptococcal glomerulonephritis.
B. Minimal change disease.
C. Intussusception.
D. Wilms tumor.
E. Mesoblastic nephroma.

Wilms tumor (nephroblastoma) is the ​most common​ renal malignancy in children, typically
affecting children 2–5 years of age. A minority of cases are associated with specific
syndromes (e.g., WAGR, Beckwith-Wiedemann) and gene mutations (e.g., WT1). Wilms
tumor is typically an incidental finding that manifests as a large abdominal mass. Other signs
and symptoms may occur, such as hematuria and abdominal pain, especially in tumors that
are large, ruptured, or metastasized. Treatment consists of tumor resection and
chemotherapy for all stages (except for very low-risk tumors), while radiation is also used in
advanced disease.

Answer is : D

60) An 8 years old girl complains of dark urine and headache, physical examination shows ill
looking patient, temperature: 38C; pulse 97bpm; spo2: 98%, urinalysis shows hematuria and
RBCs casts, her mom minions that she had a high fever and sore throat 3 weeks ago, what
is the most appropriate treatment?

A. Penicillin.
B. Glucocorticoids.
C. Supportive.
D. SMX/TMP
E. Methotrexate.

Poststreptococcal (or postinfectious) glomerulonephritis (PSGN) refers to acute glomerular


inflammation that results from a preceding infection with nephritogenic strains of
streptococci. Although most commonly seen in children following group A streptococcal
tonsillopharyngitis, skin infections such as impetigo may trigger PSGN as well. Deposition of
immune complexes containing the streptococcal antigen within the glomerular basement
membrane results in complement activation and subsequent damage to the glomeruli.
PSGN typically presents as a nephritic syndrome with hematuria, mild proteinuria, edema,
and hypertension. Elevated antistreptolysin O titers (ASO), low complement levels, and
elevated creatinine support the diagnosis. In children, close monitoring and supportive
therapy facilitate the recovery process. While most children recover fully, the prognosis in
adults is typically less favorable.

Answer is : C

61) A mother came to you for performing a circumcision on her 4 months old boy, on medical
examination you noticed two meatal openings. What will you do ?
A. Perform the circumcision.
B. Don’t do circumcision.
C. Delay circumcision after 1 year of age
D. Delay circumcision after 2 years of age
E. Delay circumcision after puberty.

Boys who are born with hypospadias should not be circumcised at birth. The extra tissue of
the foreskin may be needed to repair the hypospadias during surgery.

Answer is : B

62) Mom of a 5 months old infant asking about normal development milestones, which of the
following motor abilities should the infant have at this age ?

A. Runs.
B. Sit in parachute position.
C. Commando crawls.
D. Crawl.
E. Walks with support.

Children at 5 months of age can only Sit in a parachute position from the previous choices.

Commando crawls: 8 months


Crawl: 9 months
Walks with support: 12 months
Runs: 18 months

Answer is : B

63) A 6 months old infant brought to a pediatrician by his mom, complaining of constant
crying and refusal to feed, physical examination shows irritabile infant; temperature: 38.3C;
the doctor noticed the infant is constantly touching his ear, otoscopy shows this picture
below, what is the most likely diagnosis?
A. Chronic otitis media.
B. Acute otitis media.
C. Acute otitis externa.
D. Labyrinthitis.
E. ​Tonsillitis.

Acute otitis media (AOM) is a viral or bacterial infection of the middle ear that is most
commonly caused by Streptococcus pneumoniae. AOM is a common infection in children
under the age of five years and it usually follows an upper respiratory tract (URT) infection. It
is characterized by an acute onset of symptoms (e.g., otalgia, fever, anorexia) with signs of
middle ear inflammation (e.g., bulging tympanic membrane, effusion). Mild unilateral
infections can be managed without antibiotics, as they are often self-limiting. Bilateral AOM
or severe symptoms are usually treated with oral amoxicillin. Tympanostomy and insertions
of tympanostomy tubes is recommended in children with recurrent AOM. Complications are
rare and occur mostly in immunosuppressed patients or in AOM due to highly virulent,
drug-resistant bacteria. The most common complication is acute mastoiditis, but facial palsy,
labyrinthitis, and in rare cases, even intracranial abscesses may also occur.

Answer is : B

64) A 7 months old infant presented with dry skin on his face, on physical examination the
infant is seen touching his face constantly, and the skin around his eyes have extra fold,
some of the areas with dry skin are scaly, what is the best initial treatment?

A. Skin moisturizers.
B. Chlorpheneramine.
C. Penicillin.
D. Co-amoxiclav.
E. Stop breastfeeding.
Treatment of atopic dermatitis is centered around rehydrating the skin with ​Skin
moisturizers​ like petroleum jelly and the cautious use of topical steroids to reduce
inflammation and itching. Oral antihistamines may be helpful in breaking the "itch-scratch"
cycle.

Answer is : A

65) A mother of a child asking about the measles vaccines, according to the jordanian
vaccination program, at which age this vaccine should be started ?

A. 1 year.
B. 2 years.
C. 3 months.
D. 1st contact.
E. 9 months.

Answer is : E

66) A 10 years old complaining about right knee pain that worsens with activity, he is a
football training academy player, physical examination shows asymmetrical swelling in the
right knee, just under the patella. Lateral X-ray of the knee is shown below, what is the
diagnosis ?

A. Infective arthritis.
B. Patellar tendinitis
C. Patellar tendon rupture
D. Osgood-Schlatter disease.
E. Bursitis.

Osgood-Schlatter disease is an avascular necrosis thought to arise from overuse of the


quadriceps muscle during periods of growth. This causes a traction apophysitis at the tibial
insertion of the quadriceps tendon. The most common symptom is anterior knee pain that
worsens with exercise. A tibial bump may be felt and can often be seen on x-ray. Treatment
is usually conservative. Surgical excision is only necessary in severe and treatment-resistant
cases.

Answer is : D

67) A 7 years old boy known for sickle cell anemia, presented with left upper quadrant pain,
fatigue. On physical examination the patient is hypotensive, left upper quadrant tenderness,
no rebound tenderness, lab tests show reticulocytosis, and low hemoglobin, what is the most
likely diagnosis ?
A. Hepatitis.
B. Splenic sequestration crisis.
C. Aplastic crisis
D. Acute hemolytic crisis
E. Acute chest syndrome

Splenic sequestration crisis (SSC) is a life-threatening illness common in pediatric patients


with homozygous sickle cell disease and beta thalassemia. Up to 30% of these children may
develop SSC with a mortality rate of up to 15%. This crisis occurs when splenic
vaso-occlusion causes a large percentage of total blood volume to become trapped within
the spleen. Clinical signs include severe, rapid drop in hemoglobin leading to hypovolemic
shock and death. Pediatric patients with sickle cell disease and beta thalassemia experience
multiple splenic infarcts, resulting in splenic fibrosis and scarring. Over time, this leads to a
small, auto infarcted spleen typically by the time patients reach adulthood. Splenic
sequestration crisis can only occur in functioning spleens which may be why this crisis is
rarely seen in adults. However, late adolescent or adult patients in this group who maintain
splenic function may develop splenic sequestration crisis.

Answer is : D

68) A 7 months old infant brought by his mother to the emergency department, looking pale
and weak, physical examination reveals jaundice, enlarged abdomen, frontal bossing,
prominent facial bones, blood smear shows teardrop appearance of RBCs, what the
confirmatory test to be conducted for this patient?

A. Ferritin level.
B. Full blood count.
C. Hb-electrophoresis.
D. Abdominal CT scan.
E. Echocardiography.

Confirmatory tests for major thalassemia are:


Hb-electrophoresis or DNA analysis

Features of thalassemia major:


● Severe hemolytic anemia that often requires transfusions → secondary iron overload
due to hemolysis, transfusion, or both → secondary hemochromatosis
● Hepatosplenomegaly
● Growth retardation
● Skeletal deformities (high forehead, prominent zygomatic bones, and maxilla)
● Transient aplastic crisis (secondary to infection with parvovirus B19)

Answer is : C

69) A 6 year old boy came to the emergency department after his mother noticed he is
breathlessness while sitting near her, she mentioned he had previous attacks of asthma, on
physical examination heart rate: 110bpm; respiratory rate: 25bpm; Spo2:93%, wheezes are
heard without stethoscope, spirometry was performed showing PEF: 65%, he is not getting
better using salbutamol inhaler, what is the next step in management ?

A. Oral prednisolone.
B. IV glucocorticoids.
C. IV magnesium sulfate.
D. Add Ipratropium bromide
E. Give salmeterol.

Management of acute asthma:

Assessment of severity
If a patient has signs and symptoms across categories, always treat according to their most
severe features.
● Moderate asthma exacerbation:

○ PEFR 50-75% best or predicted.


○ Oxygen saturations (SpO2) ≥92%.
○ Speech normal.
○ Respiration <25 breaths per minute.
○ Pulse <110 beats per minute.
● Acute severe asthma - any one of:
○ PEFR 33-50% best or predicted.
○ Oxygen saturations (SpO2) ≥92%.
○ Can't complete sentences.
○ Respiratory rate ≥25 breaths per minute.
○ Pulse ≥110 beats per minute.
● Life-threatening asthma - any one of the following in a patient with severe asthma:
○ PEFR <33 best or predicted.
○ Oxygen saturations (SpO2) <92%.
○ Silent chest, cyanosis or poor respiratory effort.
○ Arrhythmia or hypotension.
○ Exhaustion, altered consciousness.
Moderate asthma management:
Treat at home or in the surgery and assess response to treatment.
● If PEFR >50-75% predicted/best:
○ Beta2 bronchodilator. Depending on the equipment available, use either:
■ Beta2 bronchodilator via spacer (give four puffs initially and give a
further two puffs every two minutes according to response up to
maximum of 10 puffs); or
■ Nebuliser (preferably oxygen-driven) with salbutamol 5 mg.
○ Give prednisolone 40-50 mg.
○ Continue or increase usual treatment.

Answer is : A

70) A 4 years old boy presented with swollen left knee and recurrent epistaxis, doppler
ultrasound of the knee reveals hemarthrosis, lab test shows prolonged aPTT, Prothrombin
time is normal, bleeding time is normal, CBC shows: low hematocrit, what is the most likely
diagnosis?

A. Von Willebrand disease.


B. Vitamin K deficiency.
C. Hemophilia B.
D. Hemophilia A.
E. Ehlers danlos syndrome.

Hemophilias are disorders of blood clotting and consequently may lead to serious bleeding.
In the majority of cases, these disorders are hereditary. There are three types of hemophilia,
determined based on which clotting factor is deficient: hemophilia A (factor VIII), hemophilia
B (factor IX), and hemophilia C (factor XI). All types result in impaired secondary hemostasis
(plasmatic coagulation) that manifests as increased activated partial thromboplastin time
(aPTT). Hemophilia presents with hemarthrosis (bleeding into joints) and muscular or soft
tissue hematomas. Depending on the remaining clotting factor activity, bleeding may occur
spontaneously or in response to trauma of varying severity. Repeated hemarthrosis can
eventually lead to joint destruction, a serious long-term complication of hemophilia.
Diagnosis is based on patient history and a mix of semiquantitative and quantitative
measurements of clotting factor activity. Severe hemophilia (enzyme activity < 1%) is treated
with prophylactic substitution of clotting factors, whereas mild hemophilia A, in particular, can
also be treated with desmopressin, a synthetic vasopressin analog.

Answer is : A
71) An 8 years old boy complaining of abdominal pain and arthralgia and dark urine, on
medical examination, respiratory rate: 19bpm; pulse: 90bpm, a red macular rash was
distributed all around the posterior aspect of both thighs, his father mentioned that he had a
bad fever and sore throat 2 weeks ago, urinalysis shows tRBCs casts, what is the
diagnosis?

A. Eczema.
B. Polyarthritis
C. Thrombotic Thrombocytopenic Purpura.
D. Henoch-Schonlein purpura.
E. Idiopathic thrombocytopenic purpura.

Henoch-Schonlein purpura (HSP) is an acute immune complex-mediated small vessel


vasculitis that most commonly occurs in children. It is often preceded by an upper respiratory
tract infection and typically presents with a tetrad of symptoms: palpable purpura,
arthritis/arthralgia, abdominal pain, and renal disease. However, in any individual patient,
only some of the classic tetrad of symptoms may be present. HSP is a clinical diagnosis, but
in particularly unclear or atypical cases a biopsy may be used to confirm the diagnosis.
Because the disease course is usually self-limiting, treatment is generally supportive. Severe
cases may require glucocorticoids, antihypertensive drugs, and possibly dialysis. HSP has
an excellent prognosis, usually resolving within one month when not complicated by
significant renal disease.

Answer is : D

72) What is the APGAR score of the following newborn baby?. On assessment, the newborn
have: pink body and hands with cyanotic feet, heart rate 109, grimace to stimulation, flaccid,
and irregular cry.

A. APGAR 4.
B. APGAR 5.
C. APGAR 6.
D. APGAR 7.
E. APGAR 8.

The Apgar score is a method to quickly summarize the health of newborn children against
infant mortality.
The Apgar score is determined by evaluating the newborn baby on five simple criteria on a
scale from zero to two, then summing up the five values thus obtained. The resulting score
ranges from zero to 10. The five criteria are summarized using words chosen to form a
backronym (​A​ppearance, ​Pu​ lse, ​G​rimace, ​A​ctivity, ​R​espiration).

A= 1 = Cyanotic hands
P= 2 = HR >100
G= 1 = Grimince
A= 0 = Flaccid
R= 1 = Irregular cry
Overall = 5 pagar score

Answer is : B

73) Which of the following comes first in order in normal puberty ?

A. Pubarche.
B. Menarche.
C. Gonadarche.
D. Growth spurt.
E. Adrenarche.

Normal order of changes in girls: adrenarche → gonadarche → thelarche (age of onset 8–11
years) → growth spurt (age of onset 11.5–16.5 years) → pubarche (mean age of onset 12
years)→ menarche (age of onset 10–16 years ; mean age: 13 years)

Answer is : E

74) A 5 years old boy refugee presented with diarrhea and jaundice, the mom says he is not
eating as much as before since 4 days, lab tests show mixed hyperbilirubinemia, and high
transaminases, Urinalysis shows higher than normal urobilinogen. What is the most likely
diagnosis?
A. Hepatitis A.
B. Hepatitis B.
C. Hepatitis C.
D. Hepatitis D.
E. Hepatitis E.

Hepatitis A infection is the most common cause of acute hepatitis in children.

Refugee means it is possible the patient is not vaccinated.

Answer is : A

75) A 7 years old child presented to the emergency department with severe diarrhea,
abdominal pain and anorexia since yesterday, physical examination shows fever, what is the
most likely causative organism ?

A. Shigella
B. Rotavirus.
C. Norovirus
D. Escherichia coli.
E. Salmonella

Gastroenteritis is caused by a variety of viral, bacterial and parasitic pathogens. Of the


infectious agents isolated from children with enteric infections, rotavirus was found most
commonly (56%), followed by Campylobacter spp. (28%), Salmonella spp. (11%), norovirus
(3%), Shigella spp. (1%), and Escherichia coli O157 (1%).

Answer is : B

76) A 27 years old woman presented concerning a lumb she felt on her left breast, physical
examination shows soft mobile mass, what is the next step in management ?

A. Reassure.
B. Mammography.
C. Ultrasound.
D. Fine needle aspiration.
E. Reexamine after menstruation.
Answer is : E
77) A 37 years old lady complains of pelvic pain, which becomes more severe during
mensis, the woman is G2P2, history of cesarean section, ultrasound has normal findings,
how to confirm diagnosis of adenomyosis ?

A. Serology.
B. MRI
C. Hysterectomy and biopsy.
D. Colposcopy.
E. Laparoscopy.

The only way to confirm adenomyosis is to examine the uterus after hysterectomy. However,
pelvic imaging such as ultrasound and MRI can detect signs of it. Other uterine diseases can
cause signs and symptoms similar to adenomyosis, making adenomyosis difficult to
diagnose.

Answer is : C

78) A 25y woman complains of irregular heavy periods, the physician noticed a more notable
facial hair, and darkness around her neck, she is obese, Which of the following is most likley
lab finding in this patient ?

A. High T4
B. Low estrogen.
C. Low glucose
D. High LH.
E. Low testosterone.

The patient’s symptoms suggest Polycystic ovarian syndrome. A High LH is seen in PCOS.

Clinical features of PCOS:


● Onset typically during adolescence
● Menstrual irregularities (primary or secondary amenorrhea, oligomenorrhea)
● Difficulties conceiving or infertility
● Obesity and possibly other signs of metabolic syndrome
● Hirsutism
● Androgenic alopecia
● Acne vulgaris and oily skin
● Acanthosis nigricans: hyperpigmented, velvety plaques (axilla, neck)
● Premature adrenarche
Diagnostics:

Diagnostic criteria
According to the American Association of Clinical Endocrinologists, at least two of three of
the criteria below are required for diagnosis of PCOS after excluding other causes of
irregular bleeding and elevated androgen levels.
● Hyperandrogenism (clinical or laboratory)
● Oligo- and/or anovulation
● Polycystic ovaries on ultrasound

Diagnosis of PCOS is possible without the presence of ovarian


cysts!

Blood hormone levels


● ↑ Testosterone (both total and free) or free androgen index
● ↑ LH (LH:FSH ratio > 2:1)
● Estrogen is normal or slightly elevated

A clinical picture of hyperandrogenism overrules any normal


hormone levels and can fulfill a diagnostic criterion of PCOS!

Transvaginal ultrasound
● Enlarged ovaries with numerous anechoic cysts (polycystic ovaries)
● "String of pearls” appearance

Answer is : D

79) A 39y african woman complains of pain during intercourse and heavy bleeding, bimanual
pelvic examination reveals a tender asymmetrical large uterus, what is the next step ?

A. Colposcopy.
B. CT.
C. MRI.
D. Ultrasound.
E. PAP Smear.
African population is a risk factor for leiomyoma.

Diagnostics for uterine leiomyoma:


● Ultrasound (best initial test)
○ Concentric, hypoechoic, heterogeneous tumors
○ Calcifications or cystic areas suggest necrosis.
● MRI: to evaluate the uterus and ovaries for potentially complicated surgical cases
and visually differentiate between leiomyomas, adenomyosis, and adenomyosis

Answer is : D

80) 60y old lady on hormonal replacement therapy for 4 years, she noticed dark blood stains
on her pants, she had a PAP smear 2 years ago and was normal, what is the next step ?

A. Transvaginal ultrasound.
B. Repeat PAP smear
C. CT scan.
D. MRI
E. High vaginal swab

If suspected endometrial cancer, a transvaginal US should be performed to look for any


thickening of the endometrium.

Pap smear is not necessary to repeat as it performed every 3-5 years

Answer is : A

81) 35y old lady came to the emergency department for lower abdominal severe pain, her
abdomen is rigid,temperature:38,9c, BP:120/80, RR:25, HR:90BPM, bimanual vaginal exam
reveals cervical motion tenderness, a history of PID 5 months ago which was treated, what
is the next step ?

A. Ultrasound.
B. Abdominal X Ray
C. Endocervical swab.
D. Colposcopy
E. Laparotomy.

An ultrasound is the first test to be done if pelvic abscess is suspected (the history of PID
raises the suspicion of pelvic abscess) The swap can take a long time for the results to come
back, this is not helpful in an emergency situation.

Answer is : A
82) 35y old female with a history of menorrhagia asking you about the best contraceptive
method, what would you recommend ?

A. Levonorgestrel IUD.
B. Copper IUD
C. Combined OCP
D. Progesterone only OCP
E. Vaginal ring

The best and most effective contraceptive method is IUD(less than 1% failure rate), in this
lady’s case copper IUD would worsen the menorrhagia.

Answer is : A

83) 24y old pregnant lady in the first trimester, is experiencing severe vomiting continuously
for several days, lab tests show ketonuria, what is the most appropriate management ?

A. Iv fluids.
B. Vitamine B6
C. Metoclopramide.
D. Vitamin B12
E. Vitamin B1

Hyperemesis gravidarum refers to persistent and severe vomiting leading to fluid and
electrolyte disturbance, marked ketonuria, nutritional deficiency and weight loss.

Management of hyperemesis gravidarum


● Advice, including dietary advice, and support.
● Fluid and electrolyte replacement:
○ Women who are severely dehydrated and ketotic need to be assessed in
secondary care, with intravenous fluid and electrolyte replacement (with
normal saline or Hartmann's solution).
○ Fluid and electrolyte balance must be reassessed frequently.
○ Potassium must be replaced appropriately.
● Nutritional support (enteral or parenteral) may be required.
● Vitamin supplements:
○ Thiamine supplements should be given routinely - orally if tolerated, or
intravenously - to all pregnant women admitted to hospital as a result of
prolonged vomiting.

Answer is : A
84) A 43y old lady presented after feeling a smooth large breast mass, a mammogram was
performed and a “Phyllodes tumor” was suspected, after which a core biopsy was performed
and confirmed the diagnosis, what is the most appropriate treatment ?

A. Complete excision.
B. Total mastectomy.
C. Ressurance.
D. Incision and drainage.
E. Tamoxifen.

Complete excision with wide margins is required for all phyllodes tumors (due to their high
recurrence rate and malignant potential).

Phyllodes tumor
● Definition: rare fibroepithelial tumor with histology similar to that of fibroadenoma
● Etiology: unknown
● Epidemiology
○ 1% of all breast tumors
○ Most commonly benign
○ Peak incidence: 40–50 years
● Clinical features
○ Painless, smooth, multinodular lump in the breast
○ Variable growth rate: may grow slowly over many years, rapidly, or have a biphasic
growth pattern
○ Average size 4–7 cm
● Diagnosis
○ Ultrasound and mammogram findings are similar to fibroadenoma, but phyllodes
tumors tend to be larger and grow faster than fibroadenomas.
○ Despite the fact that the lesion is typically benign, a suspected phyllodes tumor
should be considered a suspicious mass until proven otherwise.
○ If a phyllodes tumor is suspected (based on clinical or imaging findings) → core
biopsy
■ Leaf-like architecture with papillary projection of epithelium-lined stroma and
varying degrees of atypia and hyperplasia
● Treatment
○ Surgical excision
○ In case of recurrence: total mastectomy
● Prognosis
○ After excision of benign tumors: excellent prognosis
○ Lesions that show signs of malignancy on histology may recur and metastasize.

Core biopsy is the first-line test for biopsy, but still has a ∼ 25% false negative rate. If a core
biopsy is negative, but the mass continues to grow rapidly, an excisional biopsy should be
performed

Answer is : A
85) All the following sentences are true about IUD use, except:

A. Can only be used on women after 17y old.


B. Copper IUD is used as an emergency contraceptive.
C. Levonorgestrel IUD is used for menorrhagia.
D. Levonorgestrel IUD duration of use is 5 years.
E. It is a reversible method of contraception.

The intrauterine device is an appropriate choice of birth control for this patient from 15 year
old (with no contraindications) requiring long-term contraception. Intrauterine devices may be
used in nulliparous women and there is no age requirement for their use.

Answer is : A

86) A 22 years 22 weeks gestation presented with an asthma attack, PEF was performed
and shows: 68%, she uses inhaled short acting beta agonist (SABA), she is on 90% oxygen,
what is the next step in management ?

A. IV glucocorticoids.
B. Inhaled glucocorticoids.
C. LABA.
D. Antihistamine
E. Ipratropium inhalation.

● Asthma symptoms can be worse, better, or unchanged during pregnancy.


● Same stepwise management as with other patients
● Inhalation treatments preferred
● Poorly managed asthma can increase the risk of pregnancy complications (e.g.,
preeclampsia, premature birth, congenital abnormalities).
● Monthly monitoring of asthma is recommended.

Answer is : B

87) A 24 years old lady G2P1, gestation age: 20 came for a routine prenatal visit, all exams
are normal except for a urine dipstick which shows the presence of leukocyte esterase and
nitrites, the woman don’t complain of any pelvic pain not during urination, physical
examination shows no fever, what is the best initial treatment for this condition after
confirmation ?

A. Paracetamol.
B. NSAIDs.
C. Rifampicin.
D. Amoxicillin-clavulanate.
E. Levofloxacin.
This woman has an asymptomatic UTI.

Treatment of UTIs in pregnant women


● Uncomplicated UTI
○ Obtain urinalysis and urine culture
○ Empiric treatment options
■ Aminopenicillins such as amoxicillin-clavulanate
■ Oral cephalosporins (e.g., cefpodoxime)
■ Fosfomycin
■ Alternatives (especially during the 2nd and 3rd trimesters): TMP-SMX,
nitrofurantoin
○ Antibiotic therapy is tailored when urine culture and susceptibility results become
available.
■ Many antibiotics that are acceptable therapy for UTIs in nonpregnant patients
are contraindicated during pregnancy.
● Complicated UTI: hospitalization and administration of intravenous antibiotic
● Follow-up culture is required.

Ampicillin should no longer be used in the treatment of asymptomatic bacteriuria because of


high rates of resistance.

Answer is : D

88) Which of the following vaccines are not recommended to vaccinate a pregnant woman?

A. Influenza vaccine
B. BCG.
C. Rabies
D. Meningococcal
E. diphtheria toxoids

Immunizations During Pregnancy


Considered safe if otherwise indicated
● Tetanus and diphtheria toxoids (Td)
● Hepatitis B
● Influenza (inactive form)
● Meningococcal
● Rabies

Contraindicated during pregnancy or safety not established


● BCG
● Measles
● Mumps
● Rubella
● Varicella

Special recommendations pertain


● Anthrax
● Hepatitis A
● Japanese encephalitis
● Pneumococcal
● Polio (IPV)
● Typhoid (parenteral and Ty21a*)
● Vaccinia*
● Yellow fever*

Answer is : B

89) A 23 married woman came for routine gynecological checkup, she stated that she has
never done any cervical cancer screening tests, what should be done ?

A. Schedule pap smear after 3 years.


B. Schedule pap smear after 30 years of age.
C. Schedule pap smear with HPV test after 3 years
D. No need for screening.
E. Do pap smear only now.

Answer is : E
90) A 52 years old lady complaining of vaginal bleeding for 3 days, she stated that she is in
menopause for 3 years, no history of hormone replacement therapy, what is the next step in
investigation ?

A. Transvaginal ultrasound.
B. Speculum examination.
C. Endometrial biopsy.
D. Hysteroscopy.
E. PAP smear.

Transvaginal ultrasound ​is an appropriate first-line procedure to identify which women with
PMB are at higher risk of endometrial cancer.
The mean endometrial thickness in postmenopausal women is much thinner than in
pre-menopausal women. Thickening of the endometrium may indicate the presence of
pathology. In general, the thicker the endometrium, the higher the likelihood of important
pathology, i.e. endometrial cancer being present.

A definitive diagnosis in PMB is made by histology.

Answer is : A

91) A 23 years old G2P1 on 18 weeks gestation, presented to you and complained about
vaginal bleeding and big chunks of blood, vaginal examination reveals conceptional products
in the vaginal canal, doppler ultrasound of the fetus didn’t show fetal heart activity, what is
the next step in management ?

A. Dilation and Curettage.


B. Misoprostol.
C. Tamoxifen.
D. Tranexamic acid.
E. Expectant management.

Expectant management should be considered for women with incomplete spontaneous


abortions. It has an 82 to 96 percent success rate without the need for surgical or medical
intervention. When misoprostol (Cytotec) is used to treat women with a missed spontaneous
abortion, it should be given vaginally rather than orally.

Answer is : E

92) A 28 years pregnant lady 34 gestation, came to emergency department looking for help
after painful contractions and continuous vaginal bleeding for the last 50 minutes, on
physical examination the pulse is 95bpm; blood pressure: 90/68; temperature: 36.8C,
ultrasound was done and do not show any placental abnormality, doppler ultrasound shows
fetal bradycardia, after stabilizing the mother the fetes still have bradycardia, what is the
most appropriate treatment ?
A. Induction of vaginal delivery
B. Expectant.
C. Hysterectomy.
D. Atropine IV.
E. Cesarean section.

Placental abruption is a clinical diagnosis.

Management of abruptio placentae:

The mother's life should take priority. She should be resuscitated and stabilised before any
decision is made regarding delivery of the baby, regardless of the gestation. Surprisingly, a
Cochrane review has found no trials to inform management.
Guidance from the Royal College of Obstetricians and Gynaecologists for moderate or
severe placental abruption is to follow ABCD of resuscitation:
● Assess Airway and Breathing: high-flow oxygen.
● Evaluate Circulation:
○ Intravenous access, FBC, coagulation screen, U&E, Kleihauer test,
crossmatch four units.
○ Position in the left lateral position tilted and keeping the woman warm.
○ Until blood is available, infuse up to 2 litres of warmed crystalloid Hartmann's
solution and/or 1-2 litres of colloid as rapidly as required.
○ With continuing massive haemorrhage and whilst awaiting coagulation
studies and haematology advice, up to 4 units of fresh frozen plasma (FFP)
and 10 units of cryoprecipitate may be given empirically.
○ Ideally, measure central venous pressure (CVP) and adjust transfusion
accordingly.

After stabilizing the mother, the fetal management as follows:


Answer is : E
93) What is the advantage of performing mediolateral episiotomy over midline episiotomy ?

A. Easier surgical repair.


B. Less blood loss.
C. Less risk of extension.
D. Better tissue healing.
E. Less postoperative pain.

Answer is : C

94) A 49 years old lady presented with green nipple discharge on the right side, on physical
examination nipple was inverted backward and the was tinder, mammogram was done and
showed dilated linear branching densities in subareolar region, What is the most likely
diagnosis ?

A. Sclerosing adenosis.
B. Mastitis
C. Duct ectasia.
D. Fibroadenoma
E. Fat necrosis of the breast
Mammary duct ectasia is the most common cause of greenish discharge.

Mammary duct ectasia


● Definition​: subareolar periductal chronic inflammatory condition defined by dilated mammary
ducts which are eventually clogged
● Etiology​: inspissated luminal secretion stasis leading to periductal inflammation and fibrous
obliteration
● Epidemiology
○ Most common in perimenopausal women
○ Peak incidence: 40–50 years
● Clinical features
○ Unilateral greenish or bloody discharge
○ Nipple inversion
○ Firm, stable, painful mass under the nipple (may mimic breast cancer)
○ May progress to a breast abscess
● Diagnosis
○ Mammogram and/or ultrasound: noninvasive imaging modalities can determine duct
diameter
○ If any suspicious or inconclusive imaging findings, perform a biopsy: shows a central
cavity filled with neutrophils and secretion surrounded by inflamed and/or fibrotic
breast parenchyma, with obliteration of the ducts
● Treatment
○ Usually not necessary (most cases resolve spontaneously)
○ Antibiotic therapy if infected
○ Surgical excision for persistent lesions

Answer is : C

95) A 21 years old lady came to the ED because of severe pelvic pain during menstruation,
this is the second time this happens but this time is more severe, the doctor started
investigations for organic causes and found nothing, what is the best treatment?

A. Surgical intervention.
B. NSAIDs.
C. Morphine.
D. Paracetamol.
E. Progesterone only pills.

Since there is no organic cause for this dysmenorrhea the diagnosis is primary
dysmenorrhea.

● Treatment
○ Symptomatic treatment: pain relief (e.g., NSAIDs), topical application of heat
○ Hormonal contraceptives (e.g., combined oral contraceptive pill, IUD with
progestogen)
Answer is : B

96) A 25 years old lady complaining of vaginal discharge that is not improving despite taking
co-amoxiclav tablets her neighbor gave her. Physical examination shows a frothy green
discharge, smear test shows flagella in protozoa, what is the best treatment ?

A. Metronidazole.
B. Ceftriaxone.
C. Fluconazole
D. Rifampin.
E. Ciprofloxacin.

Trichomoniasis is most likely.


● Pathogen: Trichomonas vaginalis
○ Anaerobic, motile protozoan with flagella
● Transmission: sexual
● Clinical features
○ Foul-smelling, frothy, yellow-green, purulent discharge
○ Vulvovaginal pruritus, burning sensation, dyspareunia, dysuria, strawberry
cervix
● Diagnosis
○ Saline wet mount of vaginal smear: protozoa with multiple flagella
○ If wet mount is inconclusive → culture
○ pH of vaginal discharge > 4.5
● Treatment: oral metronidazole or tinidazole for patient and sexual partner(s)

Answer is : A

97) A 39 years old lady G2P2 came to the emergency department after right left lower
abdominal pain, on physical examination puls was 88bpm; respiratory rate: 18bpm; blood
pressure: 115/75, lower left quadrant tenderness, her husband mentions that she have
hormonal IUD, cervical examination revealed a closed cervix and presence of blood in
vaginal canal, what is the next step in investigations ?

A. Vaginal pH strep.
B. Ultrasonography.
C. CT abdomen.
D. β-hCG test.
E. Blood culture.

Every woman of reproductive age with abdominal pain should undergo a pregnancy test!

IUD is a risk factor for ectopic pregnancy


General symptoms of ectopic pregnancy
Patients usually present with signs and symptoms 4–6 weeks after their last menstrual
period.
● Lower abdominal pain and guarding
● Vaginal bleeding
● Signs of pregnancy: amenorrhea, nausea, breast tenderness, frequent urination
● Tenderness in the area of the ectopic pregnancy
● Cervical motion tenderness, closed cervix
● Enlarged uterus
● Interstitial pregnancies tend to present late, at 7–12 weeks of gestation, because of
myometrial distensibility.

Right lower quadrant pain may indicate appendicitis! Cervical motion tenderness may be a
sign of PID!

Answer is : D

98) A 42 years old pregnant lady 34 weeks gestation presented with vaginal bleeding, she
has a history of placenta previa, what is the best diagnostic tool for placenta previa ?

A. CT scan.
B. Transvaginal ultrasound.
C. Hysteroscopy.
D. Digital vaginal examinations.
E. Serum B-HCG.

Transvaginal ultrasound is the most sensitive imaging modality for diagnosing placenta
previa.

Risk factors for placenta previa


● Maternal age > 35 years, multiparity, short intervals between pregnancies
● Previous curettage or cesarean section
● Previous placenta previa, previous/recurrent abortion

Answer is : B

99) All the following sentences are true about preeclampsia in pregnancy, except:

A. Proteinuria is the hallmark of preeclampsia.


B. The majority of cases are diagnosed at the second trimester.
C. Liver enzymes can be elevated.
D. Pitting edema is a sign of preeclampsia.
E. Methyldopa is considered a safe treatment.
90% of preeclampsia cases occur after 34 weeks of gestation.

In the absence of proteinuria, the presence of new-onset hypertension (elevated blood


pressure) and the new onset of one or more of the following is suggestive of the diagnosis of
preeclampsia:
● Evidence of kidney dysfunction (oliguria, elevated creatinine levels)
● Impaired liver function (noted by liver function tests)
● Thrombocytopenia (platelet count <100,000/microliter)
● Pulmonary edema
● Ankle edema (pitting type)
● Cerebral or visual disturbances

Answer is : B

100) Which of the following is not a cause of postpartum hemorrhage ?

A. Uterine atony
B. Fetal tissue retention.
C. Factor V Leiden.
D. Von Willebrand disease.
E. Birth Canal injury.

Factor V Leiden causes a hypercoagulable state. Which is protective against PPH.

Causes of postpartum hemorrhage​ are uterine atony, trauma, retained placenta or


placental abnormalities, and coagulopathy, commonly referred to as the "four Ts".
● Tone​: uterine atony is the inability of the uterus to contract and may lead to
continuous bleeding. Retained placental tissue and infection may contribute to
uterine atony. Uterine atony is the most common cause of postpartum hemorrhage
● Trauma​: Injury to the birth canal which includes the uterus, cervix, vagina and the
perineum which can happen even if the delivery is monitored properly. The bleeding
is substantial as all these organs become more vascular during pregnancy.
● Tissue​: retention of tissue from the placenta or fetus as well as placental
abnormalities such as placenta accreta and percreta may lead to bleeding.
● Thrombin​: a bleeding disorder occurs when there is a failure of clotting, such as with
diseases known as coagulopathies.

Other risk factors include obesity, fever during pregnancy, bleeding before delivery, and
heart disease.

Answer is : C
101) 45 years old woman complaining of vomiting, bowel distention, and not able to pass
stool for 4 days, you suspect a bowel obstruction, What is the most common cause of small
bowel obstruction ?

A. Femoral hernia.
B. Colorectal cancer.
C. Adhesions.
D. Inguinal hernia.
E. Superior mesenteric artery syndrome.

Causes of small bowel obstruction


● Adhesions (e.g., prior abdominal surgery, abdominal tuberculosis): most common
cause of SBO
● Incarcerated hernias: second most common cause of SBO
● Meckel diverticulum
● Strictures (e.g., Crohn disease)
● Malignant tumors or metastases
● Gallstone ileus
● Superior mesenteric artery syndrome
● Foreign body impaction
● Tumor
● Internal hernia

Specific to infants and children :


● Congenital intestinal atresia (e.g., duodenal atresia, jejunal atresia)
● Intussusception (e.g., secondary to Meckel diverticulum)
● Congenital strictures and bands (e.g., Ladd bands in intestinal malrotation)

Answer is : C

102) Which of the following statements is true about shock ?

A. Peripheral vasoconstriction is compensation mechanism in all types of shock.


B. Loss of intravascular fluid causes hypovolemic shock.
C. IV fluids are mostly contraindicated in cardiogenic shock.
D. Septic shock is mainly caused by gram positive bacteria.
E. Causes metabolic alkalosis.

● Loss of intravascular fluid causes Hypovolemic shock.


● Peripheral vasoconstriction is not a feature of hypovolemic, neurogenic, and septic
shock.
● Septic shock is mainly caused by gram negative bacteria.
● Hypoperfusion of peripheral tissues → generalized tissue hypoxia → anaerobic
metabolism in the underperfused organs → lactic acidosis
● Unlike other causes of shock the administration of intravenous fluids in most cases of
cardiogenic shock would worsen cardiogenic pulmonary edema!

Answer is : C

103) Which of the following thyroid cancers is involved in the MEN2 ?

A. Hurthle cell cancer.


B. Papillary thyroid carcinoma.
C. Follicular thyroid carcinoma.
D. Anaplastic thyroid carcinoma.
E. Medullary carcinoma.

Hürthle cell adenoma is a rare benign tumor, typically seen in women between the ages of
70 and 80 years old. This adenoma is characterized by a mass of benign Hürthle cells.

Answer is : E

104) All the following are measures done for preventing kidney stones, except:

A. Reduced consumption of table salt


B. More consumption of Vitamin C.
C. Allopurinol.
D. Sufficient fluid intake
E. Reduced consumption of oxalate-rich foods

Vitamin C is metabolized to oxalate, thus, high vitamin C consumption increases the risk.

Prevention measures for preventing kidney stones :


● Sufficient fluid intake (≥ 2.5 L/day)
● For calcium stones:
○ Reduced consumption of salt and animal protein
○ Reduced consumption of oxalate-rich foods and supplemental vitamin C: for
oxalate stones
○ Thiazide diuretics for recurrent calcium-containing stones with idiopathic
hypercalciuria (i.e., no hypercalcemia)
● For uric acid stones or high urinary uric acid levels in those with calcium stones:
allopurinol
● Depending on urinary pH and stone composition:
○ Urine alkalinization for Calcium oxalate stones and Uric acid stones.
○ Urine acidification for Struvite stones and Calcium phosphate stones,

Answer is : B

105) Which of these antibiotics works by disrupting bacterial cell wall synthesis ?

A. Cephalosporins.
B. Tetracycline.
C. Macrolides.
D. Quinolones.
E. Sulfonamides.
Answer is : A

106) Which of the following statements is true about abdominal blunt trauma ?

A. Splenic rupture and liver injury are the most common injuries.
B. Absence of pain rules out severe injury.
C. Guarding or rigidity happens due to free air accumulation.
D. Laparotomy is indicated only for patients with Hemodynamic instability.
E. hemodynamically unstable patients should perform a CT scan if FAST not available..

The absence of pain does not rule out significant intra-abdominal injury! Imaging must be
performed!

Guarding or rigidity happens due to peritoneal irritation

Laparotomy is indicated for patients with


● Hemodynamic instability
● Signs of peritonitis
● Intra-abdominal bleeding detected on imaging

If FAST exam is not available, a hemodynamically unstable patient should be taken to the
operating room immediately!

Answer is : A

107) 17 years old female presented with sharp lower left quadrant pain, fever, nausea and
vomiting, the doctor is suspecting appendicitis, All the following tests if positive are signs
appendicitis, except:

A. Rovsing's sign
B. Blumberg's sign
C. Psoas sign
D. Chvostek sign.
E. Obturator sign

The Chvostek sign is a clinical sign of existing nerve hyperexcitability (tetany) seen in
hypocalcemia. It refers to an abnormal reaction to the stimulation of the facial nerve.
Clinical features of appendicitis
● Nonspecific symptoms
○ Progressive fever
○ Anorexia (hamburger sign )
○ Nausea, vomiting, diarrhea , and/or constipation
● Abdominal pain: classical presentation
○ Initially, dull migratory periumbilical pain (due to visceral peritoneum irritation)
○ After 4–24 hours, sharp RLQ pain (due to parietal peritoneum irritation by a
distended and inflamed appendix) with rebound tenderness
● Signs of appendicitis
○ Blumberg's sign: rebound tenderness caused upon suddenly ceasing deep
palpation of the RLQ
○ McBurney point tenderness: an area one-third of the distance from the right
anterior superior iliac spine to the umbilicus (in the RLQ)
○ Rovsing's sign: deep palpation of the LLQ causes RLQ referred pain
○ Psoas sign: RLQ pain with extension of the right leg against resistance
(secondary to inflammation of a retrocecal appendix)
○ Obturator sign: RLQ pain with flexion and internal rotation of the right leg

Answer is : D

108) 53 y/o patient presents with sudden severe leg pain 3 hours ago, he can’t feel or move
the lower part of the leg, foot looks pale and cold to the touch, a remarkable history of atrial
fibrillation, what is the initial test to be done ?

A. ESR, CRP lab tests.


B. CT scan.
C. MR Angiography.
D. X-ray.
E. Doppler ultrasound.

Acute limb ischemia (ALI) is a vascular emergency in which the arterial blood supply to one
or more extremities is critically reduced. Arterial thrombosis and cardiac emboli are
responsible for the majority of cases. The typical signs and symptoms of ALI include ​p​ain,
p​allor, ​p​ulselessness, ​p​oikilothermia, ​p​aralysis, and ​p​aresthesia of the limb distal to the
site of vascular occlusion (the 6 Ps).

Diagnostics
● Tests to confirm the diagnosis and identify the site(s) of occlusion
○ Best initial test: arterial and venous Doppler
■ Diminished or absent Doppler flow signal distal to site of occlusion.
○ Confirmatory test: angiography (DSA, CTA, MRA)
■ Digital subtraction angiography (DSA) is the imaging modality of
choice.
■ Should only be performed if delaying treatment for further imaging
does not threaten the extremity
● Depending on the suspected etiology, other tests may be indicated (e.g.,
echocardiography if an arterial embolism is suspected).

Answer is : E

109) Which of the following is not a subsequent complication of acute pancreatitis ?

A. Renal failure.
B. Pancreatic pseudocysts.
C. Shock.
D. Atrial fibrillation.
E. Pleural effusion.

Complications of pancreatitis

Localized
● Bacterial superinfection of necrotic tissue → fever
○ Diagnosis: CT-guided percutaneous drainage + culture of the aspirate
○ Treatment: surgical debridement, antibiotics
○ High mortality rate; multiple organ failure in ∼ 50% of cases
● Pancreatic pseudocysts
● Pancreatic abscess
○ Walled-off infected necrotic tissue or pancreatic pseudocyst; typically
develops > 4 weeks after an attack of acute pancreatitis
○ Abdominal CT: visible contrast-enhanced abscess capsule with evidence of
fluid (pus)
○ Ultrasound: complex cystic, fluid collection with irregular walls and septations
○ Treatment: cannulation and drainage; necrosectomy if other measures are
not effective
● Abdominal compartment syndrome
● Blood vessel erosion with bleeding

Systemic
● SIRS, sepsis, DIC
● Pneumonia, respiratory failure, ARDS
● Shock
● Prerenal failure due to volume depletion
● Hypocalcemia
● Pleural effusion, pancreatic ascites
● Paralytic ileus

Answer is : D

110) the main principles of primary health care are all, except:

A. Main cities coverage.


B. Social equity
C. Self-reliance
D. Inter-sectoral coordination
E. People’s involvement in the planning and implementation of health programs.

Primary health care is: Essential health care based on practical, scientifically sound and
socially acceptable methods and technology made universally accessible to individuals and
families in the community through their full participation and at a cost that the community and
the country can afford … (Alma-Ata, 1978)

Primary health care based on the following principles :


● Social equity
● Nation-wide coverage
● Self-reliance
● Inter-sectoral coordination
● People’s involvement in the planning and implementation of health programs

Answer is : A

111) All are characteristics of Ewing sarcoma, except:

A. Extremely aggressive.
B. Hypothermia.
C. More common in males than females.
D. Pain worsens at night.
E. Tender to palpation.

Ewing sarcoma Is highly malignant bone tumor arising from neuroectodermal cells;
associated with various chromosomal translocations of the EWSR1 gene (chromosome 22)

● Epidemiology
○ Incidence: peak at 10–15 years
○ Sex: ♂ > ♀
○ Ethnicity: primarily affects whites
● Clinical features
○ Frequently first manifests with localized pain (progressive, worsens at night),
hyperthermia, and swelling after trauma to the bone (tissue mass that is
tender to palpation and accompanied by erythema)
○ B symptoms are common
● Localization
○ Primary tumor: often diaphysis of long bones (particularly the femur, tibia,
fibula, and humerus) and bones of the pelvis
○ Metastasis: lungs, skeletal system, bone marrow
● Prognosis
○ Extremely aggressive, early metastasis
○ Five-year survival rate of ∼ 70%

B symptoms means :
The presence of fever, night sweats, and weight loss, which together are classically
associated with lymphomas.

Answer is : B

112) In seminal analysis all the following is true, except:

A. pH > 8.0 may indicate inflammation of the prostate.


B. Semen volume increases by 0.4 mL with each day of abstinence.
C. The normal sperm concentration is more than 5million/ml.
D. Up to 10% of all spermatozoa have observable defects.
E. Decreased fructose concentration may indicate ejaculatory duct obstruction.

Semen analysis is an important test in the evaluation of male infertility and is also used to
confirm sterility after a vasectomy. A semen sample is examined for sperm concentration,
morphology, motility, and semen biochemistry (e.g., pH level).

The semen is normally basic (7.2–8.0) :


pH > 8.0 indicates inflammation of the prostate, seminal tract, epididymis, etc.
pH < 7.2 indicates seminal vesicle dysfunction or obstruction of the ejaculatory ducts.

Absent fructose concentration is an indication of a congenital absence of vas


deferens/seminal vesicles, while decreased fructose concentration may indicate ejaculatory
duct obstruction.

Semen volume increases by 0.4 mL with each day of abstinence. Abstinence for 7 days
increases the concentration of sperm by 10–15 million per mL.

Seminal analysis normal findings:


Answer is : C
113) which of these types of pregnancies is sometimes misdiagnosed as normal pregnancy?

A. Complete mole
B. Cervical ectopic pregnancy
C. Ovarian ectopic pregnancy.
D. Abdominal ectopic pregnancy
E. Interstitial ectopic pregnancy.

Interstitial pregnancy describes pregnancy in the interstitial rather than extrauterine part of
the tube. Interstitial pregnancies represent 2-3% of ectopic pregnancies. Interstitial
pregnancy can be misdiagnosed by ultrasound as normal intrauterine pregnancy. It tends to
present early and suddenly and often there is catastrophic haemorrhage before diagnosis is
made.

Interstitial pregnancies are rare but dangerous types of ectopic pregnancy.

Ectopic pregnancy is a complication of pregnancy in which the embryo attaches outside the
uterus.

Localization
1. Fallopian tube (96% of cases): ampulla >> isthmus > fimbriae > interstitial pregnancy
2. Ovary (3% of cases)
3. Abdomen (1% of cases)
4. Cervix (very rare)

Answer is : E

114) Which of the following is karyotypes is found in complete mole pregnancy ?

A. 46XX.
B. 69XXY.
C. 92XXXX.
D. 23X.
E. 69XXX.

A complete mole is caused by a single sperm (90% of the time) or two (10% of the time)
sperm combining with an egg which has lost its DNA. In the first case, the sperm then
reduplicates, forming a "complete" 46 chromosome set. The genotype is typically 46,XX
(diploid) due to the subsequent mitosis of the fertilizing sperm but can also be 46,XY
(diploid). 46,YY (diploid) is not observed. In contrast, a partial mole occurs when a normal
egg is fertilized by one or two sperm which then reduplicates itself, yielding the genotypes of
69,XXY (triploid) or 92,XXXY (tetraploid).
Answer is : A

115) A 39 y/o 12 weeks gestation came with severe, persistent nausea and vomiting, vaginal
bleeding, and grape-like discharge, medical examination shows large for gestational age
uterus, next step in diagnosis ?

A. MRI.
B. Transvaginal ultrasound.
C. Uterine evacuation.
D. β-HCG level measurement.
E. Colposcopy.

The symptoms of the patient is suggestive of molar pregnancy.

Diagnostics
● Laboratory tests: β-HCG level measurement (initial test of choice), which should
reveal β-HCG that is markedly elevated (higher than expected for gestational age)
● Transvaginal ultrasound
○ Complete hydatidiform mole
■ Theca lutein cysts
■ Echogenic mass interspersed with many hypoechogenic cystic spaces
that represent hydropic villi (referred to as “swiss cheese,” “bunch of
grapes,” or “snowstorm”)
■ No amniotic fluid
■ Lack of fetal heart tones
○ Partial hydatidiform mole
■ Fetal parts may be visualized.
■ Fetal heart tones may be detectable.
■ Amniotic fluid is present.
■ Increased placental thickness
● Uterine evacuation (for definite diagnosis and treatment): histopathological
examination of evacuated uterine specimen
● Chest x-ray: in case of dyspnea or chest pain

Answer is : D

116) All the following are causes of postpartum haemorrhage, except:

A. Uterine atony.
B. Placental retention.
C. Vaginal trauma.
D. VWD.
E. Microcephaly.
Aetiology of postpartum haemorrhage :
The causes of PPH have been described as the "four T's":
● Tone: uterine atony, distended bladder.
● Trauma: lacerations of the uterus, cervix, or vagina.
● Tissue: retained placenta or clots.
● Thrombin: pre-existing or acquired coagulopathy.
The most common cause of PPH is uterine atony, followed by retained placenta.

Answer is : E

117) Brandt-Andrews maneuver is used to achieve which of the following ?

A. Stopping the bleeding from abruptio placenta.


B. Separation of the placenta from uterus.
C. Relief braxton-hicks contractions.
D. Dilation of the cervix.
E. Visualizing the embryo on U/S.

Brandt-Andrews maneuver​ is a technique for expelling the placenta from the uterus
during the third stage of labor. One hand puts gentle traction on the cord while the other
presses the anterior surface of the uterus backward.

Answer is : B
118) 23 y/o female presents with a fever, nausea, vomiting, lower abdominal pain, pelvic
exam shows severe bilateral tenderness, lab test shows elevated ESR and leukocytosis,
what is the most likely diagnosis:

A. PID.
B. Ectopic pregnancy.
C. Appendicitis.
D. Ovarian torsion.
E. Cervical cancer.

Ectopic pregnancy, Appendicitis and Ovarian torsion usually present with unilateral
tenderness.

Pelvic inflammatory disease (PID) is caused by a bacterial infection that spreads beyond the
cervix to infect the upper female reproductive tract, including the uterus, fallopian tubes,
ovaries, and surrounding tissue. The most common pathogens that cause PID are
Chlamydia and Gonococci.

Clinical features
● Lower abdominal pain (generally bilateral), which may progress to acute abdomen
● Nausea, vomiting
● Fever
● Dysuria, urinary urgency
● Menorrhagia and metrorrhagia
● Dyspareunia
● Abnormal vaginal discharge

Answer is : A

119) Which of the following is NOT a risk factor for developing pre-eclampsia ?

A. Chronic renal disease


B. Multiple gestation
C. Low BMI.
D. Diabetes mellitus
E. Age >40 years

Risk factors :
● General risk factors
○ Thrombophilia (e.g., antiphospholipid syndrome)
○ Obesity (BMI ≥ 30)
○ Age < 20 or > 40 years
○ African race
○ Diabetes mellitus or gestational diabetes
○ Chronic hypertension
○ Chronic renal disease (e.g., SLE)
● Pregnancy-related risk factors
○ Nulliparity
○ Previous preeclampsia
○ Family history
○ Multiple gestation (twins)
○ Chromosomal anomalies or congenital structural anomalies
○ Hydatidiform moles

Answer is : C

120) All the following are contraindications for normal vaginal delivery, except:

A. Active primary genital herpes


B. Transverse fetal lie.
C. Complete placenta previa.
D. Twin pregnancy.
E. less than 4 on Bishop's score

Vaginal delivery contraindications :


Absolute contra-indications include:
● Severe degree of placenta praevia.
● Transverse fetal lie.
● Severe cephalopelvic disproportion.
● Cervix <4 on Bishop's score - can be overcome by ripening with prostaglandins.
Relative contra-indications include:
● Active primary genital herpes infection.
● High and floating fetal head (risk of prolapsed cord).

Answer is : D

121) In infectious diseases, the period from exposure to disease’s onset is called ?

A. Post exposure
B. Latency.
C. Incubation.
D. Induction.
E. Pre-latency.
Infectious diseases timeline:

Answer is : D

122) Which of the following microorganisms is associated with seborrheic dermatitis ?

A. Staph aureus.
B. Malassezia. —-
C. Staph epidermidis
D. Candida albicans
E. Lactobacillus acidophilus

Seborrheic dermatitis is a common chronic inflammatory skin condition that affects areas with high
sebaceous activity (e.g., scalp). The etiology remains unknown, but microbial colonization of the skin
(esp. Malassezia), immunological factors, climate, or stress have been implicated.

Malassezia species (e.g., M. furfur most commonly, also M. globosa and M. restricta), which are
normal skin commensals, have been associated with seborrheic dermatitis. Abnormal immune
responses to these commensals may be related to the development of seborrheic dermatitis. The
pathogenesis and exact mechanism of this association is still not fully understood.

Answer is : B
123) A 67y female recently diagnosed with DVT in his right leg, she started heparin therapy.
after 7 days she developed a new thrombosis in his left leg, what is the next step:

A. Stop heparin and start warfarin.


B. Add warfarin.
C. Stop heparin.
D. Add direct thrombin inhibitor.
E. Add Vitamin K.

This is most likely a case of heparin induced thrombocytopenia, Heparin should be stopped
immediately, you can’t add warfarin because of the transient procoagulant effect.

Treatment of HIT:
● Treat presumptively as soon as HIT is suspected, and continue treating until confirmatory
testing results
● Immediate discontinuation​ of all heparin-containing products
● Send confirmatory test (ELISA, serotonin release assay, or functional assay against PF4
autoantibody)
● Initiate non-heparin anticoagulation (often a direct thrombin inhibitor [e.g., argatroban] is
first-line or synthetic heparin; NOT warfarin ) until platelet count has normalized (> 150, 000
platelets/μL)
● Transition to warfarin or other long-term oral anticoagulant for at least two months (often
longer if patient had thrombosis)
● Inform patient of their heparin allergy and to never receive any heparin products for life

Answer is : C

124) A 32y female presented with cough for the last 2 months and enlarged lymph nodes in
the neck, Lab tests shows: high serum calcium level and high serum ACE, what is the
diagnosis ?

A. HIV infection.
B. Leukemia.
C. TB.
D. Osteoporosis.
E. Sarcoidosis.

Features of chronic sarcoidosis


● In rare cases, preceded by acute sarcoidosis
● Gradual disease course; may be recurrent or progressive

Pulmonary (most common)


● Often asymptomatic in the early stages
● Dry cough​, exertional dyspnea
● Mild rales on pulmonary auscultation

Extrapulmonary
● Common
○ Peripheral ​lymph nodes​ are the most frequent site of extrapulmonary
manifestation (40%).
○ Eyes (25%): granulomatous uveitis; blurred vision (ocular sarcoidosis)
○ Skin (25%)
■ Lupus pernio: pathognomonic, extensive, purple skin lesions
(violaceous skin plaques) on the nose, cheeks, chin, and/or ears; also
referred to as epithelioid granulomas of the dermis
■ Scar sarcoidosis: inflamed, purple skin infiltration and elevation of old
scars or tattoos
● Other manifestations
○ Musculoskeletal ; bone lesions
○ Nervous system (neurosarcoidosis): cranial nerve palsy (7th cranial nerve
palsy is the most common), diabetes insipidus, meningitis, hypopituitarism
○ Heart
○ Liver
○ Kidneys
○ Spleen

Features of sarcoidosis are GRUELING: Granulomas, aRthritis, Uveitis,


Erythema nodosum, Lymphadenopathy, Interstitial fibrosis, Negative TB
test, and Gammaglobulinemia.

Laboratory tests
● Acute sarcoidosis
○ ↑ Inflammatory markers
○ Findings typical for sarcoidosis are absent (e.g., ↑ ACE, ↑ IgG, ↑ calcium)
● Chronic sarcoidosis
○ ↑ ​Calcium ​due to elevated levels of 1,25-(OH)2-vitamin D3
○ ↓ CD4+ T cells: T helper cells are consumed during granuloma formation →
CD4+ levels are low in serum and high in bronchoalveolar lavage.
○ ↑ IgG (approx. 50% of patients)
○ ↑ ​Angiotensin-converting enzyme (ACE)​ blood levels; may be used to monitor
disease activity and therapy
○ ↑ Inflammatory markers, possible lymphopenia
○ Urine analysis: hypercalciuria

Answer is : E
125) Which of the following is a favorable prognostic factor for AML?

A. Acute Promyelocytic leukemia.


B. Acute Megakaryocytic leukemia.
C. High LDH.
D. > 60 years old.
E. Chromosome translocation (6,9).

Unfavorable prognostic factors of acute leukemia :

Favorable prognostic factors of acute leukemia :

Answer is : A

126) A 47y patient presented with DKA, Glucose : 650mg/dL (Normal: 100 - 140mg/dL),
Serum potassium: 5.2 mEq/L (Normal - 3.5 to 5.5 mEq/L) , Which of the following should be
added to Iv fluid beside insulin ?

A. Glucagon.
B. Sodium.
C. Chloride.
D. Potassium.
E. Hydrocortisone.

Diabetic ketoacidosis can often present with hyperkalemia as the body attempts to
compensate its acidotic state. This high serum potassium is not necessarily indicative of the
total body potassium, and patients will often require supplemental potassium as the acidosis
corrects.

The body shifted some of the intracellular potassium to the extracellular compartment in
exchange with hydrogen ion, so potassium should be added to help shift the glucose inside
the cell.

Answer is : D

127) A 30y woman multipara came to the gynecologist for her regular checkup. a pap smear
was examined and found to have A high grade squamous intraepithelial lesion, The
appropriate treatment is which of the following ?

A. Hysterectomy.
B. Loop electrical excision procedure.
C. Radiotherapy.
D. Repeat at 12 months
E. Repeat at 3 years.

The best appropriate management for HISL for patient who is more than 24 years old.
According to the bethesda system :

This the algorithm for bethesda system:


Answer is : B
128) A 12noths old infant brought to Emergency department after intermittent episodes of
pain and crying, the parents noticed he have elongated Mass on his abdomen, they found
thick blood on his diaper, what is the next appropriate treatment ?

A. Air enema.
B. Colonoscopy.
C. Diverticulectomy.
D. Barium enema.
E. Barium meal.

This infant is having an intussusception


He presented with the classic triad of abdominal pain, a palpable sausage-shaped
abdominal mass, and blood per rectum.

Currant jelly stool is a thick mixture of sloughed mucosa, mucus, and blood, which usually indicates
long standing invagination with bowel ischemia.

Treatment
● Initial steps: nasogastric decompression and fluid resuscitation
● Nonsurgical management (performed under continuous ultrasound or fluoroscopic guidance)
○ Air (pneumatic) enema: treatment of choice
○ Hydrostatic reduction: normal saline (or water-soluble contrast enema)
○ Observe for 24 hours post-reduction, as there is a small risk of perforation and
recurrence is common during this period
● Surgical reduction
○ Indications
■ When a pathological lead point is suspected
■ Failed conservative management
■ Suspected gangrenous or perforated bowel
■ Critically ill patient (e.g., shock)
○ Open or laparoscopic method
■ Hutchinson maneuver: manual proximal bowel compression and reduction of
intussusception
■ For necrotic bowel segments: Resection and end-to-end anastomosis

Answer is : A

129) A 65 years old man presents complains of headache, his blood pressure was 210/105,
urinalysis reveals a hematuria and proteinuria, renal biopsy was taken and shows: “onion
skin” appearance, What is the Pathophysiology of his disease?
A. Hyperplastic arteriolosclerosis.
B. Atherosclerosis.
C. Hyaline arteriolosclerosis.
D. Mönckeberg's arteriosclerosis.
E. Arterial rupture.

Hyperplastic arteriolosclerosis
● Proliferation of subendothelial smooth muscle cells in response to very high blood pressure
● Biopsy with H&E stain: "onion-skin" appearance of the arteriole
● Cause: malignant hypertension

Answer is : A

130) A 47 years old male with medical history remarkable for mitral valve replacement 4
months ago, developed a high grade fever and chills, no cough, lung sounds are normal,
hemorrhages underneath fingernails was found, What is the next step ?

A. Chest CT scan.
B. Broad Spectrum antibiotics.
C. Echocardiogram.
D. NSAIDs.
E. Blood culture.

Blood culture is the next step when suspecting infective endocarditis, before administering
antibiotics

Constitutional symptoms of Infective endocarditis


● Fever and chills (∼ 90% of cases), tachycardia
● General malaise, weakness, night sweats, weight loss
● Dyspnea, cough, pleuritic chest pain
● Arthralgias, myalgias

Cardiac manifestations
● New heart murmur development or change to a preexisting one
○ Mitral valve regurgitation → holosystolic murmur, loudest at the heart's apex, and
radiates to the left axilla
○ Tricuspid valve regurgitation → holosystolic murmur; loudest at the left sternal border
; seen in IV drug users and concomitant HIV infection, immunosuppressed patients,
and patients with central venous catheters
○ Aortic valve regurgitation → early diastolic murmur; loudest at the left sternal border
● Signs of progressive heart failure (e.g., dyspnea, edema)
● Signs of acute cardiac decompensation (pulmonary edema)
● Arrhythmias
○ Suspect perivalvular abscess in patients with infective endocarditis who develop a
new conduction abnormality (e.g., a third-degree atrioventricular block)

Extracardiac manifestations
● These manifestations are mainly caused by bacterial microemboli and/or the
precipitation of immune complexes .
● Petechiae​; especially splinter hemorrhages (hemorrhages underneath fingernails)
● Janeway lesions: non-tender, erythematous macules on palms and soles (due to
microemboli and microabscesses with neutrophilic capillary infiltration)
● Osler nodes: painful nodules on pads of the fingers and toes
● Roth spots: retinal hemorrhages with pale centers
● Signs of acute renal injury, including hematuria and anuria
● Splenomegaly and possible LUQ pain
● Neurological manifestations (e.g., seizures, paresis)
● Signs of pulmonary embolism (e.g., dyspnea)
● Possible arthritis

Answer is : E

131) A 4 years old boy is having a generalized edema, urinalysis shows proteinuria, no
hematuria, what is the most likely diagnosis ?

A. Post-streptococcal glomerulonephritis.
B. HUS.
C. Heart failure.
D. Minimal change disease.
E. Amyloidosis

Minimal change disease is a disease affecting the kidneys which causes a nephrotic
syndrome.Nephrotic syndrome leads to the loss of significant amounts of protein in the urine,
which causes the widespread oedema (soft tissue swelling) and impaired kidney function
commonly experienced by those affected by the disease. It is most common in children and
has a peak incidence at 2 to 6 years of age.

Post-streptococcal glomerulonephritis usually presents with hematuria.

Answer is : D

132) A 32y female came to ED because of watery diarrhea and fever for 2 days after eating
fresh seafood in a restaurant on the beach, what is the most likely causative organism?

A. C. tetani.
B. Staph aureus.
C. Salmonella.
D. Vibrio parahaemolyticus.
E. Lactobacillus acidophilus.
Vibrio parahaemolyticus is a curved, rod-shaped, Gram-negative bacterium found in
brackish, saltwater, which, when ingested, causes gastrointestinal illness in humans, it
causes gastrointestinal illness characterized by nausea, vomiting, watery or bloody diarrhea,
and fever usually within 24 hours of consumption.

Answer is : D

133) A 39y woman complains of palpitations for 40 days she feels dizzy physical
examination reveals non pitting edema under the knees, lab tests shows: high TSH and low
T4, what is the diagnosis from the following?

A. Graves disease.
B. Hashimoto thyroiditis.
C. Pituitary tumor.
D. Systolic heart failure.
E. Diastolic heart failure

Graves' disease, also known as toxic diffuse goiter, is an autoimmune disease that affects
the thyroid. It frequently results in and is the most common cause of hyperthyroidism.

Symptoms:
● Weight loss despite an increased appetite.
● Weight gain.
● Increased or decreased appetite.
● Irritability.
● Weakness and fatigue.
● Diarrhoea ± steatorrhoea.
● Sweating.
● Tremor.
● Mental illness: may range from anxiety to psychosis.
● Heat intolerance.
● Loss of libido.
● Oligomenorrhoea or amenorrhoea.

Symptoms:
● Palmar erythema.
● Sweaty and warm palms.
● Fine tremor.
● Tachycardia - may be atrial fibrillation and/or heart failure (common in the elderly).
● Hair thinning or diffuse alopecia.
● Urticaria, pruritus.
● Brisk reflexes.
● Goitre.
● Proximal myopathy (muscle weakness ± wasting).
● Gynaecomastia.
● Lid lag (may be present in any cause of hyperthyroidism).

Answer is : A

134) A 52y coal miner complains of chronic cough for 3 months, and exertional dyspnea,
chest x-ray shows fine nodular opacifications in upper lung zone, What is the most likely
diagnosis ?

A. Pneumonia.
B. Pneumoconiosis.
C. Syphilis.
D. Pott’s disease.
E. Atelectasis.

Pneumoconiosis is the general term for a class of interstitial lung diseases where inhalation
of dust has caused interstitial fibrosis. Pneumoconiosis often causes restrictive impairment

Common clinical features


● Cough and progressive exertional dyspnea are the most common symptoms.
● Pulmonary fibrosis, cor pulmonale, lung cancer, and Caplan syndrome are possible
complications of all types of pneumoconiosis

● Coal workers' pneumoconiosis (also known as black lung disease or black lung):
○ A more severe form of anthracosis
○ Occurs only with prolonged exposure to large amounts of coal
○ Carbon-laden macrophages induce inflammation
○ Characterized by chronic bronchitis that progresses to progressive massive
pulmonary fibrosis

X-ray: fine nodular opacifications (< 1 cm) in upper lung zone

Answer is : B

135) Which of the following disorders is a type IV hypersensitivity reaction?

A. Graves disease.
B. Anaphylaxis.
C. Rheumatic fever.
D. Contact dermatitis.
E. Serum sickness.
Anaphylaxis is a Type 1 hypersensitivity.
Rheumatic fever and Graves disease are Type 2 hypersensitivity.
Serum sickness is a type 3 hypersensitivity.

Answer is : D

136) A 14y male brought to the clinic after fatigue and weakness, history is remarkable for
Sickle cell disease, his mother says he had “common cold” a week ago, physical
examination shows: scleral icterus, his CBC shows:
Hb: 5 g/dL (N=13.5-17.5 g/dL)
Platelets: 190,000/mm3 (N=150,000-400,000/mm3)
WBC: 12,500/mm3 (N=4,500-11,000/mm3)
Reticulocytes: 0.1% (N=0.5-1.5%)
What is the likely cause of his anemia ?

A. Influenza virus.
B. Hemolytic Anemia.
C. Hemorrhagic shock.
D. Acute renal failure.
E. Aplastic Crisis.

A plastic crisis after parvovirus B19 infection in SCC patient is a complication to be


considered,

Reticulocytopenia is usually a result of viral parvovirus B19 infection, which invades and
destroys red blood cell precursors and halts the red cell production.

If infection occurs in individuals with sickle cell anemia, spherocytosis, or beta thalassemia, it
will lead to incorporation of two anemia-induced mechanisms: decreased red cell production
and hemolysis. The result is a rapid and severe anemia (aplastic crisis)

Answer is : E

137) A 29y woman brought by her husband to the hospital unresponsive, he says her
medical history is unremarkable, she works as a nurse, lab tests shows serum glucose:
42mg/dL. What the most likely diagnosis?

A. Type 2 diabetes.
B. Type 1 diabetes.
C. Fictitious Hypoglycemia.
D. Islet cell adenoma.
E. Reactive hypoglycemia.

Factitious hypoglycemia or self-induced hypoglycemia can be seen in healthcare workers or


in relatives who care for diabetic family members at home.
Answer is : C

138) 19y male presented with pain and limited motion range in his left thumb after falling
when playing football 2 days ago, a scaphoid fracture was suspected, x-ray was
non-contributory, what is the next step?

A. NSAIDs.
B. Repeat x-ray
C. Open Reduction Internal Fixation
D. Splint.
E. Discharge home

Answer is : D

139) A 33y male from Germany presented with diarrhea and fever and abdominal colicky
pain, after returning from India yesterday, What the most likely cause:

A. Cholecystitis.
B. Schistosomes.
C. Volvulus.
D. Enterotoxigenic E.Coli.
E. Pancreatitis.

Traveler's diarrhea
● Infections which typically occur in patients with a history of recent travel
● Very common while traveling in Asian (“Delhi belly”), African, and Latin American
countries ("Montezuma's Revenge") or Middle east.
● A major cause of diarrhea among children in developing countries
● May be exudative-inflammatory diarrhea or secretory diarrhea
● Most commonly caused by enterotoxigenic Escherichia coli (ETEC)
● Other pathogens: Campylobacter jejuni, Shigella spp. , Salmonella spp., other E. coli
strains (e.g., EAEC), protozoa (e.g., Giardia), viral diarrhea (norovirus, rotavirus,
astrovirus)

Answer is : D

140) Which of the following drugs do NOT cause photosensitivity reaction?

A. Cefixime.
B. Methotrexate
C. Piroxicam
D. Isotretinoin
E. Amiodarone

DrugsThat Sensitize the Skin to Sunlight :

Antianxiety drugs
● Alprazolam
● Chlordiazepoxide

Antibiotics
● Quinolones
● Sulfonamides
● Tetracyclines
● Trimethoprim

Antidepressants
● Tricyclic antidepressants

Antifungal drugs (taken by mouth)


● Griseofulvin

Antihyperglycemics
● Sulfonylureas

Antimalarial drugs
● Chloroquine
● Quinine

Antipsychotics
● Phenothiazines

Chemotherapy drugs
● Dacarbazine
● Fluorouracil
● Methotrexate
● Vinblastine

Diuretics
● Furosemide
● Thiazides

Drugs used to treat acne (taken by mouth)


● Isotretinoin

Heart drugs
● Amiodarone
● Quinidine
● Pain-relief drugs (analgesics)
● NSAIDs (especially piroxicam and ketoprofen)

Skin preparations
● Antibacterials (such as chlorhexidine and hexachlorophene)
● Coal tar
● Fragrances
● Furocoumarin-containing plants, such as limes, celery, and parsley
● Sunscreens

Answer is : A

141) 5y old boy presented with high fever and red tongue with white dots on it, his lips are
cracked, conjunctivitis, hands and feet are red and swollen, what is the appropriate
treatment ?

A. Penicillin.
B. Paracetamol.
C. Aspirin.
D. Acyclovir.
E. Antihistamine..

When kawasaki disease is Clinically diagnosed, Initial high doses of aspirin are given for the
anti-inflammatory effect until the fever has subsided. Treatment is then continued with a
lower dosage for its antiplatelet effect. Kawasaki disease is a rare exception to the
contraindication of giving children aspirin, which is associated with Reye syndrome.

Clinical features
● Clinical diagnosis requires fever for at least 5 days and:
○ ≥ 4 other specific symptoms, or
○ < 4 specific symptoms if the coronary arteries are involved
● Specific symptoms
○ Erythema and edema of hands and feet, including the palms and soles (the
first week)
■ Possible desquamation of fingertips and toes (after 2–3 weeks)
○ Polymorphous rash, originating on the trunk
○ Painless bilateral “injected” conjunctivitis without exudate
○ Oropharyngeal mucositis
■ Erythema and swelling of the tongue (strawberry tongue)
■ Cracked and red lips
○ Cervical lymphadenopathy (mostly unilateral)
● Nonspecific symptoms may precede the onset of Kawasaki disease (e.g., diarrhea,
fatigue, abdominal pain)

Answer is : C

142) A 23y old man presented to ED with pain and swelling in his right hand it, his hands
looks disturbed, after punching a wall while talking on the phone, A fracture is suspected,
X-ray would show a fracture of which bone ?

A. Scaphoid bone.
B. Ulnar Head.
C. Radial head.
D. 5th metacarpal bone.
E. Lunate bone.

A boxer's fracture is the break of the 5th metacarpal bones of the hand near the knuckle.
Occasionally it is used to refer to fractures of the 4th metacarpal as well.

It’s usually caused by Hitting a hard object with a closed fist.

Answer is : D

143) A 32y old woman complains of right upper quadrant pain and fever for 2 months,
physical examination shows cervical motion tenderness, laparoscopy shows hepatic capsule
adhesions, What is the most likely causative organism ?

A. Neisseria gonorrhoeae
B. Hepatitis B
C. E. coli
D. Chlamydia trichomonas.
E. Toxoplasma.

The patient had symptoms of prolonged PID which progressed to Fitz-Hugh-Curtis


syndrome, the most common causative organism of PID is Chlamydia trichomonas.

Fitz-Hugh–Curtis syndrome is a rare complication of pelvic inflammatory disease (PID)


involving liver capsule inflammation leading to the creation of adhesions.
Fitz-Hugh–Curtis syndrome occurs almost exclusively in women. It is usually caused by
Chlamydia trachomatis (Chlamydia) or through other bacteria such as Neisseria
gonorrhoeae (Gonorrhea), Bacteroides, Gardnerella, E. coli and Streptococcus have also
been found to cause Fitz-Hugh-Curtis syndrome on occasion.

Answer is : D

144) Koplik spots are seen in an infection with which of the following viruses?

A. Rhinovirus.
B. Rubella virus.
C. Poliovirus
D. Mumps virus.
E. Paramyxovirus.

Measles (Rubeola) is a highly infectious disease that is caused by a paramyxovirus. There


are two phases of disease: a catarrhal (prodromal) stage and an exanthem stage. The
catarrhal stage is characterized by a fever with conjunctivitis, coryza, cough, and
pathognomonic Koplik spots on the buccal mucosa.

Answer is : E

145) What type of Immunoglobulin are secreted in the milk ?

A. IgA
B. IgG
C. IgM
D. IgE
E. IgD

Passive immunity is provided through colostrum and breast milk, which contain IgA
antibodies that are transferred to the gut of the infant, providing local protection against
disease causing bacteria and viruses until the newborn can synthesize its own antibodies.

Answer is : A

146) All the following are clinical features of Hemophilia, except?

A. Melena
B. Petechiae.
C. Hemarthrosis
D. Hematuria
E. Neck stiffness
Clinical features
● Spontaneous or delayed onset bleeding (joints, muscular and soft tissue, mucosa) in
response to different degrees of trauma
○ Repeated hemarthrosis → joint destruction
● Further sites/symptoms of hemorrhage:
○ CNS (e.g., headache, neck stiffness)
○ Gastrointestinal tract (e.g., melena, hematemesis)
○ Genitourinary system (e.g., hematuria)
○ Oral mucosa bleeding, epistaxis, excessive bleeding following small
procedures (e.g., dentist procedures)
● Female carriers may show mild symptoms.

Petechial bleeding is a common sign of platelet disorders, NOT coagulation


disorders such as hemophilia!

Answer is : B

147) All the following are causes of tall stature, except:

A. Gigantism
B. Marfan syndrome
C. Beckwith-Wiedemann syndrome
D. Turner syndrome.
E. 47,XYY syndrome

causes
● Marfan syndrome
● Homocystinuria
● Fragile X syndrome (Martin-Bell syndrome)
● Neurofibromatosis type 1
● Klinefelter syndrome (47,XXY)
● 47,XYY syndrome
● 47,XXX syndrome
● Beckwith-Wiedemann syndrome
● Gigantism

Answer is : D
148) on the assessment of a newborn for APGAR score, you notice the following:
Weak cry
Some flexion of arms and legs.
cries to stimulation
heart rate 145
pallor all over the body and extremities
What is his APGAR score?

A. 3
B. 5
C. 4
D. 7
E. 6

This neonate score is :


Activity = some flexion = 1 points
Pulse = >100 = 2 points
Grimence = crying when stimulated = 2 points
Appearance = pale body = 0 point
Respiration = slow and irregular = 1 point

Total = 6

Answer is : E
149) Which of the following is a zoonotic disease ?

A. HIV.
B. TB.
C. Toxoplasma.
D. Cholera
E. Mumps

Zoonoses (also known as zoonoses and zoonotic diseases) are infectious diseases caused
by bacteria, viruses and parasites that spread between animals (usually vertebrates) and
humans.

Toxoplasmosis is a zoonotic infection of animals caused by the protozoan parasite


Toxoplasma gondii.

Answer is : C

150) Which of the following is not in the jordanian national vaccination program?

A. Measles
B. Mumps.
C. TB.
D. Rhinovirus.
E. Diphtheria

Diphtheria vaccine is in the DaTP


TB vaccine is the BCG
Mumps vaccine is the MMR
Answer is : D

151) 56 y/o man presented with chest pain that radiates to the shoulders, the patient have a
history of chronic hypertension, the next step is:

A. Cardiac biomarkers.
B. ECG.
C. Nitroglycerin sublingual.
D. Morphine.
E. furosemide.

ECG should be performed immediately once Acute coronary syndrome is suspected,


followed by measurement of cardiac biomarkers. Further diagnostic workup (e.g.,
echocardiography) depends on the results of initial evaluation and further risk stratification
(e.g., TIMI score).

Nitroglycerin sublingual is contraindicated if inferior wall infarct (due to risk for hypotension),
so you need to perform ECG before giving NG

Answer is : B

152) All the following are extrapulmonary manifestations of chronic sarcoidosis, except:

A. Iridocyclitis
B. Erythema nodosum.
C. Splenomegaly
D. Nephrolithiasis
E. Liver granulomas

Erythema nodosum appears in acute sarcoidosis.

Sarcoidosis is a multisystem disorder characterized by noncaseating granulomatous


inflammation. It is classified as either acute or chronic

Extrapulmonary manifestations of chronic sarcoidosis:


● Common:
○ Peripheral lymph nodes are the most frequent site of extrapulmonary
manifestation (40%).
○ Eyes (25%): iridocyclitis
○ Skin (25%)
■ Lupus pernio: pathognomonic, extensive, purple skin lesions
(violaceous skin plaques) on the nose, cheeks, chin, and/or ears; also
referred to as epithelioid granulomas of the dermis
■ Scar sarcoidosis: inflamed, purple skin infiltration and elevation of old
scars or tattoos
● Other manifestations:
○ Musculoskeletal (E.g., arthralgias/arthritis.); bone lesions
○ Nervous system (Cranial nerve palsy (7thcranial nerve palsy is the most
common), diabetes insipidus, meningitis, hypopituitarism)
○ Heart (restrictive cardiomyopathy, pericardial effusion, AV block, dysrhythmia)
○ Liver (Liver granulomas)
○ Kidneys (e.g., nephrocalcinosis and nephrolithiasis) Most commonly related
to calcium metabolism
○ Spleen (Splenomegaly occurs in 33% of cases)

Answer is : B

153) A 46 y/o male came to the clinic after dizziness and epistaxis, after many separate BP
measurements and lab investigations, he was diagnosed with primary hypertension, what is
the most common cause of primary hypertension:

A. Idiopathic.
B. Conn syndrome.
C. Cushing syndrome.
D. Renal failure .
E. Hyperthyroidism.

Causes of primary (essential) hypertension


● Accounts for 85–95% of cases of hypertension in adults
● Idiopathic, no specific cause; multifactorial etiology including epigenetic/genetic and
environmental factors
● Non-modifiable risk factors
○ Positive family history
○ Ethnicity
○ Advanced age
● Modifiable risk factors
○ Obesity
○ Diabetes
○ Smoking, excessive alcohol or caffeine intake
○ Diet high in sodium, low in potassium
○ Physical inactivity
○ Psychological stress
Causes of secondary hypertension
● Accounts for 5–15% of cases of hypertension in adults.
● Caused by a specific organic condition
○ Endocrine hypertension
■ Primary hyperaldosteronism (Conn syndrome)
■ Primary hyperparathyroidism
■ Pheochromocytoma
■ Hypercortisolism (Cushing syndrome)
■ Hyperthyroidism
■ Acromegaly
■ Congenital adrenal hyperplasia
○ Renal hypertension
■ Renovascular hypertension (e.g., due to renal artery stenosis)
■ Polycystic kidney disease (ADPKD)
■ Renal failure (renal parenchymal hypertension)
■ Glomerulonephritis
■ Systemic lupus erythematodes
■ Renal tumors
● Others
○ Coarctation of the aorta
○ Obstructive sleep apnea
○ Medication: sympathomimetic drugs, corticosteroids, NSAIDs, oral
contraceptives
○ Recreational drug use: amphetamines, cocaine, phencyclidine

Answer is : A

154) 45 y/o woman known case of crohn’s disease presents with reddish, painful, tender
lumps located in the front of the legs. Other than crohn’s disease, all the following are
causes of erythema nodosum, except:

A. Behcet's syndrome.
B. Oral contraceptives.
C. Idiopathic .
D. Crohn's disease.
E. Heart failure.

Erythema nodosum (EN) is an inflammation of subcutaneous fat caused by a delayed


hypersensitivity reaction. Women in early adulthood are commonly affected.

Etiology
● Idiopathic (most common)
● Infection (e.g. streptococcal pharyngitis, histoplasmosis, coccidioidomycosis, TB,
leprosy)
● Autoimmune diseases (e.g. sarcoidosis, Crohn's disease, ulcerative colitis, Behcet's
syndrome)
● Drugs (oral contraceptives, sulfonamides, iodide)
● Pregnancy
● Malignancy

Answer is : E

155) 67 y/o recently diagnosed with small cell lung cancer, presents with, Weight gain
characterized by central obesity, moon face, buffalo hump, his cortisol level is high, the most
likely diagnosis is:

A. Iatrogenic cushing syndrome.


B. Primary cushing syndrome.
C. Secondary cushing syndrome.
D. Cushing disease.
E. Ectopic cushing syndrome.
Cushing's syndrome, or hypercortisolism, is an endocrine disorder that is most often caused
iatrogenically by the exogenous administration of glucocorticoids

Etiology :

Exogenous (iatrogenic) Cushing's syndrome


● Hypercortisolism as a result of prolonged glucocorticoid therapy
● Most common cause of hypercortisolism

Endogenous Cushing's syndrome


Types Primary hypercortisolism Secondary hypercortisolism
(ACTH-independent
Cushing's syndrome) Pituitary ACTH production Ectopic ACTH
(Cushing's disease) production

Relative 5–10% ∼ 75% ∼ 15%


frequency

Sex ♂ < ♀ (1:4) ♂ < ♀ (1:4) ♂=♀

Causes Autonomous Pituitary adenomas → Paraneoplastic


overproduction of cortisol ACTH secretion syndrome → ACTH
by the adrenal gland → secretion
ACTH suppression ● Small cell
● Adrenal adenomas lung cancer
● Adrenal carcinoma ● Renal cell
● Macronodular carcinoma
adrenal hyperplasia

While the term “Cushing's syndrome” can be applied to any cause of hypercortisolism,
“Cushing's disease” refers specifically to secondary hypercortisolism that results from
excessive production of ACTH by pituitary adenomas!

Answer is : E

156) All the following sentences are true about hypocalcemia, except:

A. Inadequate sun exposure can cause hypocalcemia.


B. Low serum calcium concentrations result in increased membrane excitability
C. Acidosis usually causes symptoms of hypocalcemia.
D. Low magnesium indicates malabsorption.
E. Furosemide increases calcium excretion in the kidney.
Hypocalcemia = total serum calcium concentration < 8.5 mg/dL (< 2.12 mmol/L), or ionized
(free) calcium concentration < 4.65 mg/dL (< 1.16 mmol/L)

pH influences the binding of calcium to serum proteins. This is because Ca2+ ions compete
with H+ ions for binding sites on serum proteins. Acidosis reduces calcium binding, while
alkalosis enhances it.
● ↓ pH → ↑ H+ in serum binding to proteins → ↓ Ca2+ binding to proteins → ↑ ionized
Ca2+ concentration
● ↑ pH​ → ↓ H+ in serum binding to proteins → ↑ Ca2+ binding to proteins → ​↓ ionized
Ca2+ concentration

Answer is : C

157) All the following are advantages of enteral feeding over total parenteral nutrition,
except:

A. Prevents mucosal atrophy.


B. Better choice for bowel obstruction.
C. Less metabolic complications.
D. Lower risk for bacteremia.
E. Easier to perform.

Enteral feeding is ​contraindicated ​in bowel obstruction.

Types of nutrition support:


● First-line: enteral feeding
○ Advantages
■ Easier to perform
■ Metabolic complications occur less often
■ Intestinal motility is stimulated, preventing mucosal atrophy
■ Lower risk of bloodstream infection
● Second-line: parenteral feeding.

The following principle applies in most situations: ​oral before enteral​, ​enteral before
parenteral​!

Answer is : B

158) Which of the following DON’T cause acanthosis nigricans ?

A. PCOS
B. Obesity
C. Diabetes mellitus type 1.
D. Cushing syndrome
E. Oral contraceptives
acanthosis nigricans
● Etiology
○ Endocrinal: ​Diabetes mellitus type 2​, PCOS, Cushing syndrome
○ Obesity: pseudoacanthosis nigricans
○ Familial causes: autosomal dominant inheritance
○ Drug-related causes: glucocorticoids, oral contraceptives etc.
● Clinical findings
○ Brown to black, intertriginous and/or nuchal hyperpigmentation that can turn
into itching, papillomatous, poorly-defined efflorescence
○ Localization: axilla, groin, neck
● Differential diagnosis: malignant acanthosis nigricans

Answer is : C
159) All the following are true about Atrophic gastritis, except:

A. Achlorhydria is a feature of autoimmune type.


B. H.pylori is the most common cause.
C. Can cause gastric cancer.
D. Causes folic acid deficiency.
E. Upper gastrointestinal endoscopy is the test of choice.

Atrophic gastritis is a condition characterized by chronic inflammation of the gastric mucosa


with atrophy, gland loss, and metaplastic changes. It is classified as autoimmune metaplastic
atrophic gastritis (AMAG) and environmental metaplastic atrophic gastritis (EMAG). Chronic
infection with Helicobacter pylori (H. pylori) is the ​most common cause​.

Pathophysiology

AMAG:
● Autoimmune destruction of the parietal cells → Achlorhydria → increased release of
gastrin (due to loss of negative feedback) → G cell hyperplasia → hypergastrinemia
○ Hypergastrinemia may lead to hyperplasia of enterochromaffin-like cells and,
consequently, to an increased risk of carcinoid tumors.
● Autoantibodies against intrinsic factor → ​vitamin B12 deficiency​ → pernicious anemia

EMAG:
● Helicobacter-associated: colonization by H. pylori → decreased production of mucins
→ increased production of gastric acids → inflammation primarily of the antrum→
ascending propagation → shift of the corpus-antrum border → in case of
chronification: atrophy of the gastric glands → ​hypochlorhydria (not achlorhydria)​ and
epithelial metaplasia → increased risk of gastric cancers
● Diet: bacteria in the stomach metabolize nitrates present in food → formation of
N-nitroso compounds (carcinogenic) → epithelial metaplasia → increased risk of
gastric cancers

Answer is : D

160) 54 years old female presents with high grade fever which don’t respond to antipyretics,
Rose-colored spots are seen on her chest, blood culture reveals salmonella typhi, All the
following is true about typhoid fever, except:
A. Incubation period most commonly 7–14 days.
B. Metronidazole is the preferred treatment option.
C. Humans are the main reservoir for Salmonella typhi.
D. Typhoid fever is a systemic disease.
E. Relative bradycardia..

Typhoid and paratyphoid fever are infectious diseases caused by the bacteria Salmonella
typhi and Salmonella paratyphi. Transmission occurs via the fecal-oral route. The incubation
period is typically 7–21 days, although it may be as long as 30 days.

Treatment
● First-line treatment: fluoroquinolone antibiotics (e.g., ciprofloxacin)
● Azithromycin, if resistance to fluoroquinolone antibiotics is suspected (e.g., in
patients with infection acquired from certain regions, such as South Asia)
● Third-generation cephalosporins (e.g., ceftriaxone) are preferred for severe infection.

Answer is : B

161) Which of the following causes diastolic heart failure :

A. Myocardial infarction.
B. Hypertension.
C. Myocarditis.
D. Pericardial tamponade.
E. Aortic stenosis.

Systolic dysfunction is CHF with reduced stroke volume and ejection fraction (EF)
● The ejection fraction is the percentage of blood being pumped by the left ventricle
during each contraction (stroke volume divided by end-diastolic volume); normally ∼
55%

Diastolic dysfunction is CHF with reduced stroke volume and preserved ejection fraction.
● The ejection fraction is not reduced because both the left ventricular end-diastolic
volume and stroke volume are reduced.

Causes of congestive heart failure:


Answer is : D

162) String sign of Kantour (small bowel stenosis) in barium meal indicates which of the
following conditions :

A. Meckel's diverticulum.
B. Atrophic gastritis.
C. Crohn's disease.
D. Hirschsprung's disease.
E. cysto-rectal fistula.

String sign​, or gastrointestinal string sign (also called string sign of Kantour) , is a medical
term for a radiographic finding on an upper GI series, in which the patient is given a
radio-opaque material, such as barium, to drink. X-rays are then taken of the patient's
stomach and intestines.

The gastrointestinal string sign represents a severe narrowing of loop of bowel, in which a
thin stripe of contrast within the lumen looks like a string.

It may be seen in Crohn's disease, hypertrophic pyloric stenosis, carcinoid tumor and colon
cancer. In people with Crohn's Disease, the string sign is caused by incomplete filling of the
intestinal lumen, which results from irritability and spasm associated with severe ulceration.
In such cases, the string sign is most frequently seen at the terminal ileum.
Answer is : C

163) A 32 years old shepard presented with arthralgia and fever, blood culture shows
Brucella spp. Most cases of brucellosis is transmitted by which of the following?

A. Hospital acquired.
B. Person to person.
C. Sexually transmitted.
D. Blood transfusion.
E. Animals to Humans.

Brucellosis is a zoonotic infection caused by different species of Brucella, a genus of


gram-negative bacteria. The most common vectors of the disease are cattle, sheep, goats,
and pigs. Transmission occurs through ingestion of infected animal products, contact with
infected animals, or inhalation of bacteria.

Etiology
● Pathogen: Brucella spp. are facultative intracellular gram-negative coccobacilli
● Transmission
○ Brucellosis is a zoonotic infection (transmitted from animals to humans)
○ Contaminated food, esp. raw/unpasteurized dairy products or meat
○ Contact with infected animals
● Risk factors: occupational or recreational exposure to infected animals and animal
products, e.g., farmers, veterinarians, hunters, slaughterhouse workers, laboratory
personnel

Answer is : E

164) Which of the following sentences is true about mitral stenosis ?

A. Rheumatoid arthritis is the most common cause.


B. Can lead to right ventricular dilation.
C. ECG is the most important diagnostic method.
D. Antibiotics is the first step in management.
E. ACE inhibitors is used.

Mitral stenosis occurs when there is obstruction to flow through the mitral valve separating
the left atrium and left ventricle of the heart.

● The obstruction occurs due to a structural abnormality of the valve. Mitral stenosis
increases left atrial and pulmonary arterial pressure (especially in tachycardia).
● Pulmonary hypertension can lead to ​right ventricular dilation​ and tricuspid
regurgitation. Right ventricular failure results in raised jugular venous pressure, liver
congestion, ascites and peripheral oedema. Left ventricular function and cardiac
output can be normal in isolated mitral stenosis.
● Static blood flow in the left atrium (worsened in atrial fibrillation) can cause
thromboemboli.

Causes:
● Most commonly due to ​rheumatic fever
● Autoimmune diseases: systemic lupus erythematosus, rheumatoid arthritis
● Congenital
● Some conditions may mimic mitral stenosis: bacterial endocarditis of the mitral valve
with large vegetation, left atrial myxoma
● Degenerative aortic stenosis

Echocardiography ​is most important diagnostic method for detecting and assessing valvular
abnormalities

ACE inhibitors and other afterload-reducing drugs are contraindicated because they cause
dilation of peripheral blood vessels, which may lead to cardiovascular decompensation!

Answer is : B

165) All the following are features of Hemochromatosis, except:

A. Atrial fibrillation
B. Arthralgia
C. hepatomegaly
D. Loss of skin color.
E. AV block

Hemochromatosis is a condition that leads to abnormal iron deposition in specific organs.


There are two main types: primary (hereditary) and secondary (e.g., transfusion-related).

Clinical features
● Asymptomatic in 75% of cases
● The onset of symptoms: typically between the 3rd and 5th decade of life
● Abdominal pain, ​hepatomegaly ​→ liver cirrhosis (+ hepatocellular carcinoma)
● Fatigue, lethargy
● Hyperpigmented​, bronze skin
● Signs of diabetes mellitus (polydipsia, polyuria)
● Arthralgia ​, chondrocalcinosis
● Erectile dysfunction, testicular atrophy, loss of libido, amenorrhea
● Features of cardiac hemochromatosis:
○ Cardiomyopathy (restrictive or dilated)
○ Cardiac arrhythmias: paroxysmal ​atrial fibrillation​ (most common), sinus node
dysfunction, complete ​AV block​, atrial and ventricular tachyarrythmias, and
sudden cardiac death
○ Congestive heart failure

Answer is : D
166) All the following can cause hyperkalemia, except:

A. Acidosis.
B. Spironolactone.
C. Tumor lysis.
D. Type I renal tubular acidosis.
E. Type 1 DM.

Type I renal tubular acidosis causes HYPOkalemia.

Hyperkalemia is when Serum potassium level > 5 mEq/L

Etiology
1. Potassium excess
○ Reduced excretion: acute and chronic kidney disease
○ Endocrine causes: hypocortisolism, hypoaldosteronism
○ Drugs: ​potassium-sparing diuretics​, ACE inhibitors, angiotensin receptor
blockers, NSAIDs, and trimethoprim-sulfamethoxazole
○ GI absorption: increased intake (e.g., fresh fruits)
○ Type IV renal tubular acidosis
○ Release from cells: myolysis, ​tumor lysis​, hemolysis
2. Extracellular shift
○ Acidosis ​→ ↑ extracellular H+ → inhibition of the Na+/H+ antiporter → ↓
intracellular Na+ → ↓ sodium gradient inhibits the Na+/K+-ATPase → ↑
extracellular K+ concentration
■ Hyperkalemia → ↑ extracellular K+ concentration → ↑ potassium
gradient stimulates the Na+/K+-ATPase → ↑ extracellular Na+ → ↑
sodium gradient stimulates the Na+/H+ antiporter → ↑ extracellular H+
→ acidosis
○ Insulin deficiency
○ Drugs: succinylcholine , digoxin
3. Pseudohyperkalemia: due to the release of potassium from red blood cell lysis
○ Blood drawn from the side of IV infusion or a central line without previous
flushing
○ Prolonged use of a tourniquet

Answer is : D

167) 59 y/o man on chemotherapy feels weak, and shortness of breath, ECG shows Tall,
peaked T waves, Shortened QT interval, the most likely diagnosis is:

A. Angina.
B. hypercalcemia
C. Chemotherapy side effects.
D. Myocardial infarction.
E. Hyperkalemia.
Clinical features hyperkalemia.
Symptoms usually occur if serum potassium levels are > 7.0 mEq/L or they change rapidly.
● Cardiac arrhythmias (e.g., atrioventricular block, ventricular fibrillation)
● Muscle weakness, paralysis
● ↓ Deep tendon reflexes
● Nausea, vomiting, diarrhea

ECG changes
● Tall, peaked T waves
● P-wave flattening, lengthening of the PR interval
● Shortened QT interval
● Lengthening of the QRS interval

Answer is : E

168) The definitive diagnostic test for pulmonary embolism is:

A. CT pulmonary angiography.
B. Chest X-ray.
C. Echocardiography
D. Pulmonary angiography.
E. Skull X-ray.

Pulmonary embolism (PE) is the obstruction of one or more pulmonary arteries by solid,
liquid, or gaseous masses. In most cases, the embolism is caused by blood thrombi, which
arise from the deep vein system in the legs or pelvis (deep vein thrombosis) and embolize to
the lungs via the inferior vena cava.

Imaging tests for PE


● CT pulmonary angiography (CTPA): ​best definitive diagnostic test
○ Contrast-enhanced imaging of the pulmonary arteries
○ High sensitivity, specificity and immediate evidence of pulmonary arterial
obstruction
○ Visible intraluminal filling defects of pulmonary arteries
○ Wedge-shaped infarction with pleural effusion is almost pathognomonic for
PE
● Chest radiograph
○ Initially often performed to rule out other causes (e.g., pneumonia,
pneumothorax, pericarditis, aortic dissection)
○ Findings that may indicate PE
■ Atelectasis (visible collapse or incomplete expansion of the lung)
■ Pleural effusions
■ Signs of pulmonary embolism (rare)
■ Hampton's hump: wedge-shaped opacity in the peripheral lung
with its base at the thoracic wall; caused by pulmonary
infarction and not specific for PE
■ Westermark sign: embolism leads to diminished perfusion of
downstream lung tissue, which appears hyperlucent on
radiograph.
■ Fleischner sign: prominent pulmonary artery caused by vessel
distension due to a large pulmonary embolus (common in
massive PE)
■ Cardiomegaly
● Echocardiography: to detect right atrium pressure (RAP) signs
○ Venous reflux with dilation of inferior vena cava (also liver congestion in
ultrasound of the abdomen)
○ Tricuspid regurgitation (tricuspid valve insufficiency)
○ ↑ Pulmonary artery systolic pressure
○ Dilatation and hypokinesis of the right ventricle
● Ventilation/perfusion scintigraphy
○ Indication: alternative to CT angiography in patients with severe renal
insufficiency or contrast allergy
○ Method: detects areas of ventilation/perfusion (V/Q) mismatch via perfusion
and ventilation scintigraphy
○ Assessment
■ Perfusion failure in normally ventilated affected pulmonary area
(mismatch) suggests PE
■ Evidence of normal lung perfusion rules out PE → ventilation
scintigraphy superfluous
● Pulmonary angiography
○ Indications: only conducted if CT angiography unavailable
○ Procedure: right heart catheterization → insertion of a catheter into a
pulmonary artery → radiograph after administration of contrast agent

Answer is : A

169) Which of the following conditions is inherited by autosomal recessive fashion ?

A. Marfan syndrome.
B. Hemophilia.
C. Achondroplasia.
D. Cystic fibrosis.
E. Ehlers-Danlos syndrome.

Achondroplasia and Marfan syndrome are inherited by autosomal dominant fashion.


Hemophilia and Ehlers-Danlos syndrome are inherited by X-linked Recessive fashion.

Answer is : D
170) Which of the following is not a component in anion gap calculation ?

A. Cl.
B. Na.
C. K.
D. Mg.
E. HCO3-.

Anion gap is the difference between the concentration of unmeasured anions and the concentration of
unmeasured cations

Calculation
● Anion gap = [Unmeasured anions] - [Unmeasured cations]
○ [Unmeasured anions] = [Total anions] - [Routinely measured anions]
○ [Unmeasured cations] = [Total cations] - [Routinely measured cations]
● Since [Total cations] ≈ [Total anions] to maintain electrical neutrality, the formula can be
rewritten as Anion gap = [Routinely measured cations] – [Routinely measured anions] =
([Na+] + [K+]) - ([Cl-] + [HCO3-]) (Reference range: 10–16 mmol/L)

Answer is : D

171) Which of the following can cause nail clubbing ?

A. Graves disease.
B. COPD.
C. Esophageal carcinoma.
D. Small bowel obstruction.
E. Constrictive pericarditis.

Clubbing, is a deformity of the finger or toe nails associated with a number of diseases,
mostly of the heart and lungs.

Causes
Clubbing is associated with
● Lung disease:
○ Lung cancer, mainly non-small-cell (54% of all cases), not seen frequently in
small-cell lung cancer (< 5% of cases)
○ Interstitial lung disease most commonly idiopathic pulmonary fibrosis
○ Complicated tuberculosis
○ Suppurative lung disease: lung abscess, empyema, bronchiectasis, cystic fibrosis
○ Mesothelioma of the pleura
○ Arteriovenous fistula or malformation
○ Sarcoidosis
● Heart disease:
○ Any disease featuring chronic hypoxia
○ Congenital cyanotic heart disease (most common cardiac cause)
○ Subacute bacterial endocarditis
○ Atrial myxoma (benign tumor)
○ Tetralogy of Fallot
● Gastrointestinal and hepatobiliary:
○ Malabsorption
○ Crohn's disease and ulcerative colitis
○ Cirrhosis, especially in primary biliary cirrhosis
○ Hepatopulmonary syndrome, a complication of cirrhosis
● Others:
○ Graves' disease (autoimmune hyperthyroidism) – in this case it is known as thyroid
acropachy
○ Familial and racial clubbing and "pseudoclubbing" (people of African descent often
have what appears to be clubbing)
○ Vascular anomalies of the affected arm such as an axillary artery aneurysm (in
unilateral clubbing)

Answer is : A

172) A 35 y/o male presented with left loin pain, hematuria and fever, urine analysis shows
struvite crystals, which of the following is the most likely causative organism of her UTI ?

A. Proteus mirabilis.
B. Escherichia coli.
C. Enterobacter.
D. Streptococcus.
E. Helicobacter pylori.
Proteus mirabilis is associated with struvite crystals formation because proteus is urease
positive bacteria, in which it breaks urea to form ammonium, ammonium in turn rises the pH
of urine, alkaline urine precipitate information of struvite formation.

Answer is : A

173) A man who is known to be healthy presents with fever, weight loss, night sweats and
productive cough with hemoptysis for the past month, a PPD test was conducted, how long
for the PPD test to become positive after getting infected with mycobacterium ?

A. 2-3 days.
B. 3-4 months.
C. 6-8 weeks.
D. 1-2 years.
E. 5-7 days.

Tuberculin skin test (TST) or purified protein derivative (PPD) test


● 5 units (= 0.1 mL) of purified protein derivative tuberculin is injected intradermally into the
ventral surface of the lower arm. The diameter of induration at the injection site is measured
after 48–72 hours.
● The test only becomes ​positive 6–8 weeks​ after infection
● Limited specificity and sensitivity (∼ 70%)
○ False negative results due to an inadequate T-cell response in immunocompromised,
malnourished, or elderly patients
○ False positive results in patients who have received BCG vaccination or have an
infection with nontuberculous mycobacteria

A healthy individual without any risk factors for TB infection who has an induration smaller than 15
mm is considered negative for TB!

Answer is : C

174) A 55 years old man presented with productive cough and cyanosis O​2​ saturation was
89%, Reid index was 57%, Which of the following is the most likely diagnosis ?

A. Pneumonia.
B. Emphysema.
C. Right sided heart failure.
D. Bronchiectasis.
E. Chronic bronchitis.
Chronic bronchitis is a form of chronic obstructive pulmonary disease (COPD) clinically
defined as a productive cough for at least 3 months per year for at least 2 years. On
pathologic examination, the Reid index will be greater than 50%.

Answer is : E

25) 45 years old male brought to the ED after seizure half an hour ago, he is Lethargic
And Confused, Laboratory studies show: serum sodium is 115 mEq/L, he had head trauma 2
weeks ago, what is the most appropriate intervention?

A. Hypertonic normal saline.


B. Ringer lactate.
C. Normal saline.
D. Diazepam.
E. Insulin and D5W.

The diagnosis is chronic hyponatremia caused by SIADH secondary to trauma he had


earlier.

In symptomatic hyponatremia hypertonic saline should be used.

Normal saline can exacerbate hyponatremia in patients with SIADH, who may excrete the
sodium and retain the water, therefore we restrict the fluid and only give hypertonic (3%).
Total correction in the first 24 hours must not exceed 10-12 mEq as rapid rise in serum
sodium poses a risk for central pontine myelinolysis (CMP).

Answer is : A

176) A 25 y/o male developed campylobacter jejuni infection, which of the following diseases
is associated with this infection ?

A. Mitral valve stenosis.


B. Colorectal cancer.
C. Acute renal failure.
D. Guillain-barre syndrome.
E. Rheumatoid arthritis.

Campylobacter jejuni is a genus of bacteria that is among the most common causes of
bacterial infections in humans worldwide.

It has been linked with subsequent development of Guillain–Barré syndrome, which usually
develops two to three weeks after the initial illness. Individuals with recent C. jejuni infections
develop Guillain-Barré syndrome at a rate of 0.3 per 1000 infections, about 100 times more
often than the general population.
Answer is : D

177) Charcot's neurological triad is mostly associated with which of the following:

A. Alzheimer's disease.
B. Amyotrophic lateral sclerosis.
C. Multiple sclerosis.
D. Creutzfeldt–Jakob disease.
E. Stroke.

Charcot's neurological triad is the combination of nystagmus, intention tremor, and scanning
or staccato speech. This triad is associated with multiple sclerosis, where it was first
described; however, it is not considered pathognomonic for it.

Answer is : C

178) A 3 y/o kid brought to the ED with fever, headache, and neck stiffness, he is not
oriented, Best initial test is:

A. CBC.
B. MRI.
C. CT.
D. Blood cultures.
E. Lumbar puncture.

Diagnosis of Meningitis:
■ Best initial test: Obtain a lumbar puncture (LP) for CSF analysis, Gram stain, and culture
ideally before initiation of antibiotics; obtain glucose, protein, WBC count plus differential,
RBC count, and opening pressure (in the absence of papilledema or focal neurologic
deficits).
■ Viral PCRs (eg, HSV); cryptococcal antigen (for HIV patients).
■ CT or MRI is indicated in a minority of patients before LP, in particular those with altered
mental status, papilledema, or focal neurologic deficits to exclude a mass lesion or ↑
intracranial pressure (ICP). If CT is being obtained, empiric antibiotics should be started
beforehand. Obtain blood cultures. CBC may reveal leukocytosis; CSF findings vary

Answer is : E
179) One of the following is true about Blood transfusion, except:

A. AB blood group can donate to A blood group.


B. AB blood type is the universal recipient for FFP.
C. Rh compatibility testing is not conducted in emergency scenarios.
D. Packed red blood cells is used for hemophilia A.
E. FFP is given in smaller amounts than Cryoprecipitate for the same effect.

Individuals with blood type O negative are “universal donors” of packed RBCs! Individuals
with blood type AB positive are “universal recipients” for packed RBCs!

For fresh frozen plasma transfusions, individuals with blood type O are universal recipients (type O
plasma contains A and B antibodies) and individuals with blood type AB are universal donors (AB
plasma contains no A or B antibodies)!

Blood products for hemophilia A and B are : Fresh frozen plasma (FFP), cryoprecipitate and
recombinant clotting factors
● Packed RBCs Indications :
○ Generally to maintain organ perfusion and tissue oxygenation
○ Acute hemorrhage and/or hypovolemic shock (e.g., gastrointestinal bleeding, trauma)
○ Symptomatic anemia (e.g., G6PD deficiency, aplastic anemia, β-thalassemia, sickle
cell disease, anemia of chronic kidney disease)

Indications of using cryoprecipitate are similar to FFP


● preferable if large transfusion volumes are undesirable
● Cryoprecipitate taken by small amounts for the same effect.

Answer is : C

180) during the blood transfusion to 23 y/o man he developed a severe hypotension and
fatigue, the type of reaction happened in this patient is:

A. Nonhemolytic febrile transfusion reaction.


B. Anaphylaxis.
C. Acute hemolytic transfusion reaction.
D. Minor allergic reactions.
E. Delayed hemolytic transfusion reaction.

Answers B and E occur after the transfusion.


Only anaphylaxis can cause severe hypotension.

Answer is : B
181) A 47y man presented with visible congestion of paraumbilical veins, hemorrhoids and
distended abdomen, Which of the following do NOT cause portal hypertension?

A. Splenic vein thrombosis


B. Constrictive pericarditis
C. Hepatic metastases
D. Pancreatitis
E. Schistosomiasis

Pancreatitis do not cause portal hypertension directly.

Portal hypertension is present if the portal venous pressure is ≥ 6 mm Hg (normal value: 1–5 mm Hg).
Portal venous pressure > 10 mm Hg is clinically significant and > 12 mm Hg is associated with
complications. Causes of portal hypertension can be classified as follows:
● Prehepatic
○ Portal vein thrombosis
○ Splenic vein thrombosis
○ Hepatosplenic schistosomiasis
● Intrahepatic (most common)
○ Cirrhosis including fibrous proliferation
○ Massive hepatic metastases
● Posthepatic
○ Budd-Chiari syndrome
○ Right-sided heart failure
○ Constrictive pericarditis

Answer is : D

182) A 67-year-old man comes to the emergency department because of fever and
shortness of breath for the past 6 days. He has associated chills and a cough productive of
multiple thick green sputum each morning. He has a history of end-stage renal disease
requiring regular dialysis. No recent hospitalizations. Physical examination shows bibasilar
crackles. Which of the following is the most likely diagnosis?

A. Aspiration pneumonia
B. Community-acquired pneumonia
C. Healthcare-associated pneumonia
D. Hospital-acquired pneumonia
E. Ventilator-associated pneumonia

This man has a lower respiratory infection, likely community-acquired pneumonia (CAP).
The three basic pneumonia categories are community-acquired, hospital-acquired (HAP),
and ventilator-associated (VAP); the importance of establishing the category is that it
suggests the most likely pathogens and guides treatment choices. The categories are
defined in the table below:
Answer is : B

183) 65y woman presented with shortness of breath, tachycardia and cyanosis, beside PE,
The following causes central cyanosis, Except:

A. DVT.
B. Pneumonia.
C. Tetralogy of Fallot.
D. Myocardial infarction.
E. Intracranial hemorrhage.

DVT do not cause central cyanosis, unless it develops into PE.

Central cyanosis
Central cyanosis is often due to a circulatory or ventilatory problem that leads to poor blood
oxygenation in the lungs. It develops when arterial oxygen saturation drops below 85% or 75%.
Acute cyanosis can be as a result of asphyxiation or choking, and is one of the definite signs that
respiration is being blocked.

Central cyanosis may be due to the following causes:


1. Central nervous system (impairing normal ventilation):
● Intracranial hemorrhage
● Drug overdose (e.g. heroin)
● Tonic–clonic seizure (e.g. grand mal seizure)
2. Respiratory system:
● Pneumonia
● Bronchiolitis
● Bronchospasm (e.g. asthma)
● Pulmonary hypertension
● Pulmonary embolism
● Hypoventilation
● Chronic obstructive pulmonary disease, or COPD (emphysema)
3. Cardiovascular diseases:
● Congenital heart disease (e.g. Tetralogy of Fallot, right to left shunts in heart or great vessels)
● Heart failure
● Valvular heart disease
● Myocardial infarction
4. Blood:
● Methemoglobinemia * Note this causes "spurious" cyanosis, in that, since methemoglobin
appears blue, the patient can appear cyanosed even in the presence of a normal arterial
oxygen level.
● Polycythaemia
● Congenital cyanosis (HbM Boston) arises from a mutation in the α-codon which results in a
change of primary sequence, H → Y. Tyrosine stabilises the Fe(III) form (oxyhaemoglobin)
creating a permanent T-state of Hb.
5. Others:
● High altitude, cyanosis may develop in ascents to altitudes >2400 m.
● Hypothermia
● Obstructive sleep apnea

Answer is : A

184) A 49y woman known case of HIV+, complains of chronic productive cough for 4
months, wheezing, no fever, High-resolution computer tomography (HRCT) shows dilated
bronchi with signet-ring appearance,what is the most likely diagnosis ?

A. Bronchiectasis.
B. Pneumonia.
C. Emphysema.
D. Chronic bronchitis.
E. Pneumothorax.

Bronchiectasis is an irreversible and abnormal dilation in the bronchial tree that is generally caused by
cycles of bronchial inflammation in addition to mucous plugging and progressive airway destruction.
Although the widespread use of antibiotics has made bronchiectasis rare, conditions such as cystic
fibrosis (CF), severe or protracted pneumonia, ​immunodeficiency​, and COPD continue to cause it.
The most important clinical finding is a ​chronic cough ​with copious mucopurulent sputum. Other
symptoms may include dyspnea, rhinosinusitis, and hemoptysis. Physical examination reveals
crackles and rhonchi on auscultation, often accompanied by wheezing. ​High-resolution computer
tomography is the best diagnostic test and shows thickened bronchial walls, a signet-ring appearance,
and “tram track” lines.​ Treatment focuses on alleviating symptoms and preventing exacerbations, and
includes pulmonary physiotherapy and antibiotics to treat underlying infections. In rare cases, massive
hemoptysis may complicate bronchiectasis and necessitate surgery or pulmonary artery embolization.

Answer is : A

185) A 64y woman presented with difficulty of swilling solid food, Endoscopy was done
biopsies are unremarkable, barium esophagram shows narrowing of the esophagus, What is
the most common cause of her condition ?

A. Esophageal varices.
B. Esophageal cancer.
C. Achalasia.
D. GERD.
E. Laryngitis

The diagnosis is : Esophageal strictures

Esophageal strictures: most common sequela of reflux esophagitis ​(GERD)


● Clinical features: causes solid food dysphagia
● Diagnostics
○ Barium esophagram (best initial test): narrowing of the esophagus at the
gastroesophageal junction
○ Endoscopy with biopsies: to rule out malignancy and eosinophilic esophagitis
● Treatment
○ First-line treatment: dilation with bougie dilator/balloon dilator + proton pump inhibitors
in patients with reflux
○ In refractory cases (multiple recurrences): steroid injection prior to dilation,
endoscopic electrosurgical incision
● Recurrence occurs in the majority of patients; often multiple treatment attempts necessary

Answer is : D

186) 59y old male complaining of heartburn for 2 months with no other symptoms, cardiac
problems was excluded, the next step is :

A. PPI.
B. Upper endoscopy.
C. Lifestyle changes.
D. Ranitadine
E. Discharge home.

Patients with GERD and a risk factor of old age, should undergo an upper endoscopy to
exclude any metaplastic changes.

Algorithm for approaching GERD:


Answer is : B

187) A 50 years old man presents with chest pain radiates to neck, he is Hypotensive, and
pulmonary edema, his symptoms appeared 2 hours ago when he was watching TV, the most
likely diagnosis is:

A. MI.
B. Pneumonia.
C. Cardiac tamponade.
D. Pneumothorax.
E. Angina pectoris.

With its classical signs and symptoms Myocardial infarction is the most likely cause
compared to other answers.

A history of trauma can lead to the suspicion of cardiac tamponade or Pneumothorax.

Pneumonia develops over days not 2 hours.

Angina won’t cause symptoms of heart failure (Hypotension and pulmonary edema)
Answer is : A

188) A 24y female came to the Emergency Department (ED), after an episode of panic
attack, what is the likely acid-base disturbance to happen?

A. Metabolic acidosis.
B. Respiratory acidosis.
C. Metabolic alkalosis.
D. Respiratory alkalosis.
E. Respiratory and metabolic acidosis.

A common acid-base imbalance in panic attacks is caused by hyperventilation syndrome, in


which tachypnea leads to expel the acidic molecule of CO2 making the blood more Alkalotic,
which it can cause a respiratory alkalosis.

Answer is : D

189) Which of the following Renin-angiotensin-aldosterone system (RAAS) has the most
vasoconstrictive effect:

A. Renin.
B. Angiotensin I.
C. Angiotensin II.
D. Aldosterone.
E. ACE inhibitor.

ACE inhibitors are a class of drugs, not a component of RAAS.

The renin-angiotensin-aldosterone system (RAAS)


Drops in blood pressure reduce renal perfusion.

If the pressure in the renal artery falls by more than 10–15 mm Hg → proteolytic renin is released
from the juxtaglomerular apparatus → renin converts angiotensinogen to angiotensin I → ACE
cleaves C-terminal peptides on angiotensin I, converting it to angiotensin II → increases the blood
pressure in two ways:
1. Vasoconstriction
2. stimulation of the release of aldosterone, which increases the retention of water and sodium

Angiotensin I appears to have no direct biological activity and exists solely as a precursor to
angiotensin II.

Answer is : C
190) A man recently diagnosed with primary hypertension, he also have asthma, Which of
the following beta blockers causes Bronchoconstriction ?

A. Propranolol.
B. Esmolol.
C. Metoprolol.
D. Bisoprolol.
E. Atenolol.

Answer is : A

191) A 70y woman diabetic for 15 years, presented with edema, The best screening test for
diabetic nephropathy is:

A. Renal biopsy.
B. Urine Protein level.
C. HbA1c.
D. Urine glucose Level.
E. Fasting Blood Glucose.

Screening for proteinuria in patients with type 2 diabetes is recommended at initial diagnosis, then at
least once a year. Screening in type 1 diabetic patients is also recommended yearly, beginning 5
years after disease onset. Diabetic nephropathy usually develops after 5 years with diabetes mellitus;
however, type 2 diabetic patients may present with renal damage at diagnosis. Many type 2 diabetic
patients have additional risk factors for kidney disease (e.g., hypertension).

Answer is : B

192) Cystic fibrosis is responsible for increased viscosity of all the following secretions,
except:

A. Bile.
B. Pancreatic juices
C. Saliva.
D. Sweat.
E. Pulmonary mucus

In all exocrine glands, the Cl- channel is responsible for transporting intracellular Cl- across
the cell membrane. However, in sweat glands, the Cl- channel is responsible for transporting
Cl- from the lumen into the cell. The sweat test relies on the inability of the sweat glands to
reabsorb salt, which results in elevated NaCl levels in sweat.

Answer is : D

193) A 26y man is being prepared a surgery, the surgeon asked if he has any bleeding
disorder, he answered yes, then he adds his father, two uncles from father side and
grandfather had the same condition, what is the most likely diagnosis ?

A. Bernard–Soulier syndrome.
B. Vitamin K deficiency.
C. Von Willebrand disease.
D. Hyperfibrinolysis.
E. Hemophilia A.

● The pattern of inheritance this patient shows is mostly X-linked recessive (it affects
only males)
● Hemophilia A have X-linked recessive inheritance fashion.
● Vitamin K deficiency is mostly acquired.
● Von Willebrand disease Types 1 and 2 are inherited as autosomal dominant traits.
Occasionally, type 2 also inherits recessively. Type 3 is inherited as an autosomal
recessive.
● Bernard–Soulier syndrome is inherited by an autosomal recessive fashion.
● Hyperfibrinolysis is mostly acquired.

Answer is : E
194) A 35 jordanian soldier in a military mission in haiti, presented with fever, dyspnea, rash
all over his body and dry cough, he said there are 5 soldiers who sleeps in the same
barracks have the same symptoms, chest x-ray suggest Interstitial pneumonia, what is the
most likely causative organ ?

A. Staphylococcus aureus.
B. Pseudomonas aeruginosa.
C. Streptococcus pneumoniae.
D. Haemophilus influenzae.
E. Mycoplasma.

His symptoms and chest x-ray confirms Atypical pneumonia, the most common cause of
Atypical pneumonia is Mycoplasma, Plus; mycoplasma can outbreak in crowded places such
as barracks.

Mycoplasma pneumonia
● Epidemiology
○ One of the most common causes of atypical pneumonia
○ More common in young individuals
○ Outbreaks may occur in schools, colleges, prisons, and ​military facilities
● Clinical features
○ Low-grade fever
○ Non-productive, dry cough
○ Dyspnea
○ Common extrapulmonary features include fatigue, headaches, sore throat,
myalgias, malaise
○ Auscultation often unremarkable
○ Generalized papular rash​, erythema multiforme
● Diagnostics
○ Subclinical hemolytic anemia: associated with elevated cold agglutinin titers
(IgM)
○ Interstitial pneumonia, often with reticulonodular pattern on CXR
● Treatment: macrolides, doxycycline, and fluoroquinolones

Answer is : E

195) A 59y woman presented with bronze skin, she is known case of adrenal insufficiency,
the skin hyperpigmentation is caused by which hormone?

A. Cortisol.
B. Aldosterone.
C. ACTH.
D. DHEA-S.
E. Angiotensin II.

hypoandrogenism (most common in female patients) , hypoaldosteronism, and hypocortisolism → ↑


ACTH
● ACTH is derived from precursor molecule pro-opiomelanocortin (POMC), which is also a
precursor for melanocyte-stimulating hormone (MSH).
● ↑ Production of POMC (in order to ↑ ACTH production) → ↑ melanocyte-stimulating hormone
(MSH) and hyperpigmentation of the skin (bronze skin)

Answer is : C

196) What is the APGAR score for a neonate, heart rate 120, pink body and hands with
cyanotic feet, weak cry, flexion of the arms and legs, active movement and crying when
stimulated ?

A. 10.
B. 9.
C. 8.
D. 5.
E. 3.

This neonate score is :


Activity = active movements = 2 points
Pulse = >100 = 2 points
Grimence = weak crying when stimulated = 2 points
Appearance = cyanotic extremities, pink body = 1 point
Respiration = slow and irregular = 1 point

Total = 8
Answer is :

197) A neonate is being assessed by the physician he noted:


● Length: 49 cm
● Weight: 3.1 kg
● Head circumference: 42 cm
● RR: 44 Breaths per minute
● HR: 130 bpm
Which of the following is the abnormal value ?

A. Length
B. Weight
C. Head circumference
D. RR
E. HR

Normal neonatal findings:


● Measurements
○ Normal range (10th to 90th percentile at 40 weeks gestation)
■ Length: ∼ 50 cm (48– 53 cm)
■ Weight: (2.9–3.9 kg)
■ Head circumference: ∼ 35 cm (33–37 cm)
● Vital signs
○ Respiratory rate: 40–60 breaths per minute
○ Heart rate: 120–160 beats per minute
● pH: ≥ 7.2 (slightly more acidic than adults)
● Urine and meconium
○ First passage of urine within 24 hours of birth
○ First passage of meconium within 48 hours after birth

Answer is :
198) All the following are external signs of maturity in neonates, except :

A. Defined areolas.
B. Rose skin.
C. Descended testicles.
D. Drooling.
E. Presence of lanugo

External signs of neonate maturity


● Skin color and texture: rosy
● Body hair: lanugo may be present, thinning, or mostly absent
● Eyes: open
● Ears: well formed pinna (auricular cartilage) that instantly recoils
● Breast: clearly discernible areola
● Testicles: descended
● Labia: labia minora covered by labia majora
● Plantar creases: cover the entire soles of the feet

lanugo is fine, soft hair, especially that which covers the body and limbs of a human fetus.
Answer is :

199) All the following are true about neonatal polycythemia, except:

A. it’s diagnosed when Venous HCT > 55%.


B. The majority of infants are asymptomatic
C. Small-for-gestational-age is a risk factor.
D. Large-for-gestational-age is a risk factor.
E. Can be treated with partial exchange transfusion.

Neonatal polycythemia
● Definition: venous hematocrit (HCT) greatly exceeding normal values for gestational and
postnatal age.
● Pathophysiology
○ Delayed umbilical cord clamping → erythrocyte transfusion → ↑ circulating red blood
mass (HCT)
○ Placental insufficiency or chronic intrauterine hypoxia → increased intrauterine
erythropoiesis → ↑ circulating red blood mass (HCT)
● Risk factors
○ Small-for-gestational-age
○ Large-for-gestational-age
○ Infants of diabetic mothers
○ Maternal tobacco use
○ Delayed umbilical cord clamping
○ Infants who is born at higher elevations
● Clinical features
○ The majority of infants are asymptomatic.
○ Respiratory distress, cyanosis, apnea
○ Poor feeding, vomiting
○ Hypoglycemia
○ Ruddiness (plethora)
○ Lethargy and irritability
○ Tremors or seizures
● Diagnosis : Venous HCT > 65%
● Treatment (if symptomatic)
○ Monitoring
○ IV hydration
○ Possible partial exchange transfusion (PET)
● Complications
○ Hypoglycemia
○ Hyperbilirubinemia
○ Necrotizing enterocolitis

Partial exchange transfusion: ‫ﻫﻲ ﺗﺒﺪﯾﻞ ﺑﻌﺾ ﻣﻦ ﻛﻤﯿﺔ اﻟﺪم ﺑﺎﻟﻨﻮرﻣﺎل ﺳﺎﻻﯾﻦ‬

Answer is :
200) On assessment of a neonate you find dark well defined 4 cm diameter lesion on her
back with increased hair growth inside the lesion, what is the most likely diagnosis ?

A. Milia neonatorum.
B. Melanoma.
C. Congenital melanocytic nevi.
D. Infantile hemangioma
E. Capillary malformations.

Congenital melanocytic nevi


● Epidemiology: 1/20,000 births
● Clinical features
○ Vary in size: < 1.5 cm to > 40 cm; > 40 cm in size = giant congenital melanocytic
nevus).
○ Light to darkly pigmented lesion
○ Often with increased hair growth
● Treatment: surgical excision or laser treatment (depending on the type and size of lesion)
● Prognosis: Large nevi are at risk of degeneration → frequent follow-up

Answer is :

201) A 2 years child presents with red non tender raised macule which has a strawberry
appearance in her head, which of the following is most likely to be the lesion:
A. Milia neonatorum.
B. Melanoma.
C. Congenital melanocytic nevi.
D. Infantile hemangioma.
E. Capillary malformations.

Infantile hemangioma (strawberry hemangioma)


● Definition: benign vascular tumors of infancy
● Epidemiology
○ Occurs in up to 10% of infants
○ Mostly affects girls
● Pathophysiology
○ Abnormal development of vascular endothelial cells
○ Rapid proliferation followed by subsequent spontaneous slow involution
● Clinical features
○ Manifests during the first few days to months of life
○ Progressive presentation: blanching of skin → fine telangiectasias → red
painless papule or macule (“strawberry appearance”)
○ Most commonly on head and neck
○ Usually solitary lesions
● Diagnosis
○ Based on clinical findings
○ The differential diagnosis of cherry angioma is found mostly in adults.
● Treatment
○ Active non-intervention (monitoring, parental education)
○ Systemic therapy with propranolol in complicated cases:
■ Rapidly growing cutaneous hemangiomas
■ Periorbital hemangioma: vascular anomaly in the periorbital region;
most commonly the upper eyelid
■ Hemangiomas in the airways, gastrointestinal tract, or liver
■ Hemangiomas with high risk of complications (e.g., ulceration,
disfigurement)
○ If unresponsive to medication:
■ Cryotherapy
■ Laser therapy
■ Resection if necessary
● Prognosis
○ Usually good prognosis
○ Spontaneous resolution is common (70% by age 7)
○ Visual impairment in untreated periorbital hemangioma

Answer is : D

202) which of the following is NOT a complication of Infantile hemangioma ?

A. Sight problems.
B. Ulceration.
C. Breathing difficulties.
D. Scarring.
E. Neoplastic changes.

Neoplastic changes do not happen in Infantile hemangioma.

Complications:
● Large facial strawberry naevi may cause problems with sight, hearing, breathing,
swallowing and speaking. They should be treated immediately to prevent further
problems.
● Bleeding, ulceration or scars may occur.
● Visceral haemangiomas may produce symptoms due to bleeding or mass effect. The
symptoms will vary according to site.
● Hepatic haemangiomas are associated with high-output cardiac failure that can be
life threatening.
● Occasionally, they may impinge on vital structures, ulcerate, bleed, and induce
consumptive coagulopathy or significant structural abnormalities. Those on the face
are of greatest concern.

Answer is : E

203) All the following are contributory factors in the physiological jaundice mechanism,
except:

A. Immature liver.
B. Shorter RBC lifespan.
C. Rise in erythropoietin levels.
D. Low conversion of bilirubin to urobilinogen.
E. Low activity of the enzyme glucuronosyltransferase.
Erythropoietin levels drops in the first days of life due to extrauterine oxygen rich
environment.

Physiological neonatal jaundice:


● Always unconjugated hyperbilirubinemia
○ Peak total serum bilirubin : < 15 mg/dL (in the case of a full-term, breastfed infant)
○ Daily rise in bilirubin levels < 5 mg/dL/day

Mechanism involved in physiological jaundice are mainly:


● The immature liver has a relatively low activity of the enzyme glucuronosyltransferase which
normally converts unconjugated bilirubin to conjugated bilirubin that can be excreted into the
gastrointestinal tract. Before birth, this enzyme is actively down-regulated, since bilirubin
needs to remain unconjugated in order to cross the placenta to avoid being accumulated in
the fetus. After birth, it takes some time for this enzyme to gain function.
● Shorter life span of fetal red blood cells, being approximately 80 to 90 days in a full term
infant, compared to 100 to 120 days in adults.
● Relatively low conversion of bilirubin to urobilinogen by the intestinal flora, resulting in
relatively high absorption of bilirubin back into the circulation.

Answer is : C

204) All the following are etiologies of neonatal jaundice, except :

A. Rhesus incompatibility.
B. Crigler-Najjar syndrome.
C. Biliary atresia.
D. TOF.
E. Vitamin K deficiency.

Tetralogy of fallot do not cause neonatal jaundice. It causes cyanosis.

Etiology of conjugated hyperbilirubinemia:


Decreased bilirubin excretion
○ Intrahepatic cholestasis
■ Sepsis
■ Hepatitis A
■ Hepatitis B
■ TORCH infections
■ Cystic fibrosis
○ Extrahepatic cholestasis
■ Biliary atresia
■ Choledochal cyst
■ Tumors/strictures
● Intrahepatic pathologies
○ Infectious hepatitis
○ Metabolic diseases
■ Dubin-Johnson syndrome
■ Rotor syndrome
■ Galactosemia
■ Alpha-1-Antitrypsin deficiency
■ Hypermethioninemia
○ Idiopathic neonatal hepatitis
○ Alagille syndrome

Etiology of unconjugated hyperbilirubinemia:


● Hemolytic
○ Infection or sepsis (infections in the newborn)
○ Hematomas (e.g., from vacuum-assisted delivery, vitamin K deficiency bleeding)
○ Hemolytic disease of the newborn (ABO incompatibility , Rhesus incompatibility)
○ RBC structural defects (spherocytosis, hereditary elliptocytosis)
○ Ineffective erythropoiesis (thalassemias)
○ Glucose-6-phosphate dehydrogenase deficiency
● Nonhemolytic
○ Hyperbilirubinemia syndromes (e.g., Crigler-Najjar syndrome, Gilbert's syndrome)
○ Glucuronyl transferase deficiency
○ Increased enterohepatic circulation: e.g., high GI obstruction (pyloric stenosis, bowel
obstruction)
○ Other causes: medication, hypothyroidism, malnutrition, polycythemia

Answer is : D

205) A 24h neonate presented with jaundice, Lethargy, hypotonia, what is the most likely
cause for his jaundice

A. Physiological jaundice.
B. Rh hemolytic disease.
C. Hepatitis A
D. Cystic fibrosis
E. Biliary atresia

● Physiological jaundice does not manifest until 3rd day of life


● The cause is pathological, and the most common pathological neonatal jaundice in
the first 24h is Rh hemolytic disease.

Answer is : B
206) The obstetrician in the “king hussein” hospital is evaluating a neonate, he found: heart
rate 101, cyanotic body and extremities, no response to stimulation, no flexion of extremities,
and strong cry, What is his APGAR score ?

A. 4
B. 6
C. 5
D. 7
E. 3

This neonate score is :


Activity = no flexion of extremities = 0 points
Pulse = >100 = 2 points
Grimence = no response to stimulation = 0 points
Appearance = cyanotic body and extremities = 0 point
Respiration = strong cry = 2 point

Total = 4

Answer is : A

207) An 8y boy presented with pain in his knees, severe colicky abdominal pain the doctor
noticed a red rash over his thigh and buttocks, after a throat infection with group A
streptococcus, What other manifestations are expected to present with this boy’s condition?

A. Hematuria.
B. Endocarditis.
C. Encephalitis.
D. Asthma.
E. Steatorrhea.

This boy’s symptoms goes with Henoch-Schonlein purpura, and 50% of affected shows
signs of nephritic syndrome such as hematuria.

Clinical features of HSP


History
● Symptom onset often 1–3 weeks after an infection, typically affecting the upper respiratory
tract

Manifestations
● Skin (∼ 100% of cases)
○ Symmetrically distributed, raised, erythematous macules or urticarial lesions that
coalesce into palpable purpura (non-blanching skin lesions)
○ Most common sites: the lower extremities, buttocks, and other areas of pressure or
constraint (e.g., from socks or clothing)
● Joints (∼ 75% of cases): arthritis/arthralgia, most common in the ankles and knees
● Gastrointestinal tract (∼ 60% of cases)
○ Colicky abdominal pain (may be severe enough to mimic an acute abdomen)
○ Bloody stools or melena
○ Nausea/vomiting
● Kidneys (∼ 50% of cases): HSP nephritis with signs and symptoms of nephritic syndrome
(hematuria)
● Other organs
○ Scrotum (e.g., scrotal swelling, pain, and tenderness)
○ Central and peripheral nervous system (e.g., headaches, seizures, focal neurologic
deficits, ataxia, intracerebral hemorrhage, central and peripheral neuropathy)
○ Respiratory tract (e.g., mild interstitial changes, pulmonary hemorrhage)

Answer is : A

208) A 3y girl brought to the ED with features of dehydration after many episodes of
diarrhea, she lost 7% of his body weight, her weight was 14kg, What is the amount of fluid
must be replaced ?

A. 980ml.
B. 660ml.
C. 1300ml.
D. 1150ml.
E. 440ml.
The required fluid replacement is the sum total of deficit, ongoing losses and maintenance
requirements:
● Correction of deficit: deficit in ml = wt (kg) x % dehydrated x 10 (ideally the pre-dehydration
weight should be used). Therefore, a 14 kg child who is 7% dehydrated has a deficit of 14 x 7
x 10 = 980 ml.
● Ongoing losses: calculated from fluid from nasogastric tubes, drains, urine; also need to
consider additional fluid loss in certain situations - eg, pyrexia, tachypnoea.
● Maintenance requirements:
○ 100 ml/kg for the first 10 kg.
○ 50 ml/kg for the next 10 kg.
○ 20 ml/kg for any weight after 20 kg.

Answer is : A

209) A 14y boy presented with swollen neck on the left side, tender testicles, red eyes, his
mother says he had fever 2 days ago but resolved on its own, what is the most likely
diagnosis ?

A. Coxsackievirus.
B. Common cold.
C. flu.
D. Mumps.
E. Scarlet fever.

Swollen neck on the left side, tender testicles, red eyes and history of recent fever suggest
mumps virus infection.

Answer is : D

210) Which of the following is contraindicated in enteral feeding?

A. Liver failure.
B. Lower GI bleeding.
C. Cholecystitis.
D. Bowel obstruction.
E. Neck trauma.

Enteral nutrition is the administration of nutrients directly into the stomach, duodenum, or jejunum with
the help of feeding tubes

Contraindications :
● Mechanical ileus, bowel obstruction
● Acute abdomen (e.g., severe pancreatitis, peritonitis)
● Upper GI bleeding
● Mucositis
● Severe substrate malabsorption
● Congenital GI anomalies
● High-output fistulas
● Nonfunctional GI tract (e.g., gastroschisis, short bowel syndromes)

Answer is : D

211) A 2 years old child presented with bloating, he was crying for hours, his mother states
he didn’t defecate in 2 days, what is the most likely cause?

A. Cystic fibrosis.
B. Hirschsprung disease.
C. Weaning.
D. Volvulus.
E. Intussusception.

Main explanation:

The most common cause of constipation in children is functional (95%), in this age weaning
is the most likely cause for constipation.

Further reading:
Constipation in children is divided into 2 main types:
● Organic (5%)
● Functional (95%)

Organic
Organic causes of constipation involve specific structural, neurologic, toxic/metabolic, or
intestinal disorders. They are rare but important to recognize
The most common organic cause is
● Hirschsprung disease
Other organic causes that may manifest in the neonatal period or later include
● Anorectal malformations
● Cystic fibrosis
● Metabolic disorders (eg, hypothyroidism, hypercalcemia, hyperkalemia)
● Spinal cord abnormalities

Functional
Functional constipation is difficulty passing stools for reasons other than organic causes.
Children are prone to develop functional constipation during 3 periods:
● After the introduction of cereals and solid food
● During toilet training
● During the start of school

Answer is : C

212) A 2 years old child presented with diarrhea, fever, abdominal pain, What is the most
common cause of diarrhea in this age ?

A. Giardia lamblia.
B. Norovirus.
C. Entamoeba histolytica.
D. Echoviruses.
E. Rotavirus.

Rotavirus is a genus of double-stranded RNA viruses in the family Reoviridae. Rotaviruses


are the most common cause of diarrhoeal disease among infants and young children, Nearly
every child in the world is infected with a rotavirus at least once by the age of five.

Answer is : E

213) A 6y boy developed jaundice, Petechiae, and oliguria, urinalysis shows hematuria. His
mother said he had an episodes of diarrhea 10 days ago, what is the cause of diarrhea ?

A. Shigella.
B. Enterohemorrhagic e.coli.
C. Vibrio cholerae.
D. Salmonella.
E. Yersinia.

This boy’s condition goes with hemolytic urimec syndrome (HUS), the most common cause
is enterohemorrhagic e.coli

Etiology of HUS
● Bacterial exotoxins
○ Shiga-like toxin (verotoxin) from enterohemorrhagic E. coli (EHEC) strain O157:H7
■ Usually transmitted via contaminated foods, e.g., undercooked beef or raw
leafy vegetables
○ Shiga toxin produced by Shigella dysenteriae
● Streptococcus pneumoniae infection

Approx. 15% of children infected with E. coli O157:H7 will go on to develop HUS!

Answer is : B
214) A 3 week baby brought to the pediatrician because she looks cyanotic, and become
more noticeable when crying, what is the most common cause of this baby’s condition ?

A. Endocarditis.
B. Patent ductus arteriosus
C. Tetralogy of fallot.
D. Atrial septal defect
E. Transposition of the great vessels

Tetralogy of Fallot is a congenital heart defect, It is the most common cause of cyanosis
within the rst few weeks of life. The skin becomes bluish because of the malformed
right-to-left shunt. Infants also have worsening cyanosis with agitation, difculty feeding, and
failure to gain weight.

Patent ductus arteriosus typically is not clinically apparent until adulthood.

The signs and symptoms is insufficient for the diagnosis of endocarditis.

Atrial septal defect typically is not clinically apparent until adulthood.

Transposition of the great vessels is not more common than TOF.

Answer is : C

215) Which of the following Vitamins should be supplied to infants who feed exclusively in
breast milk ?

A. Vitamin D.
B. Vitamin B12.
C. Vitamin B9.
D. Vitamin K.
E. Vitamin C.

Vitamin D supplementation
● Exclusively breast-fed infants should receive vitamin D supplementation daily
● Vitamin D supplementation can be stopped once the infant is started on vitamin D fortified
cow's milk (usually after 1 year of age)
● Infants on vitamin-D fortified formula feeds do not require further supplementation

Answer is : A
216) The Moro reflex disappears at what age ?

A. 3-6 Months.
B. 4-6 Weeks.
C. 7-9 Months.
D. 1-2 Years.
E. 3-6 Years.

The Moro reflex is an infantile reflex that develops between 25–30 weeks of gestation and
disappears between 3–6 months of age. It is a response to a sudden loss of support and
involves three distinct components:
1. spreading out the arms (abduction)
2. pulling the arms in (adduction)
3. crying (usually)

Answer is : A

217) A 15y male brought to the emergency department unconscious and was breathing and
hard his father states that his medical history is unremarkable, but he had severe abdominal
pain abdominal pain a few hours ago, lab test shows:
Arterial pH: 6.9 (Normal = 7.35-7.45)
Urine glucose and ketone: positive (Normal = negative)
Serum bicarbonate 11 mEq/L (Normal = 23-30 mEq/L)
What is the diagnosis?

A. Hypoglycemia.
B. Renal tubular acidosis.
C. Gastroenteritis.
D. Diabetic ketoacidosis.
E. Hyperventilation syndrome.

Signs and symptoms of both DKA and HHS


● Polyuria
● Polydipsia
● Recent weight loss
● Nausea and vomiting
● Signs of volume depletion (i.e., dry mucous membranes, decreased skin turgor), hypotension,
circulatory collapse
● Neurological abnormalities
○ Altered mental status
○ Lethargy
○ Coma
○ Other neurological exam abnormalities such as blurred vision and weakness
Signs and symptoms specific to DKA
● Rapid onset (< 24 h) in contrast to HHS
● Abdominal pain
● Fruity odor on the breath (from exhaled acetone)
● Hyperventilation: Kussmaul respirations: deep breaths at a normal respiratory rate

Answer is : D

218) All the following are causes of short stature, except:

A. Glycogen storage diseases


B. Achondroplasia
C. Prematurity
D. Hypothyroidism
E. Klinefelter syndrome.

People with Klinefelter syndrome are taller than average.

Etiology of short stature:

Primary growth disorders


● Clinically defined genetic syndromes, such as:
○ Down's syndrome
○ Prader-Willi syndrome
○ Silver- Russell syndrome
○ Noonan's syndrome
○ Turner syndrome
● Intrauterine growth restriction with failure to catch up:
○ Fetomaternal factors
○ Prematurity
○ Placental dysfunction
● Congenital bone disorders such as:
○ Achondroplasia
○ Hypochondroplasia
○ Osteogenesis imperfecta

Secondary growth disorders


● Endocrine:
○ Hypothyroidism.
○ Panhypopituitarism.
○ Hypothalamic or pituitary lesions (eg, trauma or tumour).
○ Laron's syndrome (growth hormone insensitivity).
○ Cushing's syndrome.
○ Growth hormone deficiency or insufficiency.
○ Precocious puberty.
○ Disorders of the growth hormone insulin-like growth factor I axis.
● Metabolic:
○ Mucopolysaccharidoses.
○ Glycogen storage diseases.
● Diabetes mellitus (poor control).
● Chronic disease:
○ Cardiovascular disease.
○ Respiratory disease (eg, cystic fibrosis).
○ Haemoglobinopathies.
○ Renal disorders disease.
○ Malignancy.
○ Neurological (eg, hydrocephalus).
○ Juvenile arthritis.
● Malnutrition:
○ Poverty or neglect.
○ Inflammatory bowel disease.
○ Coeliac disease.
○ Bowel obstruction.
○ Enzyme deficiencies.
○ Chronic bowel infection.
○ Short bowel syndrome.
○ Anorexia nervosa.
○ Rickets.
● Psychosocial deprivation, including hyperphagic short stature syndrome.
● Medication: steroid therapy.

Answer is : E

219) Which of the following is a minor criteria in “jones criteria” for rheumatic fever ?

A. Carditis
B. Erythema marginatum
C. Chorea
D. High ESR.
E. Subcutaneous nodules

Major criteria
● Polyarthritis: A temporary migrating inflammation of the large joints, usually starting in
the legs and migrating upwards.
● Carditis: Inflammation of the heart muscle (myocarditis) which can manifest as
congestive heart failure with shortness of breath, pericarditis with a rub, or a new
heart murmur.
● Subcutaneous nodules: Painless, firm collections of collagen fibers over bones or
tendons. They commonly appear on the back of the wrist, the outside elbow, and the
front of the knees.
● Erythema marginatum: A long-lasting reddish rash that begins on the trunk or arms
as macules, which spread outward and clear in the middle to form rings, which
continue to spread and coalesce with other rings, ultimately taking on a snake-like
appearance. This rash typically spares the face and is made worse with heat.
● Sydenham's chorea (St. Vitus' dance): A characteristic series of involuntary rapid
movements of the face and arms. This can occur very late in the disease for at least
three months from onset of infection.

Minor criteria
● Fever of 38.2–38.9 °C (100.8–102.0 °F)
● Arthralgia: Joint pain without swelling (Cannot be included if polyarthritis is present as
a major symptom)
● Raised erythrocyte sedimentation rate or C reactive protein
● Leukocytosis
● ECG showing features of heart block, such as a prolonged PR interval (Cannot be
included if carditis is present as a major symptom)
● Previous episode of rheumatic fever or inactive heart disease

Answer is : D

220) All the following statements are true about vaccination, except:

A. Some vaccines can cause disease.


B. Measles vaccine is live attenuated.
C. Mumps is killed vaccine.
D. Live vaccinations are not recommended for pregnant.
E. Hepatitis B shouldn't be given to infant weighing less than 2 kg.

Mumps is live attenuated vaccine.

Live attenuated vaccines can theoretically revert back to their disease-causing form.
However, this has only been observed in the case of the oral polio vaccine.

Answer is : C

221) Which of the following general anesthetics causes “Dissociative anesthesia” ?

A. Etomidate
B. Propofol
C. Thiopental
D. Ketamine
E. Fentanyl
Dissociative anesthesia:
A form of anesthesia, characterized by the uncoupling of sensory, motor, memory, and
emotional brain activity that results in catalepsy, amnesia, and analgesia. Caused by certain
anesthetic drugs (e.g., ketamine).

Ketamine:
A rapidly acting dissociative anesthetic that is typically used to sedate patients prior to rapid
sequence intubation or as an emergency anesthetic for polytrauma patients with risk of
hypotension. Antagonizes NMDA receptors and increases blood pressure, cerebral blood
flow, temperature, and pain tolerance. Side effects include confusion, hallucinations, ataxia,
and nightmares.

Answer is : D

222) 25 years old man brought to the hospital after falling from the second floor, he was
conscious, vomited 2 times, he was confused, when asked about his name he answered, his
eyes were closed and opened when asked to, What is his Glasgow coma scale?

A. 13
B. 14
C. 15
D. 12
E. 11

Confused, put appropriate words = 4


Open eyes to verbal commands = 3
Obeys command = 6

Glasgow Coma Scale (GCS) is a tool used for evaluation of consciousness

● Assessment of neurological status and trauma severity in patients with traumatic


brain injury (mostly used in acute cases)
● Maximum score 15 points (full consciousness); minimum score 3 points (coma or
death)

Points Eye opening Verbal response Motor response


6 Obeys commands

5 Appropriate words and oriented Localizes pain stimulus

4 Spontaneously Appropriate words but confused Withdraws from pain

3 To verbal command Inappropriate words Decorticate posture

2 To pain Incomprehensible sounds Decerebrate posture

1 No response No response No response

Mild head injury: GCS score 13–15

Moderate head injury: GCS score 9–12

Severe head injury: GCS score ≤ 8 (Indication for endotracheal intubation)

Decorticate posture = Rigid posture with flexed arms and straight legs

Decerebrate posture = Rigid posture with extended (and often rotated) arms and legs

Answer is : A

223) All the following are a risk factor for pancreatic cancer, except:

A. Chronic pancreatitis.
B. Small bowel obstruction.
C. Alcohol consumption.
D. Having BRCA1 and BRCA2 mutations.
E. Obesity.
Risk factors of pancreatic cancer:
● Smoking
● Chronic pancreatitis
● High alcohol consumption
● Type 2 diabetes
● Obesity
● Occupational exposure to chemicals used in the dry cleaning and metal working
industries
● Cirrhosis of the liver
● H. pylori infection → excess stomach acid
● Inherited genetic syndromes (10% of pancreatic cancers)
○ Familial pancreatic carcinoma
○ Familial pancreatitis (mutations in the PRSS1 gene)
○ Peutz-Jeghers syndrome
○ Familial atypical multiple mole melanoma (FAMMM) syndrome
○ Hereditary breast and ovarian cancer syndrome (BRCA1 and BRCA2
mutations)
○ HNPCC
○ Von-Hippel-Lindau syndrome
○ Neurofibromatosis type 1
○ Multiple endocrine neoplasia type 1

Answer is : B

224) All the following are clinical features of inguinal hernia, except:

A. Insomnia.
B. Bloating.
C. Hydrocele.
D. Vomiting.
E. Abdominal pain.

Clinical features
● Visible, palpable groin protrusion or bulge
● Inguinal pain (does not have to correlate with the size of the hernia)
● Increase of symptoms during physical activity (walking or standing, coughing,
sneezing, abdominal pressure)
● Indirect inguinal hernia may be associated with a communicating ​hydrocele
● Palpation of the inguinal canal
1. With the patient standing, palpate from the scrotal skin towards the superficial
(external) inguinal ring.
2. Ask the patient to cough or strain and bear down (Valsalva maneuver).
■ Bulging can be felt with a fingertip (expansile cough impulse).
● May present with symptoms of bowel obstruction: ​abdominal pain​, vomiting, ​bloating
and not passing gas.
● May present with symptoms of intestinal necrosis: pain and erythema in the lower
abdomen and scrotum.

Answer is : A

225) The stomach receives blood supply from which of the following ?

A. Celiac Artery.
B. Cystic artery.
C. Superior mesenteric artery.
D. Portal vein.
E. Renal arteries.

The blood supply of the stomach arises from the celiac artery (trunk).
● From the anterior surface of the aorta, at the level of T12-L1
● Travels along the superior border of the pancreas

Answer is : A
226) All the following are treatments for hemorrhoids, except:

A. Rubber band ligation


B. Stool softeners
C. Topical lidocaine
D. Sitz baths
E. Low fiber diet.

Low Fiber diet worsen the condition as it causes less stool elasticity.

Hemorrhoids, also called piles, arise from a cushion of dilated arteriovenous blood vessels
and connective tissue in the anal canal that may abnormally enlarge or protrude.
Hemorrhoids are divided into three categories: internal (above the dentate line), external
(below the dentate line), or mixed (above and below the dentate line). Hemorrhoids are
caused by increased straining or intra-abdominal pressure (e.g., due to constipation,
pregnancy, or extended periods of sitting). Patients may present with prolapse, rectal
bleeding, pain, and pruritus.

Treatment:

Hemorrhoids should only be treated in a symptomatic patient.

Conservative treatment:
● Indications: grade I–II internal hemorrhoids and external hemorrhoids
● Interventions :
○ Lifestyle modifications: weight loss, exercise, high fiber diet, avoid fatty and
spicy foods, increase water intake
○ Alter stool habits (e.g., avoid excessive straining or > 5 min periods on the
toilet)
○ Sitz baths
○ Stool softeners (e.g., docusate)
○ Topical or suppository analgesia (e.g., lidocaine)
○ Topical anti‑inflammatory (e.g., hydrocortisone, especially with pruritus, but
no longer than 1 week)
○ Topical antispasmodic agents (e.g., nitroglycerin)

Outpatient treatment :
● Indications: all internal hemorrhoids with symptoms persisting despite conservative
treatment and grade III internal hemorrhoids
● Interventions
○ Rubber band ligation (RBL)
○ Sclerotherapy
○ Infrared coagulation

Surgical treatment (stages III–IV):


● Indications: grade IV internal hemorrhoids and no improvement of condition after
clinical interventions
● Interventions
○ Arterial ligation of hemorrhoids (HAL)
○ Submucosal hemorrhoidectomy
○ Stapled hemorrhoidopexy (e.g., using the Longo procedure): only effective for
internal hemorrhoids

Answer is : E

227) All the following can cause fever immediately postoperative, except:

A. Blood transfusion reaction


B. Malignant hyperthermia
C. DVT
D. Neuroleptic malignant syndrome
E. Thyrotoxic crisis

Causes of postoperative fever:

Immediate:
● Infection acquired prior to surgery
● Febrile nonhemolytic transfusion reaction
● Trauma prior to surgery
● Malignant hyperthermia
● Thyrotoxic crisis
● Anticholinergic syndrome
● Neuroleptic malignant syndrome

Days 1-5:
● Nosocomial urinary tract infection due to an indwelling catheter
● Pneumonia
● Surgical site infection due to group A streptococci (GAS) or C. perfringens
● Catheter-related bloodstream infection
● Atelectasis
● Trauma prior to surgery
● Pancreatitis
● Acalculous cholecystitis
● Myocardial infarction
● DVT ​and pulmonary embolism
● Thrombophlebitis
● Alcohol withdrawal
● Acute gout
● Anastomotic leak

Answer is : C
228) A 55 y/o man presented to the emergency department with redness and swelling with
blisters in his right arm after opening an oven while hot 5 hours ago, he feels pins and
needles in his fingers, his hand also feels cold, the most likely diagnosis is:

A. Rhabdomyolysis
B. Compartment syndrome
C. Cellulitis
D. Colles fracture
E. Needle injury

The patient likely have a compartment syndrome secondary to circumferential burn on his
arm.

Blistered skin indicates 2nd degree burn.

Acute compartment syndrome


Typically presents with a rapid progression of symptoms.

Early presentation :
● Pain
○ Often out of proportion to the extent of injury
○ Worse with passive stretching or extension of muscles
○ Very tight, “wood-like” muscles that are extremely tender to touch
● Paresthesia (e.g., pins and needles)
● Soft tissue swelling
● Initially, peripheral circulation and distal pulses are maintained.

Late presentation:
● Muscle weakness to paralysis
● Absent (or weak) distal pulses
● Cold peripheries
● Pallor or cyanosis
● Worsened pain and swelling

Answer is : B

229) all these characteristics for breast mass suggest benign condition, except:

A. Well-defined, circumscribed mass.


B. Radiolucent mass
C. Spiculated margins
D. Size changes with menstruation cycle
E. Fluctuant mass
Breast mass characteristics that favores benign condition rather than malignant :
● Well-defined, circumscribed mass on mammography
● A fluctuant mass may indicate a breast abscess.
● Soft, movable
● Radiolucent mass
● Painful
● Reduced or unchanged size
● Size changes with menstruation cycle

Answer is : C

230) What is the most common site for aspiration pneumonia ?

A. Right upper lobe


B. Right middle lobe
C. Right lower lobe
D. Left Upper lobe
E. Left lower lobe

Aspiration pneumonia
Definition: Aspiration is the inhalation of foreign material into the respiratory tract. It often
occurs after instrumentation of the upper airways or esophagus (e.g., upper GI endoscopy),
or secondary to vomiting and regurgitation of gastric content.

Pathogens :​ mixed infections with anaerobic organisms are common (e.g., Klebsiella)

Clinical findings:
● Immediate symptoms: bronchospasms , crackles on auscultation, hypoxemia with
cyanosis
● Late symptoms: fever, shortness of breath, cough with foul-smelling sputum
Diagnosis :
● Arterial blood gas analysis (↓ PaO2, pH < 7.35, PaCO2 > 45 mm Hg)
● Radiologic imaging: The lung region in which infiltrates are seen in depends on the
patient's position on aspiration. :
○ Supine position: superior segment of the ​right lower lobe ​(​most common
site of aspiration​)
○ Standing/sitting: posterior basal segment of the right lower lobe
○ Right lateral decubitus position: posterior segment of the right upper lobe or
right middle lobe

Answer is : C

231) One of the following is true regarding esophageal atresia :

A. Type A is the most common type


B. Oligohydramnios prenatally.
C. Surgery should be performed after 12 months
D. Provides protection from pneumonia.
E. Usually present with symptoms directly after birth

Esophageal atresia is a congenital defect in which the upper esophagus is not connected to
the lower esophagus, ending blindly instead.

Pathophysiology
● A wedge of mesoderm called the tracheoesophageal septum separates the
developing foregut (esophagus) from the trachea.
● Esophageal atresia and tracheoesophageal fistula are caused by a defect in
mesodermal differentiation
● About 50% of cases are associated with other mesodermal defects (​VACTERL
association​)
○ V​ertebral anomaly
○ A​nal atresia
○ C​ardiac anomaly
○ T​racheoesophageal fistula
○ E​sophageal atresia
○ R​enal anomaly
○ L​imb malformation

Classification :

∼ 8% of cases Esophageal atresia without tracheoesophageal fistula Type A


∼ 3% of cases Esophageal atresia with tracheoesophageal fistula to the proximal Type B
esophageal segment

∼ 84% of cases Esophageal atresia with tracheoesophageal fistula to the distal Type C
esophageal segment

∼ 1% of cases Esophageal atresia with tracheoesophageal fistula to the proximal Type D


and distal esophageal segments

∼ 4% of cases H‑type tracheoesophageal fistula without atresia Type E

Clinical features :

Prenatal :
● Polyhydramnios → increased risk of premature birth

Postnatal :
● Esophageal atresia → pooling of secretions → excessive secretions/foaming at the
mouth
● Tracheoesophageal fistula
○ If the fistula is connected to the proximal esophageal segment: aspiration and
subsequent ​aspiration pneumonia
■ Coughing spells
■ Rales
■ Cyanotic attacks
○ If the fistula is connected to the distal esophageal segment: gastric distention

Newborns usually present with symptoms directly after birth! The exception is the Gross type
E fistula: The diagnosis of a small H‑type tracheoesophageal fistula may occur as late as
adulthood.

Surgical treatment should be performed within the first 24 hours of birth.

Answer is : E

232) All are true about cutaneous squamous cell carcinoma, except:

A. Keratin pearls are seen in the tissue biopsy.


B. Punch biopsy can be both diagnostic and therapeutic.
C. Most commonly on the face and neck.
D. Dark skin is more prone than light skin.
E. Actinic keratosis considered a precancerous.

Cutaneous squamous cell carcinoma ​(cSCC) is the second most common skin cancer
after basal cell carcinoma. It occurs as a result of malignant transformation of keratinocytes
in the stratum spinosum (prickle cell layer) of the epidermis. Risk factors for malignant
transformation include exposure to sunlight, chemical carcinogens, precancerous lesions of
the skin (e.g., actinic keratosis), and sites of skin damage (e.g., scars, burns, ulcers).

Epidemiology :
Second most common form of skin cancer after basal cell carcinoma
Sex: ♂ > ♀ (2:1)
Incidence: increases with age
Light skin > Dark skin

Actinic keratosis ​is a skin lesion caused by exposure to UV rays that manifests as rough,
scaly skin plaques with the potential to progress to squamous cell carcinoma.

Answer is : D

233) 22 yo male brought to the ED after motor vehicle accident, no evidence for penetrating
wound, complains of abdominal pain and tenderness, his BP is 95/56, RR is 60/m, HR is
94/m, FAST scan detected hemoperitoneum, the next step is:
A. Colonoscopy.
B. Abdominal CT scan with contrast.
C. Abdominal CT scan without contrast.
D. Conservative management.
E. Laparotomy.

Approach to blunt abdominal trauma:


● Primary survey
○ Resuscitative procedures and stabilization (e.g., rapid transfusions, CPR,
etc.) if indicated
● Imaging to assess the location and extent of abdominal injury
○ FAST exam to detect hemoperitoneum
○ CT if FAST is inconclusive and patient is stabilized
■ Detects free abdominal fluid, solid organ injury, and retroperitoneal
injury (e.g., upper retroperitoneal hematomas)
○ X-ray: detects fractures, free intra-abdominal air, or large collections of blood;
less useful than CT or FAST
● Diagnostic peritoneal lavage: has been largely replaced by the rapid, noninvasive
FAST exam, but useful for assessing hemodynamically unstable patients if FAST is
inconclusive
○ If fecal matter or significant blood are detected (positive test) → emergent
laparotomy is indicated
○ Highly sensitive, but invasive
● Negative FAST/CT: identify extra-abdominal cause of hemodynamic instability
● Laparotomy is indicated for patients with
○ Hemodynamic instability
○ Signs of peritonitis
○ Intra-abdominal bleeding detected on imaging
● Conservative management
○ Depends on the location and extent of injury
○ Close monitoring of vital signs and serial examinations
○ Angiography and embolization (e.g., control bleeds, manage low
retroperitoneal hematomas)
○ Management of pancreas injury: percutaneous drainage (with culture) and
debridement to prevent complications (pseudocysts, abscess)
○ Management of duodenal injury: nasogastric suction and parenteral nutrition
to allow healing; if patients remain unstable, laparotomy may be indicated.

The absence of pain does not rule out significant intra-abdominal injury!
Imaging must be performed!

If FAST exam is not available, a hemodynamically unstable patient should be


taken to the operating room immediately!

Answer is : E
234) Patient lost sensation in the Buttock Posterior, and lateral thigh, Lateral aspect of leg
Dorsum of foot, Medial half of sole, First, second, and third toes. after a motor vehicle
accident. the lesion will be at the level of :

A. L2
B. L3
C. L4
D. L5
E. L6

The affected sensory areas on the skin indicates L5 lesion according to dermatome.

A dermatome is an area of skin that is mainly supplied by afferent nerve fibers from a
single dorsal root of spinal nerves which forms a part of a spinal nerve.
Answer is : D

235) Which of the following pancreatic secretions is produced in the endocrine cells ?

A. Somatostatin.
B. Phospholipase A.
C. Amylase.
D. Trypsin.
E. Electrolytes.
Endocrine pancreas:
● Produces different hormones that are primarily involved in the regulation of blood
glucose levels (see endocrine pancreas below)
● Composed of islets of Langerhans embedded within the exocrine pancreas
● Islet cell types
○ Alpha cells are located peripherally and produce glucagon.
○ Beta cells are located centrally and produce insulin.
○ Delta cells are dispersed within the islets and produce somatostatin.

Content of secretions from exocrine pancreas :


● Digestive enzymes
○ Trypsin and chymotrypsin: digestion of proteins (proteases)
■ Secreted as proenzymes (e.g., trypsinogen, chymotrypsinogen) by the
pancreatic acinar cells into the pancreatic duct
■ Activated in the duodenum: Trypsinogen is activated by enterokinase
on the surface of the duodenal mucosa to form trypsin; once activated,
trypsin activates chymotrypsinogen to chymotrypsin.
○ Pancreatic amylase: digestion of carbohydrates
○ Pancreatic lipase: digestion of lipids
○ Phospholipase A: digestion of phospholipids
○ Nucleases: digestion of RNA/DNA
○ Elastase: digestion of elastin fibers
● HCO3-
● Electrolytes (Na+, K+, Cl-)
● Water

Answer is : A

236) 52 y/o woman presented with recent asymmetrical breasts, she felt a bean like mass in
the upper outer quadrant, the mass is non-tender, next step in diagnosis:

A. Mammography
B. Ultrasound
C. FNA
D. Re-examine in 2 weeks
E. Core-biopsy.

The symptoms raise the suspicion of malignant breast mass:

Clinical features of breast cancer:


Patients with breast cancer develop clinical symptoms rather late at advanced tumor stages.
Typical signs may include:
● Changes in breast size and/or shape; asymmetric breasts
● Palpable mass: typically a single, nontender, firm mass with poorly defined margins,
most commonly in the upper outer quadrant
● Skin changes
○ Retractions or dimpling (due to tightening of the Cooper ligaments)
○ Peau d'orange: skin resembling an orange peel (due to obstruction of the
lymphatic channels)
■ Redness, edema, and pitting of the hair follicles
● Nipple changes: inversion, blood-tinged discharge
● Axillary lymphadenopathy: firm, enlarged lymph nodes (> 1 cm in size), that are fixed
to the skin or surrounding tissue
● In advanced stages: ulcerations

Diagnostic approach:
Answer is : A
237) A 67 y/o man presents to your clinic with a 2 cm in diameter skin lesion on the
forehead, the man worked as a construction worker for more than 20 years, the lesion is
non-regular circle with indurated and borders and depressed ulcerated center, the most
likely diagnosis.

A. Cutaneous squamous cell carcinoma


B. Keratoacanthoma
C. Trichoepithelioma
D. Basal cell carcinoma
E. Actinic keratosis

Basal cell carcinoma (BCC), a malignant neoplasm, is the most common type of skin cancer
and primarily affects individuals with light skin.

Clinical features :
● Localization
○ Areas of skin exposed to sun
■ The palms, soles of the feet, and mucous membranes are rarely
affected.
■ Eyelid
■ Nose
● Clinical appearance
○ Pearly, indurated and nodular lesion with rolled border that is usually
nontender
○ Central depression (crater) with ulceration
● Growth
○ Slowly growing over months to years
○ Very rarely metastasizes

Answer is : D
238) All the following sentences about the anatomy of thigh, except:

A. The femur is the longest and strongest bone of the body.


B. The deep fascia of the thigh separates the muscles into three compartments.
C. Femoral nerve is the largest branch of the lumbar plexus.
D. Femoral triangle provides an access to coronary angiography.
E. Quadriceps femoris main function is knee flexion.

Femoral nerve is the largest branch of the lumbar plexus

Femoral muscle compartments:

Femoral triangle:
Answer is : E

239) A 23 y/o male brought to ED after motor vehicle accident, complains of stabbing chest
pain and dyspnea, physical examination shows distended neck veins, Vital signs are as
follows: Blood pressure is 70/50 mmHg,
pulse 140/min
Next step in management is :

A. Chest X-ray.
B. IV fluids.
C. CT scan.
D. Chest tube placement.
E. FAST.

The patient’s symptoms raise the worry of tension pneumothorax, you can intervene before
establishing diagnosis.
Tension pneumothorax: life-threatening variant of pneumothorax characterized by
progressively increasing pressures within the chest and cardiorespiratory compromise.

Clinical features:
Clinical features vary from asymptomatic to cardiopulmonary compromise
● Sudden, severe, and/or stabbing, ipsilateral pleuritic chest pain and dyspnea
● Reduced, or absent breath sounds, hyperresonant percussion, decreased fremitus
on the ipsilateral side
● Subcutaneous emphysema
● Additionally in tension pneumothorax:
○ Severe acute respiratory distress: cyanosis, restlessness, diaphoresis
○ Reduced chest expansion on the ipsilateral side
○ Distended neck veins and hemodynamic instability (tachycardia, hypotension,
pulsus paradoxus)
○ Secondary injuries (e.g., open or closed wounds)

Mnemonic:​ P-THORAX: ​P​leuritic pain, ​T​racheal deviation, ​H​yperresonance, ​O​nset


​ duced breath sounds (and dyspnea), ​A​bsent fremitus, ​X​-rays show collapse.
sudden, ​Re

Diagnostic tests :
● Chest x-ray (confirmatory test)
○ Ideally in two projections (PA and lateral), in supine and upright position
■ Ipsilateral pleural line with reduced/absent lung markings
■ Sudden change in radiolucency
■ Deep sulcus sign: dark and deep costophrenic angle on the affected
side
■ If pulmonary disease is present: airway or parenchymal lesions
■ Additional features in tension pneumothorax:
■ Ipsilateral diaphragmatic flattening/inversion and widened
intercostal spaces
■ Tracheal deviation towards the contralateral side
● Arterial blood gas analysis (ABG) to detect respiratory acidosis

Treatment :
● Emergency chest decompression via chest tube placement if immediately available
● Otherwise perform emergency needle thoracostomy, followed by chest tube
placement

Answer is : D
240) All true about lichen planus, except :

A. Antihistamines used in treatment


B. Lesions can occur in many different forms.
C. Tissue biopsy shows Hyperkeratosis.
D. Topical Corticosteroids are contraindicated.
E. Genital lichen planus can evolve to SCC

Lichen planus is a chronic inflammatory disease of unknown origin that is characterized by


different types of lesions involving the skin and mucosa in middle-aged individuals. The most
common lesions are purple papules with well-demarcated, irregular borders, which typically
occur on the wrists, lower extremities, and genitoanal region. The surfaces of the lesions
often exhibit a pattern of white lines known as Wickham's striae.

Treatment:
● First-line therapy
○ High-potency topical corticosteroids (e.g., betamethasone)
○ Intralesional application in hypertrophic areas
● Second-line therapy
○ Oral corticosteroids (e.g., prednisone)
○ Phototherapy (UVB and PUVA)
○ Oral retinoids (e.g., acitretin)
● Adjunctive therapy: oral antihistamines (e.g., hydroxyzine) to manage pruritus

Answer is :

241) All the following sentences true about breast cancer, except :

A. Mammography is the best confirmatory diagnostic tool.


B. Associated with Li-Fraumeni syndrome.
C. Soft, movable mass is non suspicious
D. Most common malignant disease in women
E. Increased estrogen exposure is a risk factor

Breast cancer is the most common malignancy in women. The lifetime risk of developing
breast cancer for women is approx. 12%. The most important risk factors include increased
estrogen exposure, advanced age, and genetic predisposition (BRCA1/BRCA2 mutations).
Most breast cancers are adenocarcinomas. Histopathologic classification differentiates
between ductal and lobular carcinomas. The two most common types of breast cancer are
invasive ductal carcinoma, which accounts for 70–80% of all cases, and the less aggressive
invasive lobular carcinoma. Both types typically develop from noninvasive carcinomas, i.e.,
ductal carcinoma in situ (DCIS), and lobular carcinoma in situ (LCIS), respectively.

Mammography:
Although mammography does not confirm the diagnosis, it is primarily useful for early
detection of breast abnormalities!
Mammography findings :
Benign Malignant

● Well-defined, circumscribed mass ● Focal mass or density with poorly


● Radiolucent ring surrounding the defined margins
lesion (halo sign) ● Spiculated margins
● Diffuse microcalcification or coarse ● Clustered microcalcifications
calcification

Answer is : D

242) 34 y/o brought to the emergency department after A car crash, complaining of
excessive vomiting, dyspnea, physical examination: Spo2 is 85%, PR is 105/m, bowel
sounds heard over the left chest, what is the most likely diagnosis ?

A. Diaphragmatic rupture.
B. Aortic aneurysm.
C. Tension pneumothorax.
D. Cardiac tamponade.
E. Splenic rupture.

Diaphragmatic rupture :
● Etiology: penetrating injuries (65%), blunt trauma (35%)
● Clinical features
○ Often initially asymptomatic
○ Chest/abdominal wall bruises
○ In case of herniation of abdominal organs into the chest
■ Decreased breath sounds, bowel sounds in the thorax, respiratory
distress
■ Signs of bowel obstruction
● Diagnostics
○ Chest x-ray
■ Disturbed contour of the hemidiaphragm
■ Displaced abdominal organs (esp. stomach and bowel segments):
hourglass sign
■ Possible mediastinal shift
■ Nasogastric tube visible above the left hemidiaphragm
○ Ultrasound FAST: rapidly detect large tears or herniation
○ CT scan to confirm the diagnosis
● Complications: diaphragm paralysis
● Treatment: most patients require surgery

Answer is : A
243) 10 months old girl brought to the emergency room after sudden crying with bilious
vomiting every 20 minutes, Now she looks lethargic and unresponsive, physical exam
reveals sausage-shaped mass in the RUQ, a diagnosis of intussusception was established,
the initial step in management is :

A. Air pneumatic enema


B. Surgical reduction.
C. IV Fluids.
D. Compression ultrasound.
E. Laxatives.

Signs of dehydration appears on the infant, Iv fluids is important to rehydrate.

Treatment
● Initial steps: nasogastric decompression and ​fluid resuscitation
● Nonsurgical management (performed under continuous ultrasound or fluoroscopic
guidance)
○ Air (pneumatic) enema: treatment of choice
○ Hydrostatic reduction: normal saline (or water-soluble contrast enema)
○ Observe for 24 hours post-reduction, as there is a small risk of perforation
and recurrence is common during this period
● Surgical reduction
○ Indications
■ When a pathological lead point is suspected
■ Failed conservative management
■ Suspected gangrenous or perforated bowel
■ Critically ill patient (e.g., shock)
○ Open or laparoscopic method
■ Hutchinson maneuver: manual proximal bowel compression and
reduction of intussusception
■ For necrotic bowel segments: Resection and end-to-end anastomosis

Urgent intervention is necessary for intussusception to prevent


potentially life-threatening complications!

Answer is : C

244) All are true about acute intestinal ischemia, except:

A. Can cause septic shock.


B. MRI is superior to CT scan in diagnosing this condition.
C. Superior mesenteric artery is the most affected artery.
D. Venous thrombosis can cause acute intestinal ischemia.
E. Embolus from the heart causes half of the cases.
Acute intestinal ischemia is an acute inadequate blood flow to the small intestine (arterial or
venous) that can result in bowel infarction.

Clinical features
● Periumbilical pain that is disproportionate to physical findings
● Nausea and vomiting
● Diarrhea (bloody in later stages)
● Gangrenous bowel: rectal bleeding and signs of sepsis (e.g., tachycardia,
hypotension)
● Clinical courses
○ Acute arterial embolism: most abrupt and painful onset of all types
(“abdominal apoplexy”)
○ Acute arterial thrombosis: presentation less severe because patients have
better collateral supply
○ Nonocclusive ischemia: symptoms develop over several days
○ Venous thrombosis: symptoms less dramatic, worsen gradually (e.g.,
abdominal discomfort evolves over a week)

Diagnostics
● Laboratory findings
○ ↑ Lactate, ↑ LDH, ↑ creatine kinase
○ Leukocytosis
○ Metabolic acidosis
● CT angiography (confirmatory test)
○ Detects disrupted flow and vascular stenosis
○ Distended intestinal loops and air-fluid levels, wall thickening, pneumatosis
intestinalis (suggests transmural ischemia or infarction)
○ Alternative: MR angiography
■ Advantage: no radiation
■ Disadvantage: ​less accurate evaluation of the IMA
● Ultrasound
○ Detection of distended intestinal loops and free fluid in the abdominal cavity in
case of perforation
○ Color Doppler ultrasound to detect stenosis in arterial branches
● Evaluation of underlying disease (e.g., ECG for atrial fibrillation or myocardial
infarction)

Answer is : B
245) 28 y/o female who works at a call center, presentes with gradual onset sensory loss in
her left hand, no history of trauma, she can’t extent her little and ring finger, physical
examination shows positive Wartenberg sign, what is the most likely diagnosis ?

A. Cubital tunnel syndrome.


B. Carpal tunnel syndrome
C. UCL Insufficiency
D. Thoracic Outlet Syndrome (TOS)
E. Scaphoid fracture

Clinical features
● Muscle weakness and atrophy
○ Claw hand deformity: little and ring finger loss of extension at proximal
interphalangeal joint and loss of flexion at metacarpophalangeal joint
○ Wartenberg sign​: little finger in persistent abduction due to weak third palmar
interosseous muscle
○ Froment sign: The thumb flexes at the interphalangeal joint while pinching a
piece of paper to compensate for a weak adductor pollicis muscle.
● Sensory loss
○ Loss of sensation over the hypothenar eminence, ulnar 1 ½ fingers.
○ Lesion at the elbow: positive Tinel test → Marked paresthesias can be
reproduced in the ulnar portion of the hand by tapping on the medial
epicondyle of the humerus.
○ Lesion at the wrist: Sensory symptoms may or may not be present.
● Pain: Elbow lesions typically present with referred pain in the forearm.
Answer is : A

246) “Iliotibial band syndrome” is commonly known injury in which of these sports ?

A. Wrestling
B. Tennis.
C. Skating.
D. Boxing.
E. Runners.

Caused by repetitive knee flexion and extension movements frequently seen in cyclists and
runners.

Iliotibial band syndrome (runner's knee)


● Definition: common overuse injury of the distal portion of the iliotibial band (over the
lateral femoral epicondyle)
● Etiology: repetitive flexion and extension of the knee (e.g., during running)
● Clinical features
○ Sharp pain at the lateral knee when the foot strikes the ground; dull aching,
constant pain at rest
○ Noble test: The patient lies on their side and the examiner passively flexes
the patient's leg while exerting constant pressure on the lateral femoral
epicondyle with his thumb → test positive if pain is elicited
Answer is : A

247) All the following true about femoral hernia, except:

A. High risk of incarceration and strangulation.


B. Chronic constipation is a risk factor.
C. Easily confused with inguinal hernia.
D. It is a common type of hernia.
E. All femoral hernias should be surgically repaired.

A femoral hernia is an uncommon type of inguinal hernia, in which intra-abdominal contents


(e.g., intraperitoneal fat, mesentery, bowels) herniate into the femoral canal through the
femoral ring. Risk factors for femoral hernias include old age, female gender, obesity, and
previous hernia repair. Patients present with a globular swelling inferior to the inguinal
ligament and medial to the femoral vein that worsens with coughing or straining.

Epidemiology
● Uncommon hernia​ (∼ 5% of all hernias)
● Sex: ♀ > ♂ (3:1)
● Peak incidence: 40–70 years

Etiology
● Advancing age and female gender
● Increased intra-abdominal pressure
○ Obesity
○ Chronic constipation
○ Chronic cough (e.g., due to COPD)
○ Straining during micturition (e.g., due to prostatic hypertrophy)
● Multiparity
● Previous abdominal surgeries (especially those involving the inguinal region)

All femoral hernias should be surgically repaired because of the high risk of complications.

Answer is : D

248) In the definition of epidemiology, “determinants” generally includes all, except:

A. Risk factors
B. Causes
C. Control measures
D. Sources
E. Agents

In the definition of epidemiology, “determinants” generally includes the ​causes ​(including


agents​), ​risk factors ​(including ​exposure ​to sources), and ​modes of transmission​, but does
not include the resulting public health action.

Answer is : D

249) The following are etiologies for paralytic ileus, except one:

A. Hypokalemia.
B. Abdominal surgery.
C. Retroperitoneal hemorrhage.
D. Paracetamol.
E. Diabetes mellitus.

Paralytic ileus: temporarily impaired peristalsis of the gastrointestinal tract in the absence of
mechanical obstruction
Etiology
● Intra-abdominal surgery (postoperative ileus)
● Abdominal trauma (e.g., due to retroperitoneal hemorrhage)
● Endocrine abnormalities (e.g., hypothyroidism, porphyria, uremia)
● Electrolyte disturbances (e.g., hypokalemia)
● Neuropathy (e.g., diabetes mellitus, spinal injury)
● Neurosurgical procedures (e.g., spinal surgery)
● Vascular diseases (e.g., mesenteric ischemia)
● Peritonitis
● Inflammation of intra-abdominal organs (e.g., appendicitis, cholecystitis, pancreatitis,
severe gastroenteritis)
● Medications (e.g., anticholinergics, opioids, antidepressants)

The common causes of paralytic ileus can be memorized using “5 Ps”: Peritonitis,
Postoperative, low Potassium, Pelvic and spinal fractures, and Parturition.

Answer is : D

250) Gas gangrene is most likely caused by which of these microorganisms ?

A. C. tetani.
B. Streptococcus pyogenes.
C. Clostridium perfringens.
D. MRSA.
E. VRSA.

Gas gangrene is a bacterial infection that produces tissue gas. This deadly form of gangrene
usually caused by Clostridium perfringens bacteria.

Answer is : C

251) A 64 y/o male presented with jaundice and high ALT and AST levels, he has a history
of bowel resection, during the surgery which of these anesthetics was most likely used ?

A. Desflurane.
B. Halothane.
C. Nitrous oxide.
D. Propofol.
E. Midazolam.

The patient mostly used inhaled halothane as it severe hepatotoxic drug.

Halothane is a potent volatile halogenated anesthetic gas that has been linked to many
cases of acute liver injury that are frequently severe.
Halothane hepatitis
● Pathophysiology: underlying mechanism not fully understood
● Clinical features
○ Occurs 2 days to 3 weeks after halothane exposure
○ Signs of acute hepatitis
○ Rash, arthralgias
● Diagnostics: diagnosis of exclusion
○ Possible laboratory findings: ↑ eosinophils, ↑ serum transaminases , ↑
bilirubin, ↑ alkaline phosphatase
○ Biopsy shows massive centrilobular hepatic necrosis
● Treatment: depending on the severity of liver damage, ranges from supportive
treatment to liver transplantation

Answer is : B

252) 41 y/o female came to the emergency department with 15% TBSA 2nd degree burn
and 10% TBSA 3rd degree burn, her body weight is: 60kg, according to parkland formula,
what is the volume of ringer’s lactate she should receive in the first 8 hours ?

A. 3 Liters.
B. 6 Liters.
C. 7 Liters.
D. 2.5 Liters.
E. 5.5 Liters.

Parkland formula​ is used to guide initial fluid therapy: the volume of lactated
Ringer's solution to be administered within a period of 24 h = 4 mL x % of total body surface
involved in 2nd and 3rd-degree burns x body weight (in kg)
Half of this volume should be administered within the first 8 hours and the remaining half
within the next 16 hours

The above case equation, 4x(15+10)x60 = 6000ml


First 8 hours = 3000ml
Next 18 hours = 3000ml

Current recommendations by the ATLS and the American Burn


Association: volume of lactated Ringer's solution to be administered
within a period of 24 h = 2 mL x % of body surface involved in 2nd and
3rd-degree burns x body weight (in kg)

Answer is : A
253) For how long the Liver stores Vitamin B12 ?

A. 10 -20 Days
B. 5 - 7 Months.
C. 1 - 2 Months.
D. 1 - 2 Years
E. 3 - 4 Years.

Characteristics of Vitamin B₁₂


● Synonyms: cobalamin
● Active forms: methylcobalamin and adenosylcobalamin (activation occurs in the liver)
● Sources
○ Almost exclusively in animal products (except honey); some dried and
fermented plant foods (e.g., tempeh, nori)
○ Synthesized by the intestinal flora
● Absorption
○ Stomach: Cobalamin is released from ingested proteins by pepsin and binds
to the glycoprotein haptocorrin (R-protein), which protects it from gastric acid.
○ Duodenum: Cobalamin is released from haptocorrin by trypsin and binds to
intrinsic factor (IF), a protein produced by the parietal cells of the stomach
that facilitates cobalamin absorption in the ileum.
○ Terminal ileum: Cubilin receptor-mediated endocytosis of the intrinsic
factor-cobalamin complex → breakdown of IF in enterocytes, releasing
cobalamin→ cobalamin binds to carrier protein transcobalamin II and then
enters the plasma → cobalamin is either delivered to metabolically active
tissues or stored in the liver
● Transport in blood: via transcobalamin
● Storage: 60% in the liver (​enough for approx. 3 - 4 years​), 30% in muscle tissue

Answer is : E

254) Hodgkin lymphoma differentiate from which of these immune cell types ?

A. Neutrophils.
B. T-Cell.
C. Natural killer cell.
D. B-Cell.
E. Lymphoid progenitor cell.

Lymphoid progenitor cells do not differentiate to cause lymphoma, they cause lymphocytic
leukemia.
Hodgkin lymphoma (HL) is a malignant lymphoma that is typically of B-cell origin. The
incidence of HL has a bimodal age distribution, with peaks in the 3rd and 6th–8th decades of
life.

Suspicious lymph nodes are excised and definitive diagnosis is made via histological
analysis, which characteristically reveals pathognomonic ​Reed‑Sternberg cells (malignant
B-cells).

Answer is : D

255) All the following conditions can cause portal hypertension, except:

A. Hepatic metastases
B. Splenic vein thrombosis
C. Chronic hepatitis
D. Cholecystitis.
E. Budd-Chiari syndrome

Portal hypertension is present if the portal venous pressure is ≥ 6 mm Hg (normal value: 1–5
mm Hg). Portal venous pressure > 10 mm Hg is clinically significant and > 12 mm Hg is
associated with complications. Causes of portal hypertension can be classified as follows:

● Prehepatic
○ Portal vein thrombosis
○ Splenic vein thrombosis
○ Hepatosplenic schistosomiasis
● Intrahepatic (most common)
○ Cirrhosis including fibrous proliferation
○ Massive hepatic metastases
○ Chronic hepatitis
● Posthepatic
○ Budd-Chiari syndrome
○ Right-sided heart failure
○ Constrictive pericarditis
Answer is : D

256) Psammoma bodies are seen in all the following tumors histopathology, except:

A. Ovarian cystadenocarcinoma
B. Hepatocarcinoma.
C. Papillary thyroid carcinoma
D. Papillary renal cell carcinoma
E. Meningiomas

A psammoma body is a round collection of calcium, seen microscopically. The term is


derived from the Greek word ψάμμος (psámmos), meaning "sand".

Psammoma bodies are commonly seen in certain tumors such as:


● Papillary thyroid carcinoma
● Papillary renal cell carcinoma
● Ovarian papillary serous cystadenoma and cystadenocarcinoma
● Endometrial adenocarcinomas (Papillary serous carcinoma ~3%-4%)
● Meningiomas, in the central nervous system
● Peritoneal and Pleural Mesothelioma
● Somatostatinoma (pancreas)
● Prolactinoma of the pituitary
● Glucagonoma
● Micropapillary subtype of Lung Adenocarcinoma
Answer is : B

257) 74 y/o man presents with a palpable lump in his neck, the doctor suspected a thyroid
tumor and ordered a thyroid scintigraphy, the results showed a cold nodule, this finding
indicates which of the following?

A. Inflamed nodule.
B. Normal functioning nodule.
C. Hyper functioning nodule.
D. Necrotic nodule.
E. Non-functioning nodule.

Thyroid scintigraphy
● Indications:
○ Thyroid nodule with ↓ TSH level
○ Evaluation of ectopic thyroid tissue or retrosternal goiter
● Findings: decreased tracer uptake suggests a ​malignant non-functioning (cold)
nodule.

Nodules that appear hypoechoic on ultrasound and cold in scintigraphy are highly suspicious
for malignancy!
Answer is : E

258) 32 years old patient complains of dull RUQ pain that becomes more notable
postprandial, he declared a yellow eyes and skin with more severe pain 2 months ago but
resolved spontaneously, ultrasound shows gallstones in the gallbladder and clear bile ducts,
the best management is:

A. Medical litholysis
B. NSAIDs
C. Laparoscopic cholecystectomy.
D. ERCP
E. MRCP

The patient has a symptomatic gallstones which potentially caused obstructive jaundice 2
months ago. The treatment of choice is Laparoscopic cholecystectomy.

ERCP and MRCP could be useful 2 months ago when he had symptoms of
Choledocholithiasis.
Treatment
● Approach: supportive therapy and dietary modifications → elective cholecystectomy
only for symptomatic patients who are surgical candidates or asymptomatic patients
at risk of gallbladder cancer
● Supportive therapy
○ Fasting or dietary modification (decreased fat intake)
○ Spasmolytics (e.g., dicyclomine)
○ Analgesia: NSAIDs, opioids
● Surgical management
○ Laparoscopic cholecystectomy (curative)
■ Indications:
■ Symptomatic cholelithiasis
■ Asymptomatic cholelithiasis with an increased risk of
gallbladder cancer (e.g., gallbladder polyps or porcelain
gallbladder present)
■ Timing: as soon as possible for biliary colic
● Medical management
○ Medical litholysis (e.g., oral dissolution therapy): oral administration of bile
(chenodeoxycholic acid, ursodeoxycholic acid)
■ Indication: cholesterol stones without calcification (≤ 20 mm) for
inoperable patients or patients who do not wish to undergo
cholecystectomy
■ Treatment duration: at least 6 months
■ Success rate < 50%
Cholecystectomy is usually not indicated in asymptomatic cholelithiasis!

Answer is : C

259) A 79 years old man came to you because of weight loss, night sweating, and rectal
bleeding, he smokes 1 pack a day for the last 45 years, he has a history of CVA, and takes
simvastatin, aspirin and Valsartan, BMI = 31, his father died with colon cancer Which of the
following is not a risk factor for colorectal cancer ?

A. 79 years old
B. BM = 31
C. Smoking.
D. Aspirin.
E. Family history.

Predisposing factors of colorectal cancer :


● Colorectal adenomas
● Family history
● Hereditary syndromes
○ Familial adenomatous polyposis: 100% risk by age 40
○ Hereditary nonpolyposis colorectal cancer (HNPCC): 80% progress to CRC.
● Conditions associated with an increased risk of colorectal cancer
○ Inflammatory bowel disease (IBD): ulcerative colitis and Crohn's disease
■ Chronic inflammation → hyperplasia → non-polypoid dysplasia→
neoplasia
○ Endocarditis and bacteremia due to Streptococcus gallolyticus is associated
with CRC.
● Diet and lifestyle
○ Smoking
○ Alcohol consumption
○ Obesity
○ Processed meat; high-fat, low-fiber diets
● Older age

Protective factors
● Physical activity
● Diet rich in fiber and vegetables and lower in meat
● Long-term use of ​aspirin a
​ nd other NSAIDs

Answer is : D

260) 40 y/o man presents with hemoptysis started 2 weeks ago and becoming more severe,
he lost 5 kg since last month, he is a heavy smoker, the patient is high risk for lung cancer
what is the next step ?

A. MRI.
B. CXR.
C. CT scan.
D. PET.
E. Bronchoscopy.

The first step in diagnosing lung cancer always CXR.

Approach to suspected lung cancer and workup of a solitary


pulmonary nodule

1. Chest x-ray and comparison to previous images if available


2. CT imaging for further evaluation indicated if
○ New lesion detected on chest x-ray
○ Changes (e.g., enlargement) compared to previous chest x-ray
○ No previous CXR is available
3. Assessment of lesion size and probability of malignancy (based on CT findings and
patient characteristics)
○ Increased probability of malignancy
■ History of smoking
■ Patient age > 40 years
■ Other known risk factors (e.g., positive family history, asbestos
exposure)

Solid lesion size Probability of malignancy Next step

Low No follow-up needed


< 4 mm
High Follow-up CT at 12 months

Low Follow-up CT at 12 months


4–6 mm
High Follow-up CT at 6–12 months

Low Follow-up CT at 6–12 months


6–8 mm
High Follow-up CT at 3–6 months

≥ 8 mm Low or high PET and/or biopsy

Answer is : B

261) The study of the association of exposure with and outcomes is called:

A. Prevalence study
B. Case-control study
C. Cohort study
D. Adoption study
E. Twin concordance study

Case-control study
● Aim: to study if an exposure is associated with an outcome (e.g., disease)
● Study method :
1. Researchers begin by selecting patients with the disease (cases) and without
the disease (controls) with matching baseline characteristics from the same
source population.
2. The observer compares the presence of risk factors between these two
groups.
3. The odds ratio is then determined between these groups.
● Example: determining the link between cervical cancer and human papillomavirus
(HPV) exposure by comparing otherwise similar (e.g., same age) patients with and
without histologically confirmed cervical cancer

Answer is : B

262) All the following are physiological changes in pregnancy, except:

A. Systolic murmur
B. Hyponatremia
C. Edema of lower limbs
D. Ketonuria.
E. Decreased total lung capacity.

Physiological changes in pregnancy:


Answer is : D

263) All the following drug use in pregnancy and their effect match, except :

A. Tetracycline - Bone damage


B. ACE inhibitors - Cardiovascular and central nervous system malformations
C. Iodides - Congenital goiter
D. Angiotensin receptor blockers - Severe renal malformation
E. Acyclovir - Teratogenic

Aciclovir, also known as acyclovir, is an antiviral medication. It is primarily used for the
treatment of herpes simplex virus infections, chickenpox, and shingles.

Classified as a Category B Drug, the CDC and others have declared that during severe
recurrent or first episodes of genital herpes, aciclovir may be used. For severe HSV
infections (especially disseminated HSV), IV acyclovir may also be used.

Answer is : E

264) 50 y/o postmenopausal woman presented with constant minimal vaginal bleeding
Ultrasonography reveals endometrial thickening, no mass was detected, endometrial
histopathology shows “Swiss cheese pattern” with no metaplastic changes, the most likley
diagnosis is:

A. Endometrial cancer.
B. Vaginal atrophy.
C. Endometrial polyps.
D. Uterine sarcoma.
E. Endometrial hyperplasia.

Endometrial hyperplasia
● Etiology: increased estrogen stimulation leads to excessive proliferation of the
endometrium, e.g., in:
○ Follicle persistence
○ PCOS
○ Estrogen-producing ovarian tumors (e.g., granulosa cell tumors, theca cell
tumors)
○ Hormone replacement therapy without progestin administration
● Classification: based on histology
○ Simple endometrial hyperplasia
■ Histology: both stromal and glandular cells. Enlarged, dilated mucous
membrane glands (​Swiss cheese pattern​) are located between
abundant stromal tissue.
■ Risk of carcinoma
■ ∼ 1% in simple hyperplasia of the endometrium without atypia
■ ∼ 10% in simple hyperplasia of the endometrium with atypia
○ Complex endometrial hyperplasia
■ Histology: pronounced proliferation of glandular tissue. The glands are
positioned, in part, back-to-back ("dos-à-dos") with no separating
stroma
■ Grades I–II: no nuclear atypia
■ Grade III: additional cell atypia such as extensive mitosis and
loss of cell polarity
■ Risk of carcinoma
■ ∼ 3–10%: complex endometrial hyperplasia without atypia
(grades I–II)
■ ∼ 30%: complex endometrial hyperplasia with atypia (grade III)
● Clinical features: constant bleeding, intermenstrual bleeding, postmenopausal
bleeding
● Diagnosis
○ Ultrasonography: endometrial thickening (> 1 cm in premenopausal women
and > 5 mm in postmenopausal women)
○ Hysteroscopy with fractional curettage
○ Clinical chemistry: FSH, estradiol, testosterone

Answer is : E

265) 30 y/o woman came to the gynecologist with amenorrhea she suspects pregnancy, the
doctor used ultrasound for diagnosis, how early the embryo can be detectable on abdominal
ultrasound ?

A. 2-3 weeks gestation


B. 5-6 weeks gestation.
C. 7-8 weeks gestation
D. 10 -12 weeks gestation
E. 15- 18 weeks gestation.

Ultrasound findings in normal pregnancy (abdominal or transvaginal) :


● At 5–6 weeks of pregnancy: detection of the embryo
● At 10–12 weeks of pregnancy: detection of fetal heartbeat with Doppler ultrasound
● At 18–20 weeks of pregnancy: fetal movements

Answer is : B
266) All the following are identified risk factors for cervical incompetence, except:

A. Ehlers-Danlos syndrome
B. Oligohydraminos.
C. Prior cone biopsies
D. Prior cesarean section.
E. Septate uterus.

Cervical insufficiency (formerly called cervical incompetence) is painless cervical dilation


resulting in delivery of a live fetus during the 2nd trimester. Transvaginal cervical
ultrasonography during the 2nd trimester may help assess risk. Treatment is reinforcement
of the cervical ring with suture material (cerclage).

Risk factors
Most women with cervical insufficiency do not have risk factors; however, the following risk
factors have been identified:
● Congenital disorders of collagen synthesis (eg, Ehlers-Danlos syndrome)
● Prior cone biopsies (particularly when ≥ 1.7 to 2.0 cm of the cervix was removed)
● Prior deep cervical lacerations (usually secondary to vaginal or cesarean delivery)
● Prior excessive or rapid dilation with instruments (now uncommon)
● Müllerian duct defects (eg, bicornuate or septate uterus)
● ≥ 2 prior fetal losses during the 2nd trimester

Answer is : B

267) 35 weeks gestation primigravida came to the birth center with moderate contractions
which were irregular, ultrasound shows no cervical dilatation, in which stage this lady is ?

A. Latent first stage.


B. Active first stage.
C. Pre labor stage.
D. False labor.
E. Braxton Hicks contractions.
Obstetric contractions :

Answer is : D

268) An HIV+ 25 weeks gestation pregnant woman with the viral load of 15000 copies/mL,
she is concerned about her baby’s health, she is on retroviral therapy, she wants to go
through normal vaginal delivery, what is the next step:

A. Repeat viral load test at 38 weeks.


B. Cesarean section at 38 weeks.
C. Normal vaginal delivery 38 weeks.
D. Give dexamethasone.
E. Stop retroviral therapy immediately.

The woman is not near the delivery time, to determine the delivery method first she have to
do viral load test near the delivery time.

Retroviral therapy should be continued during pregnancy

HIV in pregnancy
● Transmission
○ Highest risk during birth (perinatal vertical transmission)
○ Prenatal transmission is possible
○ Risk depends on maternal viral load
● Managements for reducing risk of transmission
○ Combined antiretroviral therapy (cART) is recommended throughout
pregnancy
■ However, most antiretroviral drugs are not approved for use during
pregnancy.
○ Delivery method
■ Viral load > 1,000 copies/mL (or unknown) near the time of delivery:
increased risk of HIV transmission
■ Cesarean delivery should be scheduled at 38 weeks (even if
the mother received cART during pregnancy)
■ HIV post-exposure prophylaxis with zidovudine, lamivudine
and nevirapine OR zidovudine and nevirapine
■ Viral load ≤ 1,000 copies/mL and mother has received cART during
pregnancy: low risk of HIV transmission
■ Vaginal delivery may be considered as an alternative to
cesarean section
■ HIV post-exposure prophylaxis with zidovudine for the
newborn
○ Breastfeeding should generally be avoided , because the risk of transmission
is 5–20% .
● Diagnosis in infants: if < 18 months, diagnosis is confirmed via PCR, not ELISA

Answer is : A

269) All are true about fetus orientation and presentation, except :

A. Longitudinal lie is when the fetus is in the same axis with mother
B. Complete breech is called cannonball position
C. Caput succedaneum is caused by obstructed labor.
D. Macrosomia causes obstructed labor.
E. In breech presentation intervention necessary after 35 weeks.

Caput succedaneum is a scalp swelling that is formed in the neonate by the stagnation of
fluid within the layers of the scalp. Caused by prolonged engagement of the head in the birth
canal, which restricts venous drainage of the scalp.

No intervention necessary before 37 weeks' gestation, as most fetuses spontaneously


convert to cephalic presentation as they get closer to term

Answer is : E
270) Secondary postpartum hemorrhage is defined by which of the following ?

A. Blood loss > 500 mL after vaginal birth.


B. Blood loss from 24 hours to 12 weeks postpartum
C. Blood loss > 1000 mL for cesarean section.
D. Blood loss after a Placental traction.
E. Blood loss within 24 hours.

Depending on the source, primary postpartum bleeding is defined as blood loss in excess of
500 ml following vaginal delivery or 1000 ml following caesarean section in the first 24 hours
following birth. Secondary postpartum bleeding is that which occurs after the first day and up
to six weeks after childbirth.

Answer is : B

271) All the following are true about fetal development, except:

A. Fetal sex can be identified at week 12.


B. Start of coordinated movement of the arms and legs at week 14.
C. The umbilical vein becomes ligamentum teres postnatally.
D. The umbilical vein transports deoxygenated blood.
E. Bilirubin excretion is remains the task of the placenta.

The umbilical vein transports oxygenated blood from the placenta towards the fetal heart,
whereas the umbilical arteries direct deoxygenated blood from the fetus to the placenta.
Answer is : D

272) A woman in labor in Latent phase, A CTG detected early decelerations of the fetal
heart, all are true about this condition, except:

A. Nadir occurs with the peak of a contraction


B. Typically occur during active labor
C. No fetal distress
D. Caused by umbilical cord compression.
E. Fetal HR returns to normal at the end of contraction.

● Early deceleration
○ The beginning and end of deceleration correspond with the progression of a
contraction; deceleration reaches its minimum, known as the nadir, when the
contraction curve attains its peak.
○ Onset to nadir is gradual (≥ 30 sec)
○ Typically occur during active labor when the cervix is dilated ≥ 5 cm and the
head is engaged within the pelvic cavity
○ Usually a normal reading (no fetal distress)
○ Causes: ​compression of the head during contraction​ triggers a vagal
response
● Late deceleration
○ Decrease in the FHR following the maximum contraction curve
○ Onset to nadir is gradual (≥ 30 sec)
○ Causes: uteroplacental insufficiency (leads to fetal hypoxia and acidosis)
● Variable deceleration
○ Rather variable presentation and temporal relation to the contractions
changes
○ Onset to nadir is abrupt (< 30 sec) and lasts at least 15 sec
○ Causes
■ Umbilical cord compression/prolapse
● Prolonged deceleration
○ A decrease in FHR by ≥ 15 bpm from baseline, lasting ≥ 2 min but < 10 min
○ Causes
■ Same as those for late and variable deceleration, but more prolonged
and severe
■ Continued uterine contraction, inferior vena cava syndrome, peridural
anesthetic, rapid decrease in the mother's blood pressure

Answer is : D
273) All of the following can cause postpartum haemorrhage, except:

A. Trauma
B. Uterine atony
C. Antiphospholipid syndrome.
D. Von Willebrand Disease.
E. Retained placenta

Antiphospholipid syndrome hypercoagulable state, thus it can’t cause PPH.

Primary postpartum bleeding is defined as blood loss in excess of 500 ml following vaginal
delivery or 1000 ml following caesarean section in the first 24 hours following birth.

etiology
The causes of PPH have been described as the "four T's":
● Tone: uterine atony, distended bladder.
● Trauma: lacerations of the uterus, cervix, or vagina.
● Tissue: retained placenta or clots.
● Thrombin: pre-existing or acquired coagulopathy.

The most common cause of PPH is uterine atony, followed by retained placenta.

Answer is : C

274) 30 weeks gestation presented after a profuse bleeding which stopped before arriving to
the hospital, it was not painful, on physical examination the uterus is soft and non tender, All
the following procedures can be used, except :

A. Transvaginal ultrasound.
B. CTG
C. CBC
D. Digital Vaginal exam.
E. Transabdominal ultrasound

The woman is suspected to have a placenta previa, Digital vaginal examinations are
contraindicated

Placenta previa is implantation of the placenta over or near the internal os of the cervix.
Typically, painless vaginal bleeding with bright red blood occurs after 20 weeks gestation.

Clinical features
● Sudden, painless, bright red vaginal bleeding
● Usually occurs during the 3rd trimester (before rupture of the membranes), stops
spontaneously after 1–2 hours, and recurs during birth
● Often causes preterm delivery (∼ 45% of cases)
● Soft, nontender uterus
● Usually no fetal distress
In contrast to placental abruption, bleeding in patients with placenta previa is painless!

Diagnostics
● Transvaginal ultrasound to assess the position of the placenta
● Digital vaginal examinations are contraindicated in cases of hemorrhage of unknown
cause!

Answer is : D

275) All are clinical features of endometriosis, except:

A. Cyclical pelvic pain.


B. Infertility.
C. Dysmenorrhea.
D. Dyschezia.
E. Vaginitis.

Endometriosis ​is a chronic oestrogen-dependent condition characterised by the growth


of endometrial tissue in sites other than the uterine cavity, most commonly the pelvic cavity

Clinical features
● Up to one-third of patients are asymptomatic.
● Chronic pelvic pain that worsens before the onset of menses
● Dysmenorrhea
● Pre or post menstrual bleeding
● Dyspareunia
● Infertility
● Dyschezia: A condition of difficult or painful defecation (usually due to hard
stools/constipation)

Answer is : E

276) 25 y/o woman presented with severe but gradual lower abdominal pain in the left side
nausea, shoulder pain, amenorrhea, physical examination shows diffuse abdominal
tenderness, BP: 95/65. most likely diagnosis:
A. Ectopic pregnancy
B. Ovarian cyst rupture
C. Cardiogenic shock.
D. PID
E. Endometriosis

The patient shows signs of ruptured ectopic pregnancy leads to hemodynamic instability.
Endometriosis usually don’t cause shock
Ovarian cyst rupture causes sudden pain
PID Usually bilateral and comes with vaginal discharge.

Clinical features

General symptoms of ectopic pregnancy


Patients usually present with signs and symptoms 4–6 weeks after their last menstrual
period.
● Lower abdominal pain and guarding
● Vaginal bleeding
● Signs of pregnancy: amenorrhea, nausea, breast tenderness, frequent urination
● Tenderness in the area of the ectopic pregnancy
● Cervical motion tenderness, closed cervix
● Enlarged uterus
● Interstitial pregnancies tend to present late, at 7–12 weeks of gestation, because of
myometrial distensibility.

Tubal rupture
● Acute course with sudden and severe lower abdominal pain (acute abdomen)
● Signs of hemorrhagic shock: e.g., tachycardia, hypotension, syncope
● More common in interstitial pregnancy

Answer is : A

277) All are indications of surgical intervention in ectopic pregnancy, except:

A. Renal insufficiency
B. Hemodynamic instability
C. Significant pain
D. Serum hCG level ≥5000 IU/L
E. No fetal heartbeat.

No fetal heartbeat indicates no further fetal growth, thus it is an indication for conservitive
management.
Management of ectopic pregnancy:

Conservative management
● Indications
○ Uncomplicated ectopic pregnancies
○ Hemodynamic stability
○ β-hCG ≤ 5000 mlU/mL
○ No renal, hepatic, or hematologic diseases
○ No fetal heartbeat and ectopic mass size < 4 cm
● Treatment of choice: methotrexate (MTX)
○ Outcome comparable to surgery
○ A decrease in β-hCG levels should occur within a week of MTX
administration.
● Anti-D immunoglobulin (RhoGAM)
● Alternative: expectant management

Surgery
● Indications
○ Hemodynamic instability.
○ Impending rupture :
■ Significant pain.
■ Adnexal mass ≥35 mm.
■ Fetal heartbeat visible on scan.
■ Serum hCG level ≥5000 IU/L.
○ Contraindications for MTX treatment: e.g., renal insufficiency
○ If conservative treatment is unsuccessful
● Laparoscopic removal
○ Salpingostomy (tube‑conserving operation)
○ Salpingectomy (not function-preserving)

Answer is : E

278) the round ligament of the uterus connect the uterus with which of the following ?

A. Ovaries.
B. Cervix.
C. Labia majora.
D. Fallopian tubes.
E. Pelvic wall.

The round ligament of the uterus originates at the uterine horns, in the parametrium. The
round ligament exits the pelvis via the deep inguinal ring, passes through the inguinal canal
and continues on to the labia majora where its fibers spread and mix with the tissue of the
mons pubis.
Answer is : C

279) 29 y/o female presented with vaginal bleeding 10 days before the expected menstrual
period, all are causes of this bleeding, except:

A. Gonorrhea
B. Fluoxetine
C. Vaginitis
D. Ectopic pregnancy
E. Tranexamic acid.

Tranexamic acid is a medication used to treat or prevent excessive blood loss from major
trauma, postpartum bleeding, surgery, tooth removal, nosebleeds, and heavy menstruation.
It is also used for hereditary angioedema.

Intermenstrual bleeding ​(previously known as metrorrhagia) is uterine bleeding at


irregular intervals, particularly between the expected menstrual periods.
Causes of metrorrhagia :
● Pregnancy-related, including ectopic pregnancy and gestational trophoblastic
disease.
● Physiological:
○ 1-2% have vaginal spotting around ovulation.
○ Hormonal fluctuation during the perimenopause (this should be a diagnosis of
exclusion).
● Vaginal causes:
○ Adenosis.
○ Vaginitis (bleeding uncommon before menopause).
○ Tumours.
● Cervical causes:
○ Infection - chlamydia, gonorrhoea.
○ Cancer (but bleeding is most often postcoital).
○ Cervical polyps.
○ Cervical ectropion.
○ Condylomata acuminata of the cervix.
● Uterine causes:
○ Fibroids (occur in over 25% of women of reproductive age).
○ Endometrial polyps.
○ Cancer (endometrial adenocarcinoma, adenosarcoma and leiomyosarcoma).
○ Adenomyosis (usually only symptomatic in later reproductive years).
○ Endometritis.
● Oestrogen-secreting ovarian cancers.
● Iatrogenic causes:
○ Tamoxifen.
○ Following smear or treatment to the cervix.
○ Missed oral contraceptive pills.
○ Drugs altering clotting parameters - eg, anticoagulants, selective serotonin
reuptake inhibitors (SSRIs), corticosteroids.
○ Alternative remedies when taken with hormonal contraceptives - eg, ginseng,
ginkgo, soy supplements, and St John's wort.

Answer is : E

280) Squamous cell carcinoma of the cervix will most likely metastasize to which of the
following sites?

A. Bone
B. Uterus
C. Kidney
D. Vagina.
E. Liver
Squamous cell carcinoma of the cervix spreads usually through cervical lymphatics in
sequential manner; via direct extension to vagina, uterus, parametrium, lower urinary tract,
uterosacral ligaments; distant metastases to aortic and mediastinal lymph nodes, lung,
bones, ovary (1%)

Answer is : D

281) All the following is true regarding Premature rupture of membranes, except:

A. It can cause umbilical cord compression.


B. Fern test positive indicates PROM
C. Antenatal corticosteroids is used in treatment
D. PROM is an indication for cesarean section.
E. ​Vernix or meconium can be visible.

PROM itself is not an indication for cesarean section, but it can increase the rate of cesarean
section delivery if PROM got complicated

Premature rupture of membranes (PROM)


● Definition: rupture of membranes occurring before onset of labor at term
● Epidemiology: ∼ 15% of pregnancies
● Risk factors :
○ Ascending infection
○ Cigarette smoking
○ Multiple pregnancy
○ Previous preterm delivery
○ Previous PROM
● Complications :
○ Umbilical cord prolapse
○ Placental abruption
○ Chorioamnionitis

Diagnosis
● Sterile speculum examination
○ Positive pool: amniotic fluid exiting the cervix and pooling in the vaginal fornix
○ Detection of amniotic fluid: during sterile speculum examination
■ Litmus test or nitrazine test: test strips turn blue
■ Positive fern test: fern pattern on glass slide
■ Positive IGF1: IGF1, normally present in amniotic fluid, appears in the
cervix if membranes rupture.
● Ultrasound: Oligohydramnios may be present.

Answer is : D
282) In ​vertex presentation, the most common position is :

A. ​Right occiput anterior


B. ​Left occiput anterior
C. ​Left occiput posterior
D. ​Occipitoanterior
E. ​Occipitoposterior

Vertex presentation with longitudinal lie:


○ Left occiput anterior (LOA)—the occiput is close to the vagina (hence known as
vertex presentation), facing anteriorly (forward with mother standing) and toward the
left. This is the most common position and lie.
○ Right occiput anterior (ROA)—the occiput faces anteriorly and toward the right. Less
common than LOA, but not associated with labor complications.
○ Left occiput posterior (LOP)—the occiput faces posteriorly (behind) and toward the
left.
○ Right occiput posterior (ROP)—the occiput faces posteriorly and toward the right.
○ Occipitoanterior—the occiput faces anteriorly (absolutely straight without any turning
to any of the sides)
○ Occipitoposterior—the occiput faces posteriorly (absolutely straight without any
turning to any of the sides)
Answer is : B

283) Which of the following Can’t be used for treatment of gestational diabetes ?

A. Gliclazide
B. Insulin
C. Glyburide
D. Exercise
E. Metformin

Gliclazide is a sulfonylurea that is contraindicated during pregnancy.


Gestational diabetes causes impaired glucose tolerance, diagnosed during pregnancy;
associated with an increased risk of maternal and fetal morbidity

Treatment
● Glycemic control
○ Dietary modifications and regular exercise (walking)
○ Strict blood glucose monitoring (4x daily)
○ Insulin therapy if glycemic control is insufficient with dietary modifications
○ Metformin and glyburide in patients who refuse insulin therapy
● Regular ultrasound to evaluate fetal development
● Consider inducing delivery at week 39–40 if glycemic control is poor or if
complications occur

Answer is : A

284) Which of the following statements is true regarding toxoplasmosis ?

A. Immunocompetent patients usually do not require treatment.


B. Positive IgG test within the first week of infection.
C. X-ray is used for diagnosing cerebral toxoplasmosis.
D. 90% of affected have bilateral cervical adenopathy.
E. 4th generation cephalosporins are effective against the disease.

Toxoplasmosis is a disease caused by the obligate intracellular parasite Toxoplasma gondii.


Transmission occurs either through ingestion of cysts found, for example, in raw meat or cat
feces, or from mother to fetus through the placenta.

Clinical features
● Incubation time: 3 days to 3 weeks
● Immunocompetent patients
○ Mainly asymptomatic (90% of cases)
○ Symptomatic (< 10% of cases): mononucleosis-like symptoms with bilateral
cervical adenopathy
○ Lifelong immunity following infection
● Immunosuppressed patients
○ Cerebral toxoplasmosis (the most common neurological AIDS-defining
illness)
○ Ocular toxoplasmosis

Diagnostics
● Serology
○ IgM antibody test: positive within the first week of acute infection
○IgG antibody test: positive ​2 weeks following infection​ and remains positive
for life
● CT/MRI​ of the brain for suspected cerebral toxoplasmosis

Cephalosporins are not used in the treatment.

Answer is : A

285) Which of the following types is the most common type of genitourinary prolapse?

A. Uterine prolapse.
B. Urethrocele.
C. Enterocele.
D. Cystourethrocele.
E. Rectocele.

Types of genitourinary prolapse


Prolapse can occur in the anterior, middle, or posterior compartment of the pelvis.

Anterior compartment prolapse


● Urethrocele: prolapse of the urethra into the vagina. Frequently associated with
urinary stress incontinence; other symptoms are infrequent.
● Cystocele: prolapse of the bladder into the vagina. An isolated cystocele rarely
causes incontinence and usually leads to few or no symptoms. However, a large
cystocele may cause increased urinary frequency, frequent urinary infections and a
pressure sensation or mass at the introitus.
● Cystourethrocele: prolapse of both urethra and bladder.

Middle compartment prolapse


● Uterine prolapse: descent of the uterus into the vagina.
● Vaginal vault prolapse: descent of the vaginal vault post-hysterectomy. Often
associated with cystocele, rectocele and enterocele. With complete inversion, the
urethra, bladder, and distal ureters may be included resulting in varying degrees of
retention and distal ureteric obstruction.
● Enterocele: herniation of the pouch of Douglas (including small intestine/omentum)
into the vagina. Small enteroceles are usually asymptomatic. Can occur following
pelvic surgery. The neck of the hernial sac is usually sufficiently wide to make
strangulation very rare. Can be difficult to differentiate clinically from rectocele but a
cough impulse can be felt in enterocele on combined rectal and vaginal examination.

Posterior compartment prolapse


● Rectocele: prolapse of the rectum into the vagina.
Cystourethrocele is the most common type of prolapse, followed by uterine prolapse and
then rectocele​.

Answer is : D

286) Which of the following statements is true about embryonic genital tract differentiation ?

A. The “default” sex in sexual development is female.


B. Paramesonephric ducts give rise to male reproductive system.
C. Mesonephric ducts is an embryonic structure that develops into the female reproductive
system.
D. Urogenital sinus is the precursor of the upper third of vagina
E. Genital tubercle Give rise to the ovaries.

Mesonephric ducts is an embryonic structure that develops into the male reproductive
system during fetal development..

Mullerian ducts (Paramesonephric ducts) is a pair of embryonal ducts that give rise to the
fallopian tubes, uterus, cervix, and upper one-third of the vagina in a female fetus.

Embryology of vagina and vulva :


● Paramesonephric ducts (Müllerian ducts) → upper third of the vagina
● Urogenital sinus → ​lower two-thirds of the vagina​, vaginal vestibule, vestibular glands
● Urogenital folds → labia minora
● Labioscrotal swelling → labia majora
● Genital tubercle → clitoris

Answer is : A

287) All the following are indications for anti-D administration to rh-negative mother, except:

A. Braxton-Hicks contractions.
B. Placenta previa
C. Normal delivery
D. External cephalic version
E. Stillbirth

Braxton-Hicks contractions do not cause bleeding in which the fetal blood is presented to the
mother’s immune system.
Anti-D immunoglobulin (RhoGAM)

Background :
● Anti-D prophylaxis protects children in future pregnancies
● Only indicated in unsensitized mothers​ (the fetus of sensitized mothers can no longer
benefit from anti-D prophylaxis)

Potential sensitising events in pregnancy


● Invasive prenatal diagnosis - eg, amniocentesis, chorionic villus biopsy.
● Antepartum haemorrhage.
● External cephalic version of the fetus (including attempted).
● Ectopic pregnancy.
● Evacuation of molar pregnancy.
● Intrauterine death and stillbirth.
● Intrauterine procedures (eg, insertion of shunts, embryo reduction).
● Miscarriage or threatened miscarriage after 12 weeks of gestation.
● Therapeutic termination of pregnancy.
● Delivery - normal, instrumental or caesarean section.

Answer is : A

288) Which of the following is not a component of “modified bishop score” for labor induction
?

A. Cervical consistency.
B. Cervical effacement.
C. Fetal station.
D. Cervical dilatation.
E. Fetal presentation.

Modified Bishop score


● Used to assess the cervix and the likelihood of a successful induction
● Interpretation
○ Bishop score ≥ 8 → favorable cervix for vaginal delivery
○ Bishop score ≤ 6 → unripe or unfavorable cervix; not ready for vaginal
delivery
○ Simplified Bishop score: considers only fetal station, cervical dilation, and
cervical effacement; a score ≥ 5 indicates a favorable cervix for vaginal
delivery
Answer is : E

289) Which of the following is considered ideal presentation ?

A. Occiput anterior position.


B. Face presentation
C. Brow presentation
D. Complete breech
E. Shoulder presentation

Types of presentations :
● Occiput anterior position: Fetal occiput points towards maternal symphysis pubis;
fetus faces downwards.
○ Right occiput anterior (ROA): Fetal back faces the maternal right, anterior
fontanelle faces the maternal left, sagittal suture lies in the left oblique
diameter; (most common position).
○ Left occiput anterior (LOA): Fetal back faces the maternal left, anterior
fontanelle faces the maternal right, sagittal suture lies in the right oblique
diameter.
● Occiput posterior position: Fetal occiput points towards the maternal sacral
promontory with face to pubis symphysis; the fetus faces upward
● Sacrum in breech presentation
● Mentum (chin) in extended cephalic (face) presentation

The Occipito-Anterior position is ideal for birth - it means that the baby is lined up so as to fit
through the pelvis as easily as possible. The baby is head down, facing the spine, with its
back anterior. In this position, the baby's chin is tucked onto its chest, so that the smallest
part of its head will be applied to the cervix first. The position is usually "Left Occiput
Anterior", or LOA. Occasionally, the baby may be "Right Occiput Anterior", or ROA.

Answer is : A
290) The most likely scenario to perform vaginal delivery is which of the following ?

A. Mono-amniotic twins
B. Cord prolapse .
C. Brow presentation
D. One breech in twin pregnancy.
E. Fetal weight is greater than 4500

About 30-40% of twin pregnancies result in only one baby being in the breech position. If this
is the case, the babies can be born vaginally. After the first baby who is not in the breech
position is delivered, the baby who is presented in the breech position may turn itself around,
if this does not happen another procedure may performed called the breech extraction.

Some contraindications for normal vaginal delivery

Brow presentation
● This may convert to face or vertex presentation and may be managed expectantly.
● If the patient is unstable or no conversion occurs, cesarean delivery is
recommended.

Twin pregnancy
If a non vertex second twin presentation occurs, it is managed according to gestational age,
maternal preference, and practitioner comfort. The exceptions to vaginal delivery include the
following:
● Presenting twin in breech position
● Conjoined twin anatomy
● Most cases of ​mono-amniotic twins
● Signs of fetal distress or an abnormality that warrants abdominal delivery

Macrosomia
● Fetal weight greater than 4000-4500 g is associated with a higher risk of shoulder
dystocia and birth trauma during vaginal delivery.
● Mothers with diabetes have a higher incidence of macrosomia and risk of shoulder
dystocia.
● If the estimated fetal weight is greater than 4500 g in a mother with diabetes, ACOG
recommends abdominal delivery.
● If the mother does not have diabetes, abdominal delivery is not recommended until
an estimated fetal weight of 5000 g.

Answer is : D
291) All the following are complications of oligohydramnios, except:

A. Craniofacial abnormalities.
B. Umbilical cord compression.
C. Intrauterine growth restriction.
D. Achondroplasia.
E. Chorioamnionitis.

Oligohydramnios is when the amount of amniotic fluid < 500 mL in the third trimester

● Complications
○ Intrauterine growth restriction (diminished mobility of the fetus)
○ Intrauterine compression and decreased amniotic fluid ingestion → Potter
sequence: pulmonary hypoplasia (cause of death due to severe neonatal
respiratory insufficiency), craniofacial abnormalities, limb anomalies
○ Birth complications (e.g., umbilical cord compression)
○ Chorioamnionitis (more common in case of PROM)

Achondroplasia is caused by a genetic mutation.

Answer is : D

292) 51 y/o woman is concerned if she entered the menopausal period, the most important
clinical feature in menopausal woman is:

A. Atrophic vaginitis
B. Loss of lipido
C. Irregular menses
D. Bleeding per vagina
E. Amenorrhea

Menopause is the time at which a woman permanently stops menstruating, usually between
45 and 55 years of age, and is diagnosed after 12 months of amenorrhea.

So, amenorrhea is the hallmark for menopause.

Clinical features
The onset and intensity of symptoms is dependent on the phase of menopausal transition.
● Irregular menses (which gradually decrease in frequency) → complete amenorrhea
● Autonomic symptoms
○ Increased sweating, hot flashes, and heat intolerance
○ Vertigo
○ Headache
● Mental symptoms
○ Impaired sleep (insomnia and/or night sweats)
○ Depressed mood or mood swings
○ Anxiety/irritability
○ Loss of libido
● Atrophic features
○ Breast tenderness and reduced breast size
○ Vulvovaginal atrophy
■ Atrophy of the vulva, cervix, vagina (thin, pale, smooth epithelial layer,
associated with vaginal dryness, pruritus, and dyspareunia)
■ May present with features that mimic a urinary tract infection (i.e.,
dysuria, urinary frequency and urgency)
● Weight gain and bloating

Answer is : E

293) 23 y/o Multipara presented to the delivery center, after a bloody show and rupture of
membranes, she is now at 6 cm dilation, it took her more than two hours to reach 7 cm
dilation, what best describes her labor ?

A. Obstructed labor.
B. Prolonged latent phase.
C. Arrested labor.
D. Prolonged active phase.
E. Normal latent phase.

Prolonged latent phase


● Diagnosis: poor acceleration phase with a cervical dilation ≤ 5 cm
○ > 20 hours in a primipara
○ > 14 hours in a multipara
● Management
○ Rest, hydration, and adequate analgesia
○ Oxytocin may be considered in well-rested mothers if the other measures
have been implemented.

Prolonged active phase


● Etiology: abnormalities of the 3 P's of labor
● Diagnosis: ≥ 5 cm cervical dilation without adequate dilation (< 1 cm/2h)
● Management
○ Augmentation with oxytocin for hypotonic contractions
○ Analgesia for hypertonic contractions

Arrested active phase


● Etiology: abnormalities of the 3 P's of labor
● Diagnosis: ≥ 5 cm cervical dilation with ruptured membranes and no cervical change
for ≥ 4 hours if adequate contractions are present; or no cervical change for > 6
hours if only inadequate contractions are present
● Management: cesarean section

Answer is : D

294) All the following is true about imperforate hymen, except:

A. Incidence 1/2000 females.


B. It’s both congenital and acquired disorder.
C. May present with urinary frequency.
D. A type of primary amenorrhea.
E. Only clinical diagnosis can be enough.

The disorder can only be congenital.

Imperforate hymen
● Definition: a hymen without an opening
● Etiology: congenital defect
● Incidence: 1/2000 females
● Pathophysiology: central cells of the Müllerian eminence in the urogenital sinus do
not disintegrate → imperforate hymen → cryptomenorrhea at puberty (outflow tract
obstruction leads to backup of menstrual blood) → hematocolpos
● Clinical features
○ Asymptomatic before puberty
○ Primary amenorrhea with periodic lower abdominal pain
○ Possible urinary retention, frequency, dysuria
○ Possible palpable lower abdominal mass
○ Perineal examination: tense, bulging, bluish membrane in the vulva
● Diagnostics: primarily a clinical diagnosis
○ Imaging may be conducted to rule out transverse vaginal septum.

Answer is : B

295) Which of the following can cause variable decelerations on CTG ?

A. Maternal fever.
B. Placental insufficiency.
C. Fetal head compression.
D. Pulmonary hypoplasia.
E. Umbilical cord prolapse.
Variable deceleration
○ Rather variable presentation and temporal relation to the contractions changes
○ Onset to nadir is abrupt (< 30 sec) and lasts at least 15 sec
○ Causes
■ Umbilical cord compression/prolapse (see umbilical cord complications
below)
○ Measures
■ If intermittent variable decelerations (< 50% of contractions): usually no
interventions are needed
■ If recurrent variable decelerations (≥ 50% of contractions)
■ Intrauterine resuscitation
■ If FHR pattern does not improve despite intrauterine resuscitation →
emergency c-section

Answer is : E

296) All the following are risk factors of uterine atony, except:

A. Mother exhaustion.
B. Multiparity.
C. Preterm labor.
D. Instrumental delivery.
E. Large for gestational age newborn.

Uterine atony
● Definition: failure of the uterus to effectively contract and retract after complete or
incomplete delivery of the placenta, which can lead to severe postpartum bleeding
from the myometrial vessels
● Epidemiology: responsible for ∼ 80% of PPH cases
● Risk factors: overdistention of the uterus
○ Multiparity
○ Multiple pregnancy
○ Post-term pregnancy
○ Instrumental delivery
○ Anatomical abnormalities (i.e., fetal, uterine, abnormal placental implantation)
○ Large for gestational age newborn (e.g., > 4000 g)
○ Poor myometrial contraction following prolonged or rapid and forceful birth
● Clinical findings
○ Abnormal vaginal bleeding
○ Soft, enlarged (increased fundal height), boggy ascending uterus

Answer is : C
297) 26 y/o woman presented with dysmenorrhea which were also the complaint in the last
few months, she and her partner are trying to have a baby a few months ago but all failed,
dysmenorrhea accompanied with appearance of blood in stool, the most likely diagnosis is:

A. PCOS
B. Colon cancer.
C. Ulcerative colitis.
D. Endometriosis.
E. Rectovaginal fistula

The symptoms of this lady can be present in Endometriosis where endometriosis found in
the bowel.

Rectovaginal fistula is not accompanied with infertility or sub-infertility .

Answer is : D

298) 51 y/o lady presented presented for a gynecological check, a pap smear was taken, the
results shows Low grade intraepithelial lesion, HPV test is negative, what the next step in
management ?

A. LEEP.
B. Colposcopy.
C. Repeat smear and HPV test in 12 months.
D. Cone biopsy.
E. No further action to be made.

According to the bethesda system:

The Bethesda system is used for classifying cytological results. Management guidelines
exist for each determined subtype.

If pap smear came positive for Low-grade squamous intraepithelial lesion


● 21–24 years
○ Repeat cytology at 12 months
■ If ASC-H or higher→ colposcopy
■ If negative twice → routine screening
○ Alternatively, test for high-risk HPV
■ If positive → repeat smear and HPV test in three years
■ If negative → routine screening
● 25–29 years: colposcopy
● ≥ 30 years
○ HPV status negative → repeat smear and HPV test in 12 months
■ If negative → repeat smear and HPV test in 3 years
■ If positive → colposcopy
○ HPV status positive or unknown → colposcopy
Answer is : C
299) All the following are true about Germ cell tumors in females, except:

A. Cystic teratoma tumor contains any type of tissue.


B. Dysgerminoma is the most common malignant ovarian tumor in young women
C. Yolk sac tumor causes rise in serum alpha fetoprotein.
D. Choriocarcinoma is very rare.
E. Struma ovarii can cause hypothyroidism.

Germ cell tumors


● 15–25% of all ovarian tumors
● Teratoma
○ Mature
■ Dermoid cysts: most common of all germ cell tumors (90% of cases)
■ Malignant transformation in 2% of cases
■ Can theoretically contain any type of tissue, e.g., hair, teeth,
and sebaceous glands, but mostly include parts of ectodermal
origin
■ Differentiated, mostly benign tumor
■ Struma ovarii: teratoma with endodermal differentiation into thyroid
tissue
■ Very rare: malignant transformation into a thyroid carcinoma
■ May produce thyroxine and cause ​hyperthyroidism ​symptoms
■ Differentiated, mostly benign tumor
○ Immature:
■ Rare, undifferentiated
■ May contain tissue of embryonic/fetal period
■ High risk of malignancy
● Dysgerminoma: most common malignant ovarian tumor in young women (20–30
years); female histological equivalent to the male seminoma
● Yolk sac tumor of the ovary: often malignant; occurs mainly in childhood and
adolescence
● Non-gestational choriocarcinoma: rare and extremely malignant; normally
accompanied by beta hCG production

Answer is : E

300) 59 years old woman complains of sudden uncontrollable urination, this condition puts
her in an embarrassing situation, Which of the following is better treatment :

A. oxybutynin.
B. Tamsulosin.
C. scopolamine.
D. Cephalosporins.
E. Imipramine.
Urge incontinence is a form of urinary incontinence is characterized by a sudden urge to
urinate, resulting in involuntary leakage of urine.

Treatment :

Conservative measures
Conservative measures should first be attempted before considering medical treatment.

Non Pharmacological treatment


● Behavioral therapies, exercises, lifestyle medications (see general principles of
treatment of urinary incontinence)

Medical treatment
● First-line are anticholinergics, including ​oxybutynin​, tolterodine, solifenacin,
trospium, and darifenacin
○ Effect: competitive blockage of acetylcholine at muscarinic acetylcholine
receptors → parasympathetic effect is impaired → decreased overactivity of
the detrusor muscle → reduced voiding
○ Adverse effects: dry mouth, tachycardia, glaucoma
● Alternatives or combined administration
○ Alpha blockers: e.g., tamsulosin
○ Additional spasmolytic agents: e.g., scopolamine (hyoscine) hydrobromide,
flavoxate
○ Tricyclic antidepressants (anticholinergic): e.g., imipramine
○ Beta-3 agonists

Second-line treatment
● Endoscopic injection of botulinum toxin at different points in the bladder wall
● Sacral nerve stimulation
● Last resort
○ Augmentation cystoplasty

Answer is : A

You might also like